You are on page 1of 452
Cost Flow Introduction ‘¥Control account: is found in general ledger and include aggregate of many sub - homogeneous accounts. EX. inventory control account. Cost flow- General Fact Pattern Lippy Company's beginning and ending inventories for the month of November are as follows: Description ‘| November, | November,30 | Stores control (D.M) $67,000 $62,000 Indirect materials control 10,000 5,000 Work-in-process control(WIP) | —«*145,000-—~—=«|~—S—«*71,000-—=*| Finished goods control (F.G) 85,000 78,000 Other data for the month of November follows: Description ‘Amount Direct materials purchased on credit 163,000, Transportation in 4,000 Purchase returns and allowances | 2,000 Indirect materials purchased during November | 45,000 Direct labor (DL) | $200,000 “Tadirect labor | 45,000 | Actual factory overhead (actual factory O.H) (see details later) | 132,000 ¥Lippy uses two factory overhead accounts, one for actual O.H (called factory O.H control) and the other for O.H applied and charges factory overhead to production (DR-> WIP & CR > factory O.H applied) at 70% of direct labor cost (D.L is considered the factory O.H cost driver in this case). The company does not formally recognize over/under applied factory overhead until year-end. vAssume also sales revenue during November amounted to $750,000 also marketing and selling expenses $80,000 detailed later, General and Administration Expenses $29,000 detailed later ¥Assume also interest expense $50,000 and income tax rate is 30% Description Factory | Marketing | Administrative | Total O.H | andselling | Expenses | expenses Utilities expense- heat, light and air 2,750 3,000 1250 conditioning. ..etc (Semi variable cost) Depreciation expense-equipments and vehicles | 13,400 3,000 9,600 46,000 (Fixed cost) | Depreciation expense- buildings(Fixed cost) 5,600 | 4,000 400 10,000 Property taxes and insurance expenses | 2.250 7,000 2,750 12,000 | {Fixed cost) | Fringe benefits (Semi Fixed cost) - 26,000 7,000 33,000 Maintenance and repairs expense 4500 3,500 7,800 9,800 | Rent expense (Fixed cost) 6500 | 3,000 700 10,200 | Power (Variable cost) 2,000 300 soa” | 3,000 Wages and salaries (other than factory) - 10,000 3,000 15,000 Total 37,000__| 80,000 _ | 29,000 146,000 Direct labor are wages paid to machine operators only, but factory supervisor's salary is considered indirect labor and included in factory O.H. Also wages paid to all production service workers like machinery mechanics are considered indirect, labor and included in factory OLE. Direct labor costs are assumed to be variable, unless otherwise stated specifically. ‘YDirect Costs: are costs that can be feasibly and specifically identified with specific cost object. Accordingly: A) Direct materials used and direct labor both are direct costs because they can be specifically identified with the production of finished goods. B) Direct materials used and direct labor costs can be associated with specific product (traceable) hence, D.M. used and D.L. costs are product costs, ) Prime Costs = D.M. used + Direct Labor costs Thus both D.M. used and D.L costs are Variable costs, direct costs, product costs and prime costs vFactory O.H. includes all indirect costs (both variable components and fixed components) related to manufacturing activity and total factory O.H is considered [Pavey product cost under full absorption costing approach, while fixed factory O.H is considered period cost under variable costing approach. ¥On the other hand total marketing costs such as salaries of sales personnel, commissions, advertising and total administrative expenses are period costs and are expensed in income statement as incurred, ¥Cost of D.M. used = Beg. D.M. 67 Plus Net D-M. pure Ending DM. 62= 170 sed (163 + 4-2) Minus ¥ Prime cost = D.M used 170 Plus D.L 200 = 370 ¥Conversion cost (C.C) = D.L 200 Plus Factory O.H applied 140 =340 ‘Total Factory O.H applied to production = $200 * 70% = $140 curred 13; ‘vOver applied O.H = O.H applied 140 — actual O.H $8 ‘yManufacturing costs during the period =D.M used 170 + D.L 200 + Factory O.H applied 140 = 510 vYotal manufacturing costs (T.M.C) = BWIP 145 + Manufacturing costs during the period 510 = 655 ¥ Cost of goods Manufactured (COGM) = Total manufacturing costs (T.M.C) 655 Minus EWiP 171 = 484 '¥ Cost of goods available for sale = beginning F.G 85 + COGM 484 = 569 ¥ Cost of goods sold (COGS) = Cost of goods available for sale569 - Ending F.G inventory 78 = 491 ¥ Gross profit (gross Margin) = Sales revenue 750 - COGS 491 = 259 | {209498 00'16r ‘aw 49 so ua 000'16y 1orUoAU spooy paYystuLA AD 000't6r 900 ua ‘000‘T6r =000'82 54a - O00'r8r Povied ayp FuuNp ‘PUsiuy spo08 Jo 1509 + 000'SE D's'd = S904 000°01SS = 000'0F! 000'8s = H'O patidde 4940, 00°F ES = %40L, x 000°00ZS = Uonanpoad 03 payidde "9 A4oi9Kg [OL 170 Aone + 00°00T'G + 000°DLI past dim € ud ouied ayy Bupinp 109 Bu ooo'or! | “Ta wo pasta 3D. Doudde HO Moe aa 000'OFES = 000'OY! paudde 10 Aiowe + 000°007 “Td = $1809 woyssaauo5 (7 ysRO Qoo'zers | pure dap" soy ~ a/v ‘ooo'oces = | 000'002 Ta + doo'OLI pasn Wra=sso9 amid (1 | 4D 1021005 “H'O {10198 a ‘oorooes | 101 | O00 00es TOL 00'0%1 paddy “}'0 Aioi0e4 “49 ‘o00'002 fenues “rq 4D, 00'0ut jonuos Azowaau wa 4p | [00000 | aIM € wa | o0oOZ |_saHEM BONG covoss = ai sa] [I Toru05 “Ta ed wos Doss ‘o0's Suypwa_|-ooNs ‘oo0'os ee a TOAIWOS SHOVE TTTPUT 7a 1. Statement of cost of goods manufactured (COGM) ‘Amount ©) Item Subtotal Total Beg. DM 67,000 Classification Add Net D.M. purchased 165,000 Total D.M. available for use 232,000 Less Ending D.M. inventory (62,000) * D.M. used during the month 170,000 Product Cost * Direct labor 200,000 Product Cost * Factory O.H. applied to production | 140,000 Product Cost Manufacturing cost during the month 510,000 Add BWIP 145,000 Total manufacturing costs (T-M.C) 655,000 Less EWIP (471,000) Cost of goods manufactured (C.G.M) | 484,000 2. Statement of cost of goods sold (COGS) aml ‘Amount ($) Item i Subtotal Total Cost of goods manufactured (C.G.M) 484,000 Add Beg. Finished goods 85,000 Costs of goods available for sale 569,000 Less Ending finished goods (78,000) Cost of Goods Sold (COGS) (491,000) 3. Full absorption income statement for the month of November Description Subtotal Total Net sales 750,000 Less Cost of Goods Sold (COGS) 491,000 Gross profit (gross margin) 259,000 ‘Less Marketing and selling expenses 80,000 Less General and administrative expenses 29,000 Total operating expenses (109,000) ~~ Operating income (EBIT) 150,000 ‘Less Non-operating expenses (interest) (50,000) EBT 100,000 Less Income tax (30%) (30,000) Net income 70,000 00'SPL —_foanHOD 14°09 AzoHBy YD. 09's 900 4a “as09 [eV0y ‘wt ody 0% ays 40 (a[qusayoud Ayjeonaaoays) PHayIAAO [810 008 anaaa | J0.% 9) 40 @lqusayoad Aqpeapozooy)) Praytado [e101 000'Or! paydde pO KioiE YG 30 % au} Joye Wo paseq aq eu yuouUoNsodde sty], 000°8hI [o4pU0d “H'O AtoveI “AD. H’0 A209) Jo uonvanddy: 000° $900 Wa |__42puin yueaysis 25015 0} 000'0rL pondde H’o A101} A pondde r9pun HT (a voo'es | %6001 | ooo00't | re104 ooo'tz mot | 00'00e dIMa YD joauos wo Gene 2, [oot [00's 00z | 00°00 paudde Ho oe ya | —008S | 000'eS 001 | 000001 000"0E $909 49 000'021 YoaIu09 HO 40198) AD 110 &107904 Jo uoneonddy pauddy paddy KO0'DS! Paddy HG Kore Na. =sepun AO % | S)anea | suiayy Jo SouR|Eq HUIpUD TUNSEY =a PIMEITOT (Vv —_wr—e Pouyaut syo-ap11M apeIpoMUY payyea— “S900 $909 © Apaaatp pasoys aq pinoys aoueyeq ay, pur A10juDAut “D-y BUIPUD ‘CIM Jo SPOULTEG ay) BuOWE pareio.d aq Pinoys souLING Oy | Smorjoy st pasopp 2q snuu “f° A10}94} Jo SoU] “4D 40 “4G Pua EDK IY 00's so aaueqed aqap sey “H'O Lovey 000'0E Jo sourpeq y1p949 sey “HO L1o}ET 000°8P | ‘00'S TeEmay Vv a wos | 30 Toniw05 WO SORE pordde - sopug, paytdde - 1940, pua avad 38 "ECO A1oyaegy Butsopy, sunowre ap Jo souvayzustg ay) uodn spuadap aus yp fsnyy $509 01 poauIp paxaysteN UaYo st 2auEjEG a1NUE ap “UOALIP AARDe pue 1S09 Jo areUTISS aredordde ay pue SIaAEIC] areLidosdde ip Bujsooyo Surunsse) fea TUN AITEASH st NOME dup as 94 “INg] -AIOWAAUE TAX PUL ‘ArojUaAU spoo8 paystUY “SOOO Suowe uoneoo|(2 atp se punos <{jenidsouoo se 10u st PoMpaut Jjo-ayaM ayespouUN axp {pa4aajoad sy UOPEDOE ayp ‘A4peOHaA091 “AIOUOAL! gIA\A pur ‘101UaAu Spoos paystUy ‘SDOD 01 parwaoyqE 40 (Siyatiag amp paaoxa Aeus yf aredo]]e ot 1809 ay asTVeI0) [ULLA,BUN JOU St JUNOWE ay 31 Aqu0 SOOO 01 JoNNIe eouE,eG OU SuMLAYStIEN Aq wos YP Jo pud atp 18 Jo pasodsip 9q 1snUw PeoyZoro Aso198y por|ddexepun 10 patjdde1eng, sjunoure axp so sounayfudis ay) uo spuadag panddesado 10 pay{ddesapun Jo uoRysodstp amp 205 poyjow aywudosdde ayy Cost flow- Questions is available for the Crites Manufacturing Company for the Essay: The following inform: year ended December 31, 2012: Deseription ‘Amount Sales $325,000, Work in process, January 7 19,000 Work in process, December 31 15,600 Direct materials inventory, January 1 21,400 Direct materials inventory, December 31 19,800 Finished goods inventory, January 1 36,700 Finished goods inventory, December 31 13,600 Direct materials purchased 32,400 Direct labor 52,000 Supervisory and indirect labor 27,300 ‘Administrative salaries 43,000 Supplies and indirect materials 5,400 Fleat, Tight, and power (76% for manufacturing plant) 31,600 ‘Amortization (80% for manufacturing plant) 37,500 Property taxes (75% for manufacturing plant) 9,900 Administrative costs 10,950 Marketing costs 42,600 Required Prepare an income statement with a supporting schedule of cost of goods manufactured and ‘cost of goods sold. Schedule of cost of goods manufactured and cost of goods sold ‘Description Subtotal Total Direct materials inventory, January 1 21,400 ‘Add Direct materials purchased 32,400 Direct materials available for use 53,800 Less Direct materials inventory, December 31 (49.800) Direct materials used 34,000 Direct labor 52,000 Supplies and indirect materials 3,400 ‘Supervisory and indirect labor 27,300 Heat, light, and power (70% * 31,600) 2,120 Amortization (80% * 37,500) 30,000 Property taxes (75% * 9,900) 7.425 “Total manufacturing O.H 92,245 “Manufacturing cost during the period 178,245 ‘Add Work in provess, January 1 19,000 ‘Total Manufacturing cost 197,245 L Less Work in process, December 31 (05,600) COGM 181.645, ‘Add Finished goods inventory, January 1 36,700 Cost of goods available for sale 218,345 Less Finished goods inventory, December 31 (13.600) cogs 204,745, Income statement Deseription ‘Subtotal Total Sales $325,000 Less COGS. (204,745) ‘Gross profit 120,255 ‘Less operating expenses Marketing costs, 72,600 ‘Admainistrative costs 10,950 ‘Administrative salaries 43,000 Heat, fight, and power (30% x 31,600) 9,480 ‘Amortization (20% * 37,500) 7,500 Property taxes (25% * 9,900) 2.475 “Total operating expenses (116,005) EBIT 4,250 Essay: The following information pertains to Stoney Manufacturing for 2012: Description ‘Amount Direct labor $30,000, Sales 400,000 Selling expenses 50,000 Raw (Direct) materials on hand: January 1 8,000 December 31 4,000 “General and administrative expenses 18,000, Finished goods: January | 25,000 Work in process January 1 19,000 December 31 18,000 Direct material purchases 47,000 Depreciation 20,000 Indirect labor 3.000 a Indirect materials used 7,000 Marketing promotions 1,500 Factory taxes 11,000 Utilities 20,000 Courier costs (office) 900 Miscellaneous plant overhead 4,000 Plant repairs and maintenance 9,000 CC Customer service costs 3,000 Fire insurance: factory equipment 3,000 Materials handling costs 3,000] Additional Information: A. The gross profit margin is 73.25% ion is charged to production at 70% 1. Prepare a schedule of cost of goods manufactured for the year ended December 31 2. Prepare a schedule of cost of goods sold 3. Prepare an income statement for the year ended December 31. 10 Schedule of cost of goods manufactured Description Subtotal Total Direct materials inventory, January 1 8,000 ‘Add Direct materials purchased 47,000 Direct materials available for use 35,000 Less Direct materials inventory, December 31 (4,000) Direct materials used 31,000 Direct labor $30,000 Indirect materials used 7,000 Indirect labor 3,000 Factory taxes 11,000 Fire insurance: factory equipment 3,000 lant repairs and maintenance 9,000. Materials handling costs 8,000 Utilities 20,000 » 90% 18,000 Factory Depreciation 20,000 * 70% 14,000 ‘Miscellaneous plant overhead 4,000 Manufacturing O.H 77,000 ‘Manufacturing cost during the period 138,000 ‘Add Work i process, January 1 19,000 ‘Total Manufacturing vost. 177,000 ‘Less Work in process, December 31 (118,000) ‘cOGM 159,000 ‘Schedule of cost of goods Sold Description Subtotal Total Finished goods inventory, January 1 25,000 ‘Add COGM 159,000 Cost of goods available for sale 184,000 Less Finished goods inventory, December 31 (77,000) ‘COGS 400,000 * (100 % = 73.25%) 107,000 Tncome statement Description Subtotal Total Sales $400,000 Less COGS (307,000) Gross profit 293,000 Less Operating expenses Selling expenses 50,000 Customer service costs 3,000 General and administrative expenses 18.000 Courier costs (office) 900 Marketing promotions 1,500 Utilities 20.000 = 10% 2,000 Depreciation 20.000 « 30% 6,000) W Total Operating expenses | “@i4007 EBIT 211,600 ceview questions QI: Management accounting differs from financial accounting in that financial accounting A. Is more oriented toward the future. B. Primarily concerned with external financial reporting, C. Concemed with non-quantitative information D. Heavily involved with decision analysis and implementation of decisions. Answer (A) is incorrect because Management accounting is future oriented. Answer (B) is correct. Financial accounting is primarily concerned with historical accounting, i.e, traditional financial statements, and with external financial reporting to creditors and shareholders. ‘Management accounting applies primarily to the planning and control of organizational operations, considers non-quantitative information, and is usually less precise. Answer (C) is incorrect because financial accounting is primarily concerned with quantitative information. Answer (D) is incorrect because Decision analysis and implementation are characteristics of management accounting. Q2: Which one of the following is least likely to be an objective of a cost accounting system? A. Product costing, B. Department efficiency. C. Inventory valuation. D. Sales commission determination. Answer (A) is incorrect because Product costing is an objective of a cost accounting system. Answer (B) is incorrect because Department efficiency is an objective of a cost accounting system. ‘Answer (C) is incorrect because Inventory valuation is an objective of a cost accounting system. Answer (D) is correet. A cost accounting system has numerous objectives, including product costing, assessing departmental efficiency, inventory valuation, income determination, and planning, evaluating, and controfling operations. Determining sales commissions is not an objective of a cost accounting system because such commissions are based on sales, not costs. Q3: All of the following would appear on a projected schedule of cost of goods manufactured except for A. Ending work-in-process inventory. B. Beginning finished goods inventory. C. The cost of raw materials used. DD. Applied manufacturing overhead. Answer (B) is correct Q4: The appropriate method for the disposition of underapplied or overapplied factory overhead. A. Isto CO C. Is apportioned to COGS and F.G inventory. only. B.4sto F.G inventory only. D. Depends on the significance of the amount. Answer (A) COGS, WIP, and F.G. Answer (B) is incorreet because whether the amount of underapplied or overapplied factory overhead incorrect because if the amount is Material (significant) it should be allocated among material or immaterial no such method for the disposition of it by closing the amount of underapplied or overapplied factory overhead to finished goods inventory only. Answer (C) is incorrect because a Material amount should be allocated among COGS, WIP, and FG Answer (D) is correet. overapplied or underapplied factory overhead must be disposed of at the end of an accounting period by transferring the balance either to COGS only if the amount is not material (because the cost to allocate it may exceed the benefits) oF allocated to COGS, F.G inventory, and WIP inventory, Theoretically, the allocation is preferred; the immediate write-off method is not as conceptually sound as the allocation among COGS, F.G inventory, and WIP inventory. But, because the amount is usually immaterial (assuming choosing the appropriate Drivers and the appropriate estimate of cost and activity driver), the entire balance is often transferred directly to COGS thus; the entry depends upon the Significance of the amount. QS: When the amount of overapplied factory overhead is significant, the entry to close overapplied factory overhead will most likely require A.A debit to COGS. B. Debits to COGS, F.G inventory, and WIP inventory. C. A credit to COGS only _D. Credits to COGS, F.G inventory, and WIP inventory. ited (not debited) Answer (A) is incorrect because cost of goods sold should be ei For its allocated portion of overapplied overhead. Answer (B) is incorrect because cost of goods sold, finished goods inventory, and work-in-process inventory should be eredited (not debited). Answer (C) is incorrect because, although commonly used, the immediate write-off method is not as conceptually sound as the allocation among cost of goods sold, finished goods inventory, and work- process inventory. Answer (D) is correct, Under a normal costing system, overkead is applied to WIP* for all jobs worked on, during the period at a predetermined rate. Because cost of goods sold, finished goods inventory, and work-in-process inventory all relate to those jobs, each shoutd be adjusted by its proportionate share of over-or underapplied overhead. This apportionment may be based on either the percentage of total overhead (theoretically preferable) or the percentage of total cost. The entry (0 close overapplied overhead requires credits to these three accounts. Q6: A review of the year-end accounting records of Elk Industries discloses the following information. Raw materials: $80,000, WIP: 128,000, Finished goods: 272,000, COGS: 1,260,000. ‘Also BWIP is $140,000. The Company's under applied overhead equals $133,000. On the basis of this information, Elk’s cost of goods sold is most appropriately reported as A. $987,000 B.$1,213,100 C.$1,218,000 _D. $1,358,000 Answer (D) is correct: COGS 1,260,000 included BWIP of $140,000 which includes its share of coverapplied or underapplied from last year, accordingly the amount of COGS that should allocated under applied overhead =1,260,000 - $140,000 = $1,120,000, thus COGS is most appropriately reported as $1,260,000 + 98,000 = $1,358,000 Item |__ Amount % Allocation Adjusted amount wip | _ 128,000 84% 11,200 | 139,200 FG | 272,000 11.9% 23,800 295,800 COGS 1,120,000 | 73.7% | 98,000 1,358,000 Total 1,520,000 | 100% | 133,000 1,793,000 Q7: The total direct labor cost of producing 100 units of Product X is $50, the direct Material cost of producing these 100 units is perfectly variable and the cost driver is the number of units produced. ‘The cost of the direct material traced to each unit is $1.25. Indirect costs are completely fixed at $75, for the production of these 100 units. What are the total conversion costs for these 100 units of Product X? A.8275 B, $250 c. $200 D.$175 E.$125 Answer (E) is correct Q8: If the beginning balance for May of the materials inventory account was $27,500, the ending balance for May is $28,750, and $128,900 of materials was used during the month, how much the materials purchased during the month ? (Beg. Balance | 27,500 | ‘Used. —|_—*128,900 MPurchased |? 1 Ending balance] 28,750 Toatenians psa | — 50 (7 | Sa) Beginning Balance + materials purchased = Materials used + ending Balance Materials purchased = materials used + ending Balance ~ Beginning Balance Materials purchased = $128,900 + $28,750 -$27,500 Materials purchased = $130,150 QO: The debits in WIP are BWIP, D.M used, D.L, and factory overhead applied. ‘The WIP account should be credited for production that is completed and sent to finished goods inventory. The balance i A. EWIP, which is a credit. B. EWIP, which is a debit. C. Total production costs to be accounted for. D. Closed out at the end of the accounting period. Answer (A) is incorrect because EWIP should never have a credit balance. Answer (B) is correct. The sum of the debits to WIP equals total production costs to be accounted for. Ignoring possible spoilage, production consists either of goods that have been completed or 4 those still in process. Accordingly, after the account is credited for the cost of goods completed and transferred to the F.G inventory, the debit balance in the account is EWIP. Answer (C) is incorrect because total produetion costs to be accounted for would be included in finished goods as well as EWIP. Answer (D) is incorrect because work-in-process is an asset (inventory) account, Asset accounts are real accounts that are not closed out at the end of the period Q10: The manufacturing (work process) account is A. Neither a real nor a nominal account. B. An inventory account indicating the beginning and ending inventory of goods being processed. CC. A hybrid account (both a real and a nominal account). D. A nominal account to which indirect costs are charged as incurred then these costs are Charged to production Answer (A) is incorrect because WIP is an inventory or real account, Answer (B) is correct. The manufacturing account is an inventory account to which D.M, D.L, and factory O.H costs are charged as they are incurred in the Production process (actual costing system) the sum of these costs plus the cost of BWIP is the Total production cost to be accounted for in any one period. The total is allocated to goods Completed during the period, ie. to finished good, and to EWIP. The Manufacturing Account may also be credited for abnormal spoilage. Answer (C) is incorrect because WIP is a real account. Answer (D) is incorrect because the factory overhead account, not the manufacturing account (WIP) pools the indirect costs as incurred and charges them to production systematically QU: During the current accounting period, a manufacturing company purchased $70,000 of raw materials (D.M and indirect materials), of which $50,000 of direct materials and $5,000 of indirect materials were used in production The Company also incurred $45,000 of total labor costs and $20,000 of other factory overhead costs. An analysis of the work-in-process control account revealed $40,000 of direct labor costs. Based upon the above information, what is the total amount accumulated in the factory overhead control account?” Dr ‘WIP Cr. M used $50,000 $40,000 O.H Applied? Dr Factory O/H Control Cr. Indirect Materials used | $5,000 Indirect wages $5,000 Other factory OH $20,000 Total actual F.O.H | $30,000 Q12: Many companies recognize three major categories of costs of manufacturing a product these are direct materials, direct labor, and factory overhead. Which of he following is an overhead cost in the production of an automobile company? A. The cost of small tools used in mounting tires on each automobile. B. The cost of the tires on each automobile. C. The cost of the labor that place tires on each automobile. D. The delivery costs for the tres on each automobile. Answer (A) is correct. The cost of small tools used in mounting tires can't be identified solely with the manufacturing of a specific automobile. This cost should be treated as factory overhead because it identifiable with the production process as a whole but not with the manufacturing of specific automobile. Answer (B) is incorrect because tires costs are seadily and directly identifiable with each automobile and, are direct materials costs. Answer (C) is incorrect because the cost of the labor that place tires on each automobile is readily and directly identifiable with each automobile. Hence, itis a direct labor cost. Answer (D) is incorrect because delivery costs are readily and directly identifiable with the tres delivered Thus, they are included in direet material costs. QUB: In practice, items such as wood secews and glue used in the production of school desks and chairs ‘would most likely be classified as A. Direct labor. B, Factory overhead, Direct materials. D. Period costs. Answer (B) is correct ‘Q14: Farber Company employs a normal (non-standard) absorption costing system. The following information is from the financial records of the company for the year. + Manufacturing costs during the period were $2,500,000 * Cost of goods manufactured was $ 2,425,000 + Applied factory overhead was 30% of manufacturing costs during the period. + Factory overhead was applied to production at a rate of 80% of direct labor cost. + BWIP was 75% of EWIP. Dr wiP Cr. BWIP (75% of EWIP) COGM. $2,425,000 — ‘Added _ (3) DM used $2,500 — [S750 + $937.3) 812,500 | (2) D.L= ($750,000 / 0.80) $937,500 | @ Factory O.FT applied ($2,500,000 * 30%) | $750,000 EWIP | 222299 | (4) 0.75 EWIP + $2,500,000 = $2,425,000 + EWIP 0.25 EWIP =875,000 sewer = $75.00 300,000 (5) BWIP ~ $300,000 *75% = $225,000 QI5: Morton Company’s manufacturing costs for 2012 were as follows: D.M $300,000 D.L 400,000 Factory overhead: Variable 80,000 Fixed 50,000 Prime costs totalled ‘A, $300,000 B. $380,000 C. $700,000 1D, $830,000 Answer (C) is correct. By definition prime costs are equal to direct materials plus direct manufacturing labor. Thus, prime costs are $700,000 ($300,000 DM + $400,000 DML). QU6: For a manufacturing company, which of the following is an example of a period rather than a product cost? A. Depreciation on factory equipment. _B. Wages of salespersons. C. Wages of machine operators. D. Insurance on factory equipment. ‘Answer (A) is incorrect because depreciation on factory equipment is an element of the product cost, manufacturing overhead. Answer (B) is correct. Salespersons’ wages are not associated with the production process and, therefore, would not be considered a product cost. The wages of salespersons are considered a period cost and are expensed in the period incurred. ‘Answer (C) is incorrect because wages of machine operators would be considered direct manufacturing, labor and therefore a product cost. Answer (D) is incorrect because insurance on factory equipment is an element of the product cost, manufacturing overhead, QU7: The variable portion of the semi variable cost of electricity for a manufacturing plant is a Choice Conversion cost Period cost | Yes No B Yes Yes c y No Yes No Answer (A) is correct. A cost that attaches to the physical units is termed a product cost. Product costs would include D.M, D.L, and manufacturing O.H. Conversion cost is the cost involved in converting the direct materials into a finished product. It is composed of D-L and manafacturing O.H. Any cost that does not attach to the physical units would be termed a period cost and would be expensed as incurred. Therefore, a cost is either a period or a product cost. Electricity cost, whether variable or fixed, would be included in manufacturing O.H and classified as conversion costs, and therefore can't be classified as a period cost. as: A. Places minimal emphasis on the cost of materials used in manufacturing a product. ‘onversion cost pricing B, Could be used when the customer furnishes the material used in manufacturing a product. C. Places heavy emphasis on indirect costs and disregards consideration of direct costs. D. Places heavy emphasis on direct costs and disregards consideration of indirect costs. Answer (A) is incomect because conversion cost pricing does not place any emphasis on raw material cost. C is the costs of converting raw Answer (B) is correct. C.C consists of D.L and factory O.H; materials into finished goods. Normally, a company does not consider the cost of D.M when applying CC pricing only C.C are considered in making pricing decisions, C.C pricing would be appropriate if the customer were to furnish the raw materials, Answer (C) is incorrect because DAL is an element of C.C. Answer (D) is incorrect because factory O-H is an indirect cost that is an element of C.C Q19: Given the following data for Scurry Company, what is the COGS? Beginning inventory of finished goods $100,000 Cost of goods manufactured 700,000 Ending inventory of finished goods 200,000 Beginning work-in-process inventory 300,000 Ending work-in-process inventory 50,000 A. 500,000 B. 600,000 €. 800,000 1D. 950,000 Answer (B) is correct Q20: Which one of the following items would not be considered a manufacturing cost? ‘A, Cream for an ive cream maker, B. Sales commissions for a car manufacturer. C. Plant property taxes for an ice cream maker. D. Tires for an automobite manufacturer. Answer (B) is correct Q21: Finley Painters Co., a painting contractor, maintains a job-order cost system. Job costs are accumulated by tracking the actual cost of paint and other materials used on each job, as well as the actual cost of wages earned by the painters on each job. In addition, overhead is applied to each job by using a predetermined rate based on the actual painters’ wages. Leonard Wayne, a painter earned $168 today by working on Job 97-45. In computing prime cost and conversion cost for Job 97-45, how would the wages earned today by Wayne be classified? A. Asa component of both prime and conversion cost. B. As a component of prime cost but not as a component of conversion cost. C. As a component of conversion cost, but not as a component of prime cost. D. As a component of neither prime cost nor conversion cost Answer (A) is correct Q22: Roberta is the manager of Sleep-Well Inn, one of a chain of motels located throughout the U.S. An example of an operating cost at Sleep-Well that is both direct and fixed is A. Johnson's salary. B. Water. C. Toilet tissue. D, Advertising for the Sleep-Well Inn chain, Answer (A) is correet: A direct cost can be traced directly to a cost object. Salary does not vary over a set period of time and is therefore a fixed direct labor cost. The other items are either variable (Water and Toilet tissue) or indirect (Advertising for the Sleep-Well Inn chain) (Q23: The Profit and Loss Statement of Madengrad Mining Inc. includes the following information for the current fiscal year: Sales $160,000, Gross profit 48,000, Year-end finished goods inventory 58,300, Opening finished goods inventory 60,190. The cost of goods manufactured by Madengrad for the current fiscal year is A, $46,110 B. $49,890 C. $110,110 D. $113,890 = $160,000 - 48,000 = $112,000 ‘Beg F.G inv + cost of goods manufactured = COGS + End F.G inv 60,190 + cost of goods manufactured = $112,000 + 58,300 Cost of goods manufactured = $110,110 Answer (C) is correct: COGS. Q24: The schedule of COGM of Gruber Fittings, Inc. shows the following balances for its fiscal year- end. Direct manufacturing labor $280,000 Manufacturing overhead 375,000 Ending work-in-process inventory 230,000 Raw materials used in production 450,000 Cost of goods manufactured 1,125,000 the value of the BWIP was A. $625,000 B. $250,000 €. $210,000 D. $20,000 Answer (B) is correct: BWIP + D.M used + D.L + O.H applied = Cost of goods manufactured +EWIP BWIP + 450 + 280 + 375 = 1,125 + 230 BWIP =1,125 +230- 450- 280- 375 BWIP =250 Q25: Mello Joy produces 200,000 units of a good that has the following costs. Direct material costs $2,000,000 direct manufacturing labor costs 1,000,000 indirect manufacturing labor costs 600,000 Mello Joy’s per unit prime costs and conversion costs, respectively, are AB and 15 B.8 and 18 C.10 and 8 D.1S and 8. Answer (D) is correct: Essay: A portion of the costs incurred by business organizations is designated as direet labor cost. As used in practice, the term ‘direct labor cost” has a wide variety of meanings. Unless the meaning intended in a given context is clear, misunderstanding and confusion are likely to ensue. Ifa user does not understand the elements included in direct labor cost, erroneous interpretations of the numbers may occur and could result in poor management decisions. The National Association of Accountants has issued Statement on Management Accounting (SMA) Number 4C, ‘Definition and Measurement of Direct Labor Cost,’ to assist ‘management accountants in dealing with problems that may arise in interpreting and understanding direct labor costs. Along with providing a conceptual definition of direct labor cost, this Statement describes how direct labor costs should be measured, Measurement of direct labor costs involves two aspects: (1) the quantity of labor effort that is to be included, that is, the types of hours or other units of time that are to be counted; and (2) the unit price by which each of these quantities is multiplied to arrive at a monetary cost. Required: A. Distinguish between direct labor and indirect labor. B. Explain why some non-productive labor (e.g., coffee breaks, personal time) is treated as direct labors while other non-productive labor (¢.g,, downtime, training) is treated as indirect labor. Answer A. Direet labor can be directly traced to a cost object such as a product or service this is accomplished through a source document, such as a time ticket, or through automated tracking systems such as touch screens or bar code scanners at a workstation. An example of direct labor is the hours spent by an artist in creating a painting. Indirect labor can't be easily traced to a particular product or service (or management chooses not to trace this directly because of the cost involved in attempting direct tracing, and/or the insignificance of the labor cost involved). Often, indirect labor is a common cost applicable to many products. For example, the time involved by an artist in obtaining supplies is an indirect labor cost because the supplies are used on more than one painting. B. Non-productive labor included in direct labor: Some types of non-productive labor are normal, unavoidable, and/or required by law. Examples include break time and bathroom time. Thus the amount of time that is budgeted (and must be paid for) as direct labor includes both productive time directly working on a product or service plus an allocation of this non- productive time Non-productive labor classified as indirect fabor: Other types of labor time, such as downtime and training time are classified as indirect labor costs because these activities are (potentially) avoidable and not attributable to an individual product. Inventory systems Assume ABC inventory balance $2,500, cost of purchases $5,000 and COGS $6,200 Deseripti Perpetual inventory system rentory system ‘At Purchase "DR Inventory $5,000 (charge) —SSS*|SSSCIR Purchases $5,000 CR AIP or cash $5,000 CR AIP or cash $5,000 When units DR COGS $6,200 No entries sold CR Inventory $6,200 (discharge) Ending inventory ABC $2,500 » Purchases $5,000 ~ COGS $6,200 = XYZ inventory $1,300 DBR Ending inve Egyptian American center 7 Mohammed el Mahdy st.-el doky- Egyptian American center @ face book. Com 0237499046 -01227246481 [ Year end entries ‘No entries. Uniess there is inventory over and shortage Perpetual inventory system results in Better control because this method provides a continuous record keeping of the quantities of inventory and inventory costs, DR COGS (plug) 6.200 CR Beginning inventory 2,500 CR Purchases 5,000 | Periodic inventory system fs Simple and needs lower costs to maintai accordingly it is used with immaterial Advantages | accordingly it is used in manufacturing companies and items that do not carry high inherent risk raw materials or merchandise units. (EX, inventory of coal) ee items are added to or taken from inventory, Accordingly itis used when cost /benefit criterion is justified. determine inventory quantities; accordingly misappropriations can’t be detected and accourted for. Under a perpetual inventory system the purchase of raw materials (both direct material and indirect material) is debited to materials inventory ‘ontrol or Direct materials costs vDire object in an economically fea: Compauents of D.M costs ‘Direct material costs include: vendor’s invoice costs plus sales taxes, custom's duties, cost ‘stores control”. material costs are costs of materials that can be specifically identified (assigned) with the cost of delivery (transportation in) and net of (deducted) trade discounts, rebates and refunds. But Purchasing costs (like preparat n of pure! are not a direct material cost, per se. rather it is a period costs. Cash discounts on direct materials (cost accounting view) se order), inspection costs, and storage costs ¥if cash discount is taken; it is viewed as interest saving, not as a reduction in the cost of materials. Thus Cash discounts should not be deducted from the direct material’s costs unless the rate of cash discount exceeds reasonable interest rates, then in substance cash discounts are considered as trade discounts or rebates under these conditions cash discounts should be deducted from the cost of direct materials, Cash discounts (financi: Egyptian American center 7 Mohammed el Mahdy st.—el doky- Giza—T -0237499046 -01227246481 Egyptian American center @ face book. Com ‘Assume the company purchased merchandise costing $1,000 on 5/10-30 and the company uses periodic inventory method. - Description Gross method Net method ‘At purchase date DR Purchases 51,000 DR Purchases $950 CRAP $1,000 CRAP $950 — DR A/P $1,000 DR AP ‘$950 ‘Assuming payment in tenth day CR Cash $950 CR Cash $950 CR Cash discount $50 DRAP $1,000 DRAP $950 ‘Assuming payment in due date CR Cash $1,000 DR Cash discount forfeited $50 €R Cash $1,000 See Gleim MCQ" # 296,298,299,300,301,305,307,308,309,310,312,313,314,317,321,322,327,356,357,358,360,361,36 2,363,364,369,370,371,372,373,374,403,404,414,504,505,507,508,509,532,533,542,543,546, 547, Remember ‘vPrime cost elements (DM and DL) can be physically traced to the cost object (units of output) Egyptian Ameri ‘an center 7 Mohammed el Mahdy st.-el doky- Gi T -0237499046 01227246481 Egyptian American center @ face book. Com ‘osting Approaches 1, Under full costing (absorption costing) WIP is charged with DM + DL + Total factory O.H (fixed and variable), accordingly all these elements are product costs (inventoriable) thus under absorption costing, fixed manufacturing overhead costs are best described as Indirect product costs because they can't be directly traced to specific units produced. Absorption costing is required under GAAP. 2. Under variable costing (direct costing) WIP is charged with DM + DL + Variable factory O.H, accordingly fixed factory O.H is a period cost. Variable costing (direct costing) is preferred for internal reporting purposes. QI: Assuming absorption costing approach is applied which of the following columns includes only product costs? Cost item L Ti. | WH. | TH. Direct labor x x |X Direct materials x |x x Sales materials x ‘Advertising costs x Indirect Factory materials x |x |x Indirect labor x|x|x Sales commissions x Factory uiilities x x |X ‘Administrative supplies expense x ‘Administrative labor x Depreciation on administration building x Cost of research on customer demographics x Correet Answer is the last column Product costs are the costs of producing the product and are usually directly identifiable Product costs include direct materials, direct labor, and factory (not general) overheads. Factory overhead includes both fixed and variable elements under absorption-costing system, e. factory utilities, indirect labor, and indirect materials. Product costs are inventoried until the product is sold, at which time they are expensed as cost of goods sold. Q2: Inventoriable costs A. Include only the prime costs of manufacturing a product. B. Include only the conversion costs of manufacturing a product, C. Are expensed when products become part of finished goods inventory. D. Are regarded as assets before the products are sold. 23 Answer (A) is incorrect because overhead costs as well as prime costs (direct materials and Jabor) are included in inventory. Answer (B) is incorrect because materials costs are also included. Answer (C) is incorrect because inventory costs are expensed when the goods are sold, not when they are transferred to finished goods. Answer (D) is correct. Under an absorption costing system, inventoriable (product) costs include all costs necessary for good production. These include direct materials and conversion costs (direct labor and overhead). Both fixed and variable overhead is included in inventory under an absorption costing system. Inventoriable costs are treated as assets until the products are sold because they represent future economic benefits. These costs are expensed at the time of sale Q3: Absorption costing and variable costing are two different approaches of assigning costs to units produced. Of the following cost items listed, identify the one that is not correctly accounted for as a praduct cost. Choice Cost Item Part of Product Cost under “Absorption costing] Variable costing A. t Manufacturing supplies Yes ‘Yes’ | B Insurance on factory Yes No c rect labor Yes Yes D Packaging and shipping costs Yes Yes J ible costs inventoried under ‘Answer (A) is incorrect because manufacturing supplies are vai both methods. Answer (B) is incorrect because factory insurance is a fixed manufacturing cost inventoried under absorption costing but written off as a period cost under variable costing. Answer (C) is incorrect because direct labor cost is a product cost under both methods. Answer (D) is correet. Under absorption costing, all manufacturing costs, both fixed and variable are treated as product costs. Under variable costing, only variable costs of manufacturing are inventoried as product costs. Fixed manufacturing costs are expensed as period costs. Packaging and shipping costs not product costs under either method because they are incurred afier the goods have been manufactured. Instead, they are included in selling and administrative expenses for the period. Q4: In the application of direct costing as a cost-allocation process in manufacturing, ‘A. Variable direct costs are treated as period costs. B. Non-variable indirect costs are treated as product costs. C. Variable indirect costs are treated as product costs D. Non-variable indirect costs are not treated as period costs. Answer (A) is incorrect because variable manufacturing costs, whether direct (direct materials and direct labor) or indirect (variable factory overhead), are accounted for as product costs, not period costs. Answer (B) is incorrect because non-variable indirect costs are treated as period costs in di costing, Answer (C) is correct. Direct (variable) costing considers only variable manufacturing costs to be product costs. Variable indirect costs included in variable factory overhead are therefore treated as inventoriable. Fixed costs are considered period costs and are expensed as incurred. Answer (D) is incorrect because, in direct costing, non-variable indirect costs are treated as period costs, not product costs. QS: Ifa manufacturing company uses variable costing to cost inventories, which of the following costs are considered inventoriable casts? A. Only raw material, direct labor, and variable mamufacturing overhead costs. B. Only raw material, direct labor, variable and fixed manufacturing overhead costs. C. Only raw material, direct labor, variable manufacturing overhead and variable selling and administrative cos D. Only raw material and direct labor costs. Correct answer is (A) Q labor, and (8) finished goods shipping costs under absorption costing and variable costing? Choice Absorption Costing Variable Costing Product Cost | Period Cost | Product Cost | Period Cos Which of the following correctly shows the treatment of (1) factory insurance, (2) dire A | 1,2 3 2 nS B 1 2 1,3 12 3 c 1,2 3 1 alps) D 1 2,3 2,3 1 Correct answer is (A) Q7: Bethany Company has just completed the first month of producing a new product but has not yet shipped any of this products, the product incurred variable manufacturing costs of $5,000,000, fixed ig costs of $2,000,000, variable marketing costs of $1,000,000, and fixed marketing manufacti costs of $3,000,000. If Bethany uses the variable cost method to value inventory, the inventory value of the new product would be ‘A. $5,000,000 B. $6,000,000 C. $8,000,000 D. $11,000,000 Correct answer is (A) Q8: A review of Plunkett Corporation’s accounting records for last year disclosed the following selected information. Variable costs direct materials used $36,000 direct labor 179,100 manufacturing overhead 154,000 selling costs 108,400 fixed costs Manufacturing overhead 267,000 ‘Selling costs 121,000 Administrative costs 235,900 in addition, the company suffered a $25,700 uninsured factory fire loss during the year and $2,000 abnormal spoilage. What were Plunkett’s product costs and period costs for last year? A $235,100 $974,000 B T ‘$497,500 ‘$651,600 | Cc ] ‘$656,100 $493,000 D | ‘$683,800 $465,300 ] Answer (C) is correct Q9: Chasse Company, a cracker and cookie manufacturer, has the following unit costs for the month of June. Variable ] Variable ] Fixed Fixed ] manufacturing cost | Marketing cost | manufacturing cost | marketing cost 35 T $3.50 32 u A total of 100,000 units were manufactured during June, of which 10,000 remain in ending inventory. Chasse uses the first-in, first-out (FIFO) inventory method, and the 10,000 units are the only finished goods inventory at month-end. Using the fall absorption costing method, Chasse’s finished goods inventory value would be ‘A. $50,000 B. $70,000 C. $85,000 D. $145,000 Answer (B) is correct QUO: Ina broad sense, cost accounting can best be defined within the accounting system as A. Internal and external reporting that may be used in making non-routine decisions and in developing plans and policies. B. External reporting to government, various outside parties, and shareholders, C. Internal reporting for use in management planning and control, and external reporting to the extent its product-costing function satisfies external reporting requirements. D. Internal reporting for use in planning and controlling foutine operations. Answer (A) is incorrect because Cost accounting is concerned with more than just reporting to be used in making Non-routine decisions. 26 ‘Answer (B) is incorrect because Cost accounting also provides information for internal reporting. Answer (C) is correct. Cost accounting is a combination of (1) management accounting in the sense that its purpose can be to provide intemal reports for use in planning and control and in making non-routine decisions, and (2) financial accounting because its product-costing function satisfies extemal reporting requirements for reporting to shareholders, government, and various outside parties. Answer (D) is incorrect because Management accounting entails internal reporting for use in planning and controlling routine operations. See Gleim MCQ" # 297,304,306, 315,384,396,402,482,487,489,490,501,502, Notes 1. Fixed versus variable cost classification is based on the activity level of cost driver, cost that Changes with activity level of cost driver are variable costs. Thus there is a cause-and-effect Relationship between the level of activity (of the cost driver) and incurrence level of Variable costs, while costs that are non-responsive to change when activity level Changes (Within relevant range) are fixed costs. ixed Cost is the cost that continues to be incurred in the absence of activity (ie. no out put produced). EX. Factory rent, property tax, and depreciation. Fixed cost does not have cost drivers over the short run, but over the long run all Costs have cost drivers. 3. Engineered cost is a cost that bears an observable and known relationship to a quantifiable activity base. Engineered costs have a clear relationship to output. Direct materials would be an example of an engineered cost. Thus engineered cost is a variable cost 4, Direet versus indirect cost classification is based on cost object, costs that can directly and specifically identified (traced) in an economic manner to the cost object are direet costs (example DM when the cost object is the unit produced, equipment depreciation expense when the cost object is the equipment itself) while indirect costs are those costs that are not linked directly with the cost object (supervisory salary when the cost object is the unit produced, factory depreciation expense When the cost object is a line of production). Indirect cost is allocated to the cost objects. Thus the determination ofa cost as being either direct or indirect depends upon the cost object chosen. Costs that are direct to some specific cost object could be indirect to other cost object 5. Cost object could be a single product, line of products, service, department, single machine, factory, one manufacturing process, a job, a contract or a customer. 6. Direct costs could be variable or fixed. 7. Indirect costs could be variable or fixed. 8. Prime costs also called direct manufacturing costs = D.M + D.L 9. Conversion costs (C.C) = D.L + (variable and fixed Factory. O.H) 27 10. Costs are accounted for as follow: A) Cost accumulation (in cost pools accounts’) B) Cost assignment (includes cost tracing for direct costs and cost allocation for indirect costs) vTracing direct costs is quite straightforward, whereas Assigning Indirect costs to a number of different cost objects can be very challenging, ‘vThe materiality of the cost is a factor in classi ng the cost as a direct or indirect. ‘¥Cost tracing is the assignment of direct costs to the chosen cost object Cost allocation is the assignment of indirect costs to the chosen cost object. QI: Given that a cost has been already identified as a variable cost, which of the following additional descriptions of that cost is incompatible with that identification? A. The cost, in total, does not change with changes in the volume of the cost driver. B. The cost can be traced directly to the cost object. C. The vost, in total, does change with changes in the volume of the cost driver. D. The cost can't be traced directly to the cost object. E. The cost is not a prime cost. Answer (A) is correct Q2: Roberta is the manager of Sleep-Well Inn, one of a chain of motels located throughout the U.S. An example of an operating cost at Sleep-Well that is both direct and fixed is A, Johnson’s salary. B. Water. C. Toilet tissue. D. Advertising for the Sleep-Well Inn chain. Answer (A) is correct: A direct cost can be traved directly to a cost abject. Salary does not vary over aset period of time and is therefore a fixed direct labor cost. The other items are either variable (Water and Toilet tissue) or indirect (Advertising for the Sleep-Well Inn chain) Q3: Roberta Johnson is the manager of Sleep Well Inn, one of a chain of motels located throughout the United States. An example of an operating cost at Sleep Well that is semi variable is, A, Postage for reservation confirmations _B. The security guard’s salary C. Electricity D. Local yellow pages advertising Answer (C) is correct:. Semi variable costs are costs that ate composed of both fixed and variable components. Electricity is considered a semi variable cost as there is a fixed base charge, and as electricity usage increases, the cost of electricity increases. Q4: Consider a single hard copy of a book asa cost object. What would be the best labels to classify the relation between this cost object and the following two costs of producing the cast object respectively: (1) the paper and (2) the one-time fees paid to the authors (i.¢., not royalties)? A. (1) Direct cost, (2) Variable cost. B. (1) Variable cost, (2) Unavoidable cost. 28 C. (1) Fixed cost, (2) Variable cost. D. (1) Direct cost, (2) Fixed cost. E. (1) Avoidable cost, (2) Variable cost. Answer (D) is correct Q5: Cost traceability is best described as A. Assigning a cost to a cost object through a cause-and-effect relationship. B. Distinguishing between marginal costs and revenues. C. Evaluating whether or not to include opportunity costs in a decision analysis. ig quantitative rather than qualitative factors because of their financial The correct answer is (A). By definition, cost traceability refers to the ability to assign a cost to a cost object in an economically feasible way by means of a cause-and-effect relationship. Direct costs are easily traced; indirect costs are not easily and accurately traced, Firms need to trace costs directly to cost objects and/or business segments during decision making when itis feasible to do 50, Exam QI: Direct costs A. Are always indirect costs. B. Increase in total when the actual level of activity increases. C. Include most personnel costs and depreciation on machinery. D. Can always be traced directly to the cost object. Q2: Cost objects include A.Products B. Customers C. Departments. alll of the above Q3: The determination of a cost as being either direct or indirect depends upon A. The accounting system — B, The allocation system. C. The cost tracing system D. The cost object chosen Q4: Classifying a cost as either direct or indirect depends upon A. The behaviour of the cost in response to volume changes B. Whether the cost is expensed in the period in which it is incurred C. Whether the cost can be easily identified with the cost abject. D. Whether an expenditure is avoidable or not in the future. QS: A manufacturing plant produces two product lines: football equipment and Hockey Equipment, Direct costs for the football equipment line are the ‘A, Beverages provided daily in the plant break room B, Monthly lease payments for a specialized piece of equipment needed to manufacture the football helmet C. Salaries of the clerical staff that work in the company administrative offices. D. Utilities paid for the manufacturing plant Q6: A manufacturing plant produces two product lines: football equipment and hockey equipment. An indirect cost for the hockey equipment line is the A. Material used to make the hockey sticks. 29 B, Labor to bind the shaft to the blade of the hockey stick. C. Shift supervisor for the hockey line. D. Plant supervisor. Q7: Which one of the following items is a direct cost? A, Customer-service costs of a multiproduet firm; Product A is the cost object. B. Printing costs incurred for payroll check processing; payroll check processing is the cost object C. The salary of a maintenance supervisor in a multiproduct manufacturing plant; Product B is the cost object. D. Utility costs of the administrative offices; the accounting department is the cost object. Q8: Indirect manufacturing costs ‘A. Can be traced to the product that created the costs. B. Can be easily identified with the cost object. C. Generally include the cost of material and the cost of labor. D. May include both variable and fixed costs, Q9: All of the following are true EXCEPT that indirect costs A. May be included in prime costs. B. Are not easily traced to products or services. C. Vary with the selection of the cost object. D. May be included in manufacturing overhead Q10: For a manufacturing-sector company, the cost of factory insurance is classified as A. Direct material cos B. Direct manufacturing labor cost. C. Manufacturing overhead cost. _D. Prime cost. QUI: Fora printing company, the cost of paper is classified as a A. Direct material cost. B. Direct manufacturing labor cost. C. Manufacturing overhead cost. D. Period cost. Q12: Wages paid to machine operators on an assembly line are classified as a A. Direct material cost. B. Direct manufacturing labor cost and part of C.C, also it is considered as a part of prime cost. C. Manufacturing overhead cost. D. Period cost. Q13: Manufacturing overhead costs in an automobile manufac include: ‘A. Labor costs of the painting department B. Indirect material costs such as lubricants (machine oil). C. Sales commissions, D, Steering wheel costs. ing plant MOST likely Q14: Manufacturing overhead costs are also referred to as ‘A. Indirect manufacturing costs _B. Prime costs. C. Period costs. D. Conversion costs. QUS: Prime costs inchide A. Direct materials and direct manufacturing labor costs. B. Direct manufacturing labor and manufacturing overhead costs. C. Direct materials and manufacturing overhead costs. D. Only direct materials. Q16: Conversion costs include A. Direct materials and direct manufacturing labor costs. B. Direct manufacturing labor and manufacturing overhead costs. C. Direct materials and manufacturing overhead costs. D. Only direct materials. QI7: The cost classification system used by manufacturing firms include all of the following EXCEPT A. Direct materials costs and conversion costs B. Direct materials costs, direct manufacturing labor costs, and manufacturing overhead costs. C. Indirect materials costs, indirect manufacturing labor costs, and manufacturing overhead costs. D. Prime costs and manufacturing overhead costs. QI8: The general term used to identify both the tracing and the allocation of accumulated costs to acost object, A.Costaccumulation B. Costassignment —_C. Cost tracing D. Conversion costing Q19: The collection of accounting data in some organized way is A. Cost accumulation B. Cost assignment — C. Cost tracing D. Conversion costing Q20: Cost tracing is A. The assignment of direct costs to the chosen cost object B. A function of cost allocation. C. The process of tracking both direct and indirect costs associated with a cost object. D. The process of determining the actual cost of the cost object. Q21: Cost allocation is A. The process of tracking both direct and indirect costs associated with a cost object. B. The process of determining the actual cost of the cost object. C. The assignment of indirect costs to the chosen cost object. D. A function of cost tracing. Q22: Assigning direct costs to a cost object is called A. Cost allocation B. Cost assignment —C. Cost pooling —_D. Cost tracing Q23: is the process of distributing indirect costs to products A.Costallocation B. Job cost recording C. Cost pooling _D. Cost tracing Q24: Which of the following includes both traced direct costs and allocated indirect costs? A.Cost tracing B, Cost pools C. Cost assignments _D. Cost allocations 31 True or false 1. Products, services, departments, and customers may be cost objects. 2. The same cost may be direct for one cost object and indirect for another cost object. 3. The cost ofa customized machine only used in the production of a single product would be classified as a direct cost if the cost object is the machine itself 4. The materiality of the cost is a factor in classifying the cost as a direct or indirect cost. 5. Direct costs are allocated to the cost object using a cost-allocation method, 6. Cost objects may be jobs, products, or customers. 7. Conversion costs include all direct manufacturing costs. 8, Part of prime cost is indirect manufacturing costs. 9, Costs are accounted for in two basic stages: Assignment followed by Accumulation 10. Assigning direct costs poses more problems than assigning indirect costs. 11. Direct costs are allocated to the cost object using a cost-allocation method. 12. Direct costs plus indirect costs equal total costs. 13, Direct tracing of costs improves cost accuracy. Exam answers QI: (PD) Q2:(D) _Q3: (D)_ Qa: (C) (Note Answer (A) concems fixed /variable cost classification, Answer (B) concems period/product cost classification, Answer (D) concerns avoidable /non-avoidable cost classification) Q5: (B) Q6:(D) Q7:(B) Q8:(D) Q9: (A) QNO:(C) QUE) Q12:(B) QI3: (B) Note: Answer (A) is D.L Answer(C) is period cost Answer (D) is D.M Q14: (A) QU5: (A) QI6:(B) QUT: (C) All of the items included represent only one category of manufacturing costs (factory O.H) costs whereas D.M and D.L are not mentioned. Q18:(B) QUI: (A) Q2OE(A)_ Q21:(C)_Q22:(D)_Q23: (A) Q24: (C) ‘True or false answers 1. True. 2. Tru 3. True. 4, True, 5, False because indirect costs are allocated to the cost object using a cost-allocation method. 6. True. 7. False because Prime costs include all direct manufacturing costs. 8. False because Prime costs include all direct manufacturing costs. 9. False because Costs are accounted for in two basic stages: Accumulation followed by assignment. 10. False 11, False, because indirect costs are allocated to the cost object using a cost allocation method 12.True 13. True ‘ingle (plant ic) predetermined factory O.H allocation rate Single (plant-wide) predetermined factory O.H allocation rate (R) = Budgeted..Annual..Total...factory.O.H Say.$6,000.Numerator) Estimated. Annual .activity.level.of .cost.driver Say.3D.L.H * 1,000.unit.of output.(Deno min ator) ¥ Predetermined factory O.H rate = $2 of factory O.H per D.L.H (Preferable) ¥ Predetermined factory O.H rate = $6 of factory O.H per unit of output (Not Preferable) Notes 1) Under single plant- wide O.H allocation rate total manufacturing O.H are pooled to only one single pool and are related to single cost driver, this makes single plant- wide O.H rate is not accurate or logical especially if the company have many factory O.H costs that related to many cost drivers thus single plant wide rate is inappropriate in most cases. 2) The order of steps to determine the O.H application rate A. The annual output of finished produet is to be forecasted (say 1,000 Units) B. Determine the most appropriate cost driver that causes the incurrence of O.H costs (Say D.L.H) C. Determine the estimated activity level of the chosen cost driver that necessary to realize forecasted annual output (1,000 unit of output x 3 DLH per unit of output) D. Estimate total annual factory O.H (Numerator) through cause and effect relationship between the cost driver and its estimated activity level and the incurrence of factory O.H cost elements, Management judgment with respect to the nature and amount of costs to be incurred and production volume are the most important factors in budgeting manufacturing OH E. Calculate Predetermined annual O.H allocation rate ($2 per D.L.H) 3) Predetermined O.H rate is sometimes called standard O.H rate. +r (3,000 DLH / 1,000 units of output) is called the 4) Estimated activity level of cost OH allocation base (the base or Denominator) it could be any activity driver (D.L.H, M.H, D.M pounds or cost, D.L Costs or even the units of the output) as long as it has an effect on the occurrence of the most elements of factory O.H. 5) The O.H. application rate is established at the beginning of each year. So it is called predetermined OH allocation rate Q: Why O.H. application rate is predetermined based on annual estimates? S. Because annual rates smooth seasonal variability of factory O.H. costs (numerator) and activity levels (denominator). If factory O.H costs were charged to the product as incurred 33 (Actual costing systema) The O.H. allocated to WIP would vary considerably from week to week or period to period. Thus The purpose of an annual factory O.H. application rate is to simulate constant factory O.H application throughout the year. On the other side charging WIP for actual O.H as incurred Simulates seasonal variability of activity levels and actual incurrence of O.H items. 6) Application of factory O.H. to WIP During the year means that factory O.H. is a product cost (inventoriable). 7) Allocating overhead on the basis of the number of units produced is usually not appropriate because O.H Costs should be allocated on the basis of some plausible relationship between the cost object and the incurrence of the cost, preferably cause and effect. Overhead costs may be incurred regardless of the level of production volume, When multiple products are involved, the number of units of production may bear no relationship to ‘the incurrence of the allocated cost. If O.H1 is correlated with machine hours but different products require different quentities of that input, the result may be an illogical allocation, However, if a firm manufactures only one product, the allocation of factory O.H based on the number of units produced may be acceptable because all costs are to be charged to the single product. 8) Cost drivers are activities that cause costs to increase as the activity increases such as DLH, M.H, beds occupied, computer time used, flight hours, miles driven, or contracts, Cost rivers are causal factors to the incurrence of cost in an entity” .It is a basis used to assign costs to cost objects. Cost drivers are not accounting measurements but measures of activities that cause Costs to incur, Cost driver can take the form of mechanical basis, such as M.H, computer time, size of equipment, or square footage of factory, used to assign costs to activities and cost driver could take the form of financial basis for example a cost driver for ‘pension benefits is employee salaries QU: Ina labor intensive industry in which more overhead (service, support, more expensive equipment, etc.) is incurred by the more highly skilled and paid employees, which activity base is most likely to be appropriate for applying overhead? A. DLH. B. DM cost. Cc. MH. D. DL cost. Answer (A) is incorrect because direct labor hours are appropriate when overhead is incurred uniformly by all types of employees. Answer (B) is incorrect because direct materials cost would be inappropriate for a labor intensive industry. ity base when Answer (C) is incorrect because machine hours are an appropriate acti overhead varies with machine time used. Answer (D) is correet. In labor intensive industries, overhead is usually allocated based on a labor activity base. If more O.H is incurred by the more highly skilled and paid employees, the OM rate should be based upon DL cost rather than DLH. Q2: The two most appropriate factors for budgeting manufacturing O.H expenses would be: A. Machine hours and produetion volume, B, Management judgment and C.M. C. Management judgment and production volume. D. Management judgment and sales dollars. ‘Answer (A) is incorrect because machine hours may not be the appropriate activity base. Moreover, some overhead is fixed regardless of the activity base. Answer (B) is incorrect because the C.M can be calculated only after variable costs and sales prices are determined. Some overhead is variable. Answer (C) is correct. The most important factor in budgeting manufacturing O.H is production volume. Many overhead items have variable costs, and those that are fixed with a relevant range of output may increase if production exceeds that range. The other essential consideration is management’s judgment with respect to the nature and amount of costs to be incurred in the light of expected production volume. Because O.H is applied based on predetermined rates, accurate judgment is important. Answer (D) is incorrect because sales volume (or dollars) is less significant because overhead is based on production volume. Q3: Baldwin Printing Company uses a job order costing system and applies overhead based on machine hours. A total of 150,000 machine hours have been budgeted for the year. During the year, an order for 1,000 units was completed and incurred the following. Direct material costs $1,000, direct labor costs 1,500, Actual overhead 1,980, Machine hours 450. The accountant calculated the inventory cost of this order to be $4.30 per unit. The annual budgeted overhead in dollars was 35 A, $577,500. B. $600,000 C. $645,000. $660,000 Answer (B) is correct: Total manufacturing cost per unit =1,000 unit's x $4.3 = $4,300 ‘Factory O.H applied = $4,300 - ($1,000 DM +$1,500 D.L) = $1,800 $1,800 a8 54 per MA Boman POM ‘Annual budgeted overhead =150,000 budgeted machine hours « $4 per M.H = $600,000 +Underapplied factory O.H for this job = $1,980 - $1,800 = $180 Q4: Patterson Corporation expects 10 incur $70,000 of factory O.H and $60,000 of general ‘Factory O.H application rate = and administrative expenses next year. DL costs at $5 per hour are expected to total $50,000. If factory O.H is to be applied per DLH, how much O.H will be applied to a job incurring 20 DLH? A. $28 B. $120 C. $140, D. $260 Answer (C) is correct: Budgeted D.L.H = $2.20 19,090 DL 35.per.DLH Predetermined factory O.H allocation rate based on DLH= onpei = $7 per DLH ‘Factory O.H allocated to the job = $7 per DLH * 20 DLH = $140 QS: felicity Corporation manufactures a specialty line dresses using a job-order cost system, During January, the following costs were incurred in completing job J-1 Direct materials $27,400, direct labor (400 DLH) $9,600, Administrative costs 2,800, Selling expenses 11,200.Factory overhead was applied at the rate of $50 per direct labor hour, if {job J-1 resulted in 4,000 good dresses, what is the cost of goods sold per dress A. $14.25 B. $14.95 C. $17.25 D. $59.25 Answer (A) is correct. ‘Total manufacturing costs (j-1) = D.M + D.L + factory O.H applied Total manufacturing costs (j-1) = $27,400 + $9,600 + ($50 * 400 DLH) = $57,000 COGS per unit = $57,000 ~ 4,000 units = $14.25 Q6: Units of production is an appropriate overhead allocation base when: A. Several well-differentiated products are manufactured. B. Direct labor costs are low. C. Direct material costs are large relative to direct labor costs incurred. D. Only one product is manufactured. ‘Answer (A) is incorrect because the number of units of production may have no logical relationship to overhead when several different products are made. Answer (B) is incorrect because a low level of direct labor costs means that fixed overhead is substantial, and an appropriate cost driver should be used to make the allocation Answer (C) is incorrect because the allocation should be made on the basis of the appropriate cost drivers without regard to the relationship between direct materials and labor costs. Answer (D) is correct. Allocating overhead on the basis of the number of units produced is usually not appropriate, Costs should be allocated on the basis of some plausible relationship between the cost object and the incurrence of the cost, preferably cause and effect. Overhead costs may be incurred regardless of the level of production volume. When multiple products are involved, the number of units of production may bear no relationship to the incurrence of the allocated cost. If overhead is correlated with machine hours but different products require different quantities of that input, the result may be an illogical allocation However, if'a firm manufactures only one product, this allocation method may be acceptable because all costs are to be charged to the single product. Using Multiple O.H rates YMultiple O.H rates are more appropriate than single Plant wide O.H rate especially if the company produces multiple products and each product consumes different quantities of each cost driver. Q: A company allocates factory overhead to jobs in process using raw material costs, direct labor costs, and machine hours. The overhead application rates for the current year are 20% of direct materials cost, 100% of direct labor cost and $117 per machine hour, a particular production run incurred the following costs: Direct materials, $2,000, direct labor $8,000 and the total a total of 140 machine hours were required for the production run, What manufacturing cost of the production run? Answer Total manufacturing costs of the production run = D.M + D.L + factory OH applied Factory O.H applied = ($2,000 * 20%) + ($8,000) + (140 M.H * $117) = $24,780 T.M.C of the production run = $2,000 + $8,000 + $24,780 = $34,780 Essay: Amur Company manufactures three lawn care component parts: fuel systems, transmission assemblies, and electrical systems. For the past five years, manufacturing ‘overhead has been applied to products on standard direct labor hours for the units actually produced. The standard cost information is shown below. | Fuel” | Transmission | Electrical Description Systems | Assemblies | Systems Units produced and sold 19,000 | 20,000 30,000 Standard labor hours 2 15 1 Standard direct material cost per unit 325 $36 $30 Budgeted and actual manufacturing overhead $53,920,000 ‘The current direct labor rate is $10 per hour. New machinery that highly automates the production process was installed two years ago and greatly reduced the direct labor time to produce the three products. The selling price for each of the three products is 125% of the manufacturing cost. ‘Amur’s segment of the lawn care component industry has become very competitive, and the company’s profits have been decreasing. Eric West, Amur’s controller, has been asked by the president of the company to analyze the overhead allocations and pricing structure. West thinks that future allocations should be based on machine hours and direet labor hours rather than the current allocation method, which is based on direct labor hours only. West has determined the additional product information as follows Fuel Transmission Electrical Description Systems | Assemblies Systems Standard machine hours 2 4 6 f Manufacturing overhead: ' Direct labor hours $560,000 Machine hours $3,360,000 Questions: 1. by allocating all of the budgeted overhead based on direct labor hours, calculate the unit ‘manufacturing cost and unit sales price for each of the three products manufactured at Amur Company. 2. Prepare an analysis for Amur Company using the appropriate cost driver (s) determined by Eric west for manufacturing overhead, Calculate the unit manufacturing cost and unit sales price for each of the three products. 3. Based on your calculations in questions I and 2, prepare a recommendation for the president at Amur Company to increase the firm's profitability Answer: 1. the allocation of all of Amur Company's budgeted manufacturing overhead based on direct 38 labor hour's results in the unit manufacturing costs and unit sales prices for its three products is calculated as follows _ Fuel | Transmission | Electrical aa Description Systems| Assemblies | Systems Total budgeted OH $3,920,000 Units produced and sold | 10,000 20,000 30,000 Standard labor hours 2 15 1 Total standard labor hours | 20,000 30,000 30,000 80,000 O.H allocation rate 349 per DL ‘Manufacturing cost calculation DM $25 336 $30 DL 20 15 10 O.H applied 98 BS 49 Manufacturing cost per unit | 143 124.50 89 Price (125%) 178.75 155.625 111.25 2. An analysis for Amur Company using the appropriate cost driver(s) determined by Eric West for manufacturing overhead. The unit manufacturing cost and unit sales price for each of the three products are as follo Fuei_ | Transmission | Electrical an Description Systems | Assemblies Systems Total budgeted 0. based on on $560,000 I Units produced and sold 10,000 20,000 30,000 Standard labor hours 2 15 H Total standard labor hours | 20,000 30,000 30,000 80,000 (O.H allocation rate (I) ST per DL Total budgeted O.H based on aoe 39 DLA Units produced and sold 10,000 20,000 30,000 Standard machine hours 2 4 6 Total standard labor hours | 20,000 80,000 180,000 280,000 (O.H allocation rate 2) 322 per ML ‘Manufacturing cost calculation “DM $25 336 330 DL 20 is 10 ‘O.H applied based on DLA 4 105) 7 ‘O.H applied based on MH um B R Manufacturing cost per unit | 83 109.30 119 Price (125%) 103.75 136.875 148.75 3. Based on calculations in questions I and 2, a recommendation for the president at Amur Company to increase the firm’s profitability The summary of the revised margins for each of Amur Company’s three products, 2 suming the sales prices developed in question 1 (allecation of all manufacturing ovethead based on direct labor hours) is compared to revised costs developed in question 2 (allocation of manufacturing overhead based on cost drivers), is as follows: Fuel Transmission | Electrical Description Systems Assemblies Systems Current price 178.75 155.625 111.25 Revised cost 83 109.50 119 Gross profit (loss) $95.75 $46.13 (87.75) Gross Margin % on sales 54% 30% NA Based upon this analysis, fuel systems and transmission assemblies are producing a higher return than Amur Company previously thought. Fuel systems are the most profitable (54 % gross margin on sales rather than 25% on cost as previously thought) followed by transmission assemblies; however, electrical systems are losing money on a full-cost basis Recommend: reducing the price to increase sales. + Improve protitability of electrical systems through changes to the manufacturing process to ions for improving profitability include the following: + Focus on fuel systems, through actions such as incr sing marketing expenditures and 40 reduce the machine hours required. + Decrease marketing of this electrical system, and increase the selling pric if possible. Conclusion: the concept of Multiple O.H. Rates is more precise and relatively more complex than single plant-wide rate, it requires many separate factory O.H. cost pools to more accurately apply factory O.H. costs to cost objects, Accordingly many factory O.H. rates, many factory O.H. control accounts must be established. Under Multiple O.H. Rates concept many cost drivers are to be identified based on cause and effect relationships with factory O.H. cost incurrence. Q: Henry Manufacturing, which uses direet labor hours to apply overhead to its product line, undertook an extensive renovation and modernization program two years ago. Manufacturing processes were reengineered, considerable automated equipment was acquired, and 60% of the company’s non-union factory workers were terminated. Which of the following statements would apply to the situation at Henry? I. The company’s factory overhead rate has likely increased, IL. The use of direct labor hours seems to be appropriate. III, Henry will lack the ability to properly determine labor variances. IV. Henry has likely reduced its ability to quickly cut costs in order to respond to economic downturns. A.1, III, and IV B.LandIVonly C.Iand1V only D.1and Ill only Answer (B) is correet Essay: Moss Manufacturing has just completed a major change in its quality control (QC) process. Previously, products had been reviewed by QC inspectors at the end of each major process, and the company's ten QC inspectors were charged as direct labor to the operation or job. In an effort to improve efficiency and quality, a computerized video QC system was purchased for $250,000. The system consists of a minicomputer, 15 video cameras, other peripheral hardware, and software. The new system uses cameras stationed by QC engineers at key points in the production process. Each times an operation changes or there is a new operation, the cameras are moved, and a new master picture is loaded into the computer by a QC engineer. The camera takes pictures of the units in process, and the computer compares them to the picture of a “good” unit. Any differences are sent to a QC engineer who removes the bad units and discusses the flaws with the production supervisors. The new system has replaced the ten QC inspectors with two QC engineers. The operating costs of the new QC system, including the salaries of the QC engineers, have been included as factory overhead in calculating the company’s plant-wide factory overhead rate which is based on direct labor dollars. The company’s president is confused. His vice president of production has told him how efficient the new system is, yet there is a large increase in the factory overhead rate. The computation of the rate before and after automation is shown below. Description Before automation | After automation Budgeted overhead $1,900,000 $2,100,000 Budgeted direct labor 1,000,000 700,000 41 T ‘Budgeted overhead rate 190% 300% ‘| | “Three hundred percent,” lamented the president. “How can we compete with such a high factory overhead rate?” Required 1) Define “factory overhead,” and cite three examples of typical costs that would be included in factory overhead. 2) Explain why companies develop factory overhead rates. 3). Explain why the increase in the overhead rate should not have a negative financial impact on Moss Manufacturing. 4). Explain, in the greatest detail possible, how Moss Manufacturing could change its overhead accounting system to eliminate confusion over product costs. 5). Discuss how an activity-based costing system might benefit Moss Manufacturing, Solution 1) Factory overhead costs include all indirect manufacturing costs (all production costs except direct material and direct labor). These costs can't be practically or economically traced to end products and, therefore, must be allocated to production using an appropriate allocation method. Typical factory overhead costs include + Indirect labor ie., lif-truck driver’s wages, maintenance and inspection labor, engineering labor. and supervisors. + Other indircet factory costs, i.c., factory building maintenance, machine and tool maintenance, property taxes on production premises, manufacturing property insurance, + Depreciation on plant and equipment, rent expense, and utility expense. 2) Companies develop factory overhead rates to facilitate and smooth the costing of products as they are completed and shipped, rather than waiting until actual costs are accumulated for the period of production. 3), The overhead rate increase should not have a negative impact on Moss Manufacturing because the increase in indirect costs was offset by a decrease in direct labor. 4), Rather than using a universal plant-wide overhead rate, Moss Manufacturing could implement separate overhead pools and allocate the overheads to the activities using the appropriate pools. Examples are as follows. A. Using dual O.H application rates by pooling total factory O.H costs into two pools one for variable O.H and the other for fixed 0.H instead of single plant wide rate B, Using departmental O.H rate for each production and service department by pooling factory O.H costs at the level of each department, C. Using departmental O.H dual rates for each production and service department by pooling factory O.H costs at the level of each department into two cost pools one for variable and the other for fixed factory O.H D, Using Multiple 0.H rates by pooling cost according to its relationship to specific cost driver E. Using an activity-based costing system ABC. 5). An activity-based costing system might benefit Moss Manufacturing because it + Differentiates costs between value adding and non-value adding activities. * Costs products according to the activities involved in the production process. * Considers all organizational expenses as variable. Universal rate V.S Multiple rates based on nature of expense involved Essay: Best Test Laboratories began as a one-man operation 25 years ago to evaluate the reaction of materials to extreme increases in temperature. Much of the company's early growth was attributable to government contracts to test the properties of weapons, transportation equipment, and clothing for use in arid desert regions. Recent growth has come from diversification and expansion into commercial markets. Environmental testing at Best Test now includes Heat testing (HTT) Air turbulence testing (ATT) Stress testing (SST) Aretic condition testing (ACT) Aquatic testing (AQT) Currently, all of the budgeted operating costs are collected in a common overhead pool; all of the estimated testing hours are also collected in a common pool. One rate per test hour is used for all five types of testing. This hourly rate is marked up by 45 % in order to recover administrative expenses, taxes, and profit in the sales price. Rick Shaw, Best Tests controller, believes that there is enough variation in the test procedures and cost structure to establish separate costing and billing rates. He also believes that the inflexible rate structure currently being used is inadequate in today’s competitive environment. After analyzing the following data, he has recommended that new rates be put into effect at the beginning of Best Test’s fiscal year. ‘The budgeted total test lab costs for the coming year are as follows. Description Budgeted amount Test pool labor (10 employees) $420,000 Supervision 72,000 Equipment depreciation 178,460 Heat 170,000, Electricity 124,000 Water 74,000 Set-up 58,000 Indirect materials 104,000 [— ‘Operating supplies 62,000, Total test lab cost 1,262,460 Total estimated test hours 106,000 Shaw has determined the resource usage by test type in the chart shown below. Description BIT ATT ssT ACT ‘AQT Test pool labor employees 3 2 2 i 2 Supervision 40% 15% 15% 15% 15% Depreciation $48,230 | $22,000 | $39,230 } $32,000 | $37,000 Feat 30% 3% 3% 30% 10% Electricity 30% 10% 10% 40% 10% Water > 20% 20% 60% Set-up 20% 15% 30% 15% 20% Indirect materials 13% 15% 30% 20% 20% Operating supplies 10% 10% 25% 20% 35% Test hours / 29,680 12,720 | 27,560 | 22,260 | 13,780 Competitors hourly billing rates | $17.50 319 315.50 | S16 $20 Required A. Compute the common pool hourly cost and hourly billing rate for Best Test Laboratories. B. Compute the five separate costs for Best Test Laboratories. C. Discuss what effect the new costing method will have on the pricing structure for each of the five test types given the competitors hourly billing rates. D. In general, identify at least three other internal or external determinants of pricing structure. Solution: A. The common poo! hourly cost = Total test lab cost $1,262,460 Total estimated test, 106,000 = $11.91 per test hour ¥Commor hourly billing rate $11.91 x 145% = $17.27 per test hour 44 B. hourly billing rate by test type, for Best Test Laboratories are calculated below. Description Combined | HTT AIT ‘SST act | AQT Number of employees 10 3 2 2 T 2 Test pool labor (1) $420,000 | 126,000 | 84,000 | 84,000 | 42,000 | “84,000 Supervision @y 72,000 | 28800 | 10,800 | 10,800 | 10,800 | 10,800 Equipment Depreciation | 178,460 | $48,230 | $22,000 | $39,230 | $32,000 | $37,000 Heat 170,000 | 85,000 | 8,500 8500 | 51,000 | 17,000 Electricity 124,000 | 37200 | 12,400 | 12,400 | 49,600 | 12,400 Water 74,000 = : 14,800 | 14,800 | 44,400 Set-up 38,000 | 11,600 | 8,700 | 17,400 | 8,700 | 11,600 Indirect materials 104,000 | 15,600 | 15,600 | 31,200 | 20,800 | 20,800 Operating supplies 62,000 | 6,200 6200 | 15,500 | 12,400 | 21,700 Total lab cost $1,262,460 | $358,630 | $168,200 | $233,830 | $242,100 | $259,700 Total lab hours 106,000 | 29,680 | 12,720 | 27,560 | 22,260 | 13,780 Hourly Tab cost 1191 12.08 13.22 Ba 1088 | 18.85 Notes: (1) Allocated based on number of employees, using the appropriate ratio. (2) Except for depreciation. Other expenses are allocated based on given percentages. C. The new costing method will have the following effects on the pricing structure for each of the five test types given the competitors’ hourly billing rates. Description HIT ATT SST ACT AQT New hourly billing rate (145%) 317,52 $19.17) $1230 | Si.77 | $2733 Competitors” hourly billing rates | $17.50 $19 315.50 | $16 $20 New rate over/(under) market $02 3017 | $620 | $023) | $733 Percent over! (under) market 0.1% 09% | 206% | (A% | 36.7% ‘Common pool rate $1727 siz27_ | $1727 | $i727 | $1727 New rate over/(under) old rate $0.25 $1.90 | $4.97) | $0.50) | $10.06 45 Percent over/(under) old rate 14% 1% (28.8)% | (8&7)% 58.3% + Best Test will now be pricing all their lab tests more competitively in the market. + For Heat Testing (HTT), there is minimal variance between the common pool rate, the new separate rate, and the competitor's rate. The HTT rate could either be left at the old rate, or nominally raised to the competitor's rate or new pool rate without much impact, depending upon how Best Test wanted to position the test compared to the competition. + For air Turbulence Testing (ATT) and Arctic Turbulence Testing (ACT), the new separate computed billing rate is significantly different than the common pool rate as well as close to the competitor's rates. In both cases, Best Test would probably want to adjust billing rates (raise ATT rate and lower ACT rate) to the newly computed rates or competitors rates to better reflect resources consumed by the tests. + For Stress Testing (SST), the newly computed rate is dramatically less than both the common pool rate and the competitors’ rates. Best Test would want to significantly reduce the price to at least meet the competitor's price or reduce it further towards the newly computed price, depending upon how aggressively they wanted to market this test. + For Aquatic Testing (AQT), the newly computed rate is significantly higher than both the common pool rate and the competitors’ rate. Best Test would want to raise the billing rate at least to the competitors’ rate to recover their cost plus some contribution towards administrative costs. Their current common billing rate of $17.27 is below the $18.85 cost to perform the AQT test. + Because the newly computed billing prices for both SST and AQT are significantly different than competitors’ prices, the cost assumptions should be further analyzed to verify accuracy and identify opportunities. D. In general, at least three other internal or external determinants of pricing structure include the + Number and nature of competitors for additional tests and their quality and timeliness of service. + Company's overall capacity and its ability to react to volume and mix changes for tests if the demand changes due to the new pricing structure. + Number of potential customers, overall demand for the tests, and price elasticity of demand for the tests. + Strategic focus, such a8 desire to gain or defend market share, long-term support for entry into or exit from a market, or stage in the tests product life cycle (introduction, growth, mature, or dying). Conclusion here each cost item is incurred for the benefit of more than one cost object (test type) so each cost item is considered common cost, but each cost object (test type) consumes different amounts of each cost item thus each Common cost (cost item) does not benefit each test type (cost object) by equal benefits accordingly; allocating the amounts of these cost items (common cost) based on single driver such as test hours would give misleading results. Hence a cause and effect relationship or benefits received from each cost item by each cost object are proper methods far allocating each common cost to each cost object 63, See Gleim MCQ" #303, 320, 407, 408,467, 529, 531,548,549,552,5 46 Costing approaches Assume the following data are available: Cost item Planned ‘Actual On PR Qn PR DM SQ4KG SPSS AQ3KG ‘AP ST DL SH2DLH_ SRSI2 ‘AH2.5 DL AR S10 On Estimated $30,000 ‘Actual $28,000 | Base | O.His allocated based on DLH Output | Planned 1,200 units | ‘Actual 1,000 units [Notes ‘SI>PO =2 DLH «1,200 units =2,400 DLA SI-AO =2 DLH «1,000 units =2,000 DL AT-+A0 =2.5 DLH «1,000 units =2,500 DLH ‘WIP (DR) ‘Actual eosting Standard costing Normal costing | Extended costing system system system system D.M used (AQ AP) | (SQofinput for AO x (AQ * AP) (AQ = SP) sp) DL (AH x AR) (SIP for AO x SR) (AH? AR) (AIP SR) Factory O.H- ‘Actual Rx (SISA0) R*(AISA0) | RX(ATDAO) | applied 1- Under Actual costing system no variances can be calculated for both D.M and D.L and there is no ‘under / over applied factory O.H because WIP is charged with actual factory O.H. 2- Under Standard costing system all manufacturing cost variances can be calculated (the best variance calculation and analysis approach) 3+ Under a Normal costing system, factory overhead is applied at a predetermined factory O.H allocation rate x Actual input of the cost driver for the actual output, this system allows to calculate under / over applied factory O.H but no variances can be calculated for both D.M and D.L. Normal costing system is the most often used system, Q: From the following budgeted data, calculate the budgeted indirect cost rate that would be used in a normal costing system. Total direct labor hours 250,000, direct costs $10,000,000, Total indirect labor hours $0,000, Total abor related costs $7,000,000 indirect-labor-related costs $5,000,000, Total indirect non: 47 A820 B- $28 c-$40 D- $48 Answer (D) is correet: ($5,000,000 + $7,000,000) +250,000 DLH = $48 per DLE 4- Extended costing system is used to 1) Avoid actual price or actual rate fluctuations during the year 2) When the price of DM or rate of D.L is subject to an underlying that is not known until the end of the period. Q: Which method of measuring the costs to be assigned to produets or services uses budgeted rates for direct inputs but applies those rates to the actual quantities of the inputs? A- Actual costing. B- Normal costing C- Extended costing. D- Standard costing Answer (A}is incorrect because actual costing uses only actual direct and overhead costs Answer (B) is incorrect because normal costing uses a budgeted rate only for overhead costs. ‘Answer (C) is correct. Extended costing assigns both direct costs (such as labor and materials) and overhead to cost objects (such as WIP) by using budgeted rites. The direct eost assigned equals the budgeted rate times the actual amount of the direct-cost input. The overhead assigned equals the budgeted rate times the actual amount of whichever driver or other base is used for cost assignment purposes. The use of budgeted rates, for overhead as well as direct costs may be helpful to avoid fluctuations during the year. Its also helpful when some direct costs, such as direct labor, may not be known until year-end. Answer (D) is incorrect because standard costing applies budgeted rates to the standard (not actual) inputs allowed for actual output. Factory pplied to WIP could be either ‘Based on Standard Costing System Based on Normal Costing system Predetermined O.H. application rate Predetermined O.H. application rate * Planned level of activity driver for actual | Actual level of activity driver for actual Production (output) Production (output) Rx (SI>A0) Rx(AIDAO) Gy SElye Sh doe A ds GI 9 1 Goby Standard Costing : Normal costing systems are said to offer a user several distinct benefits when compared with actual costing systems. Which one of the following is not a benefit associated with normal costing systems? A Timelier costing of jobs and products, B- A smoothing of product costs throughout the period. C- Improved accuracy of job and product costing D- A more economical way of attaching overhead to a job or product. Answer (C) is correct. 48 Q: Wagner Corporation applies factory overhead based on machine hours. At the beginning of the year, Wagner budgeted factory overhead at $250,000 and estimated that 100,000 machine hours ‘would be used to make 50,000 units of product. During the year, the company produced 48,000 units, using 97,000 machine hours. Actual overhead for the year was $252,000. Under a standard cost system, the amount of factory overhead applied during the year was ‘A-240,000 B- 242,500 €-250,000 D-252,000 Answer (A) is correct: $250,000 100,000.M.H 1- Factory O.H applied (Standard cost system) = $2.5 per M.H * 2 M.H 48,000 units = 240,000 2- Factory O.H1 applied (Normal costing system) = $2.5 per M.H * 97,000 MJ =82.5 per MH = 242,500 Q: felicity Corporation manufactures a specialty line dresses using a job-order cost system. During January, the following costs were incurred in completing job J-l Direct materials $27,400, direct labor (400 DLH) $9,600, Administrative costs 2,800, Selling expenses 1 1,200.Factory overhead was applied at the rate of $50 per direct labor hour, if job J- | resulted in 4,000 good dresses, what is the cost of goods sold per dress Answer Total manufacturing costs (j-1) = D.M + D.L + factory O.H applied Total manufacturing costs (j-1) = $27,400 + $9,600 + ($50 x 400 DLH) = $57,000 5 COGS per unit = $57,000 + 4,000 units = $14. Q: A company allocates factory overhead te jobs in process using raw material costs, direct labor costs, and machine hours. The overhead application rates for the current year are. 20% of direct materials cost, 100% of direct labor cost and $117 per machine hour, a particular production run incurred the following costs: Direct materials, $2,000, direct labor, $8,000 and a total of 140 machine hours were required for the production run, What is the total manufacturing cost of the production run? Answer ‘Total manufacturing costs of the production run = D.M + D.L + factory O.H applied Factory O.H applied = ($2,000 * 20%) + ($8,000) + (140 M.H x $117) =$24,780 2,000 +$8,000 + $24,780 = $34,780 ‘Total manufacturing costs of the production run = 49 Q: Watson Company uses a predetermined factory overhead application rate based on direct labor cost (denominator). Watson’s budgeted factory overhead was $756,000 based on a budgeted volume of 60,000 direct labor hours, at a standard direct labor rate of $7.20 per hour. Actual factory overhead amounted to $775,000 with labor efficiency variance equal zero and actual volume of output equalled budgeted volume, actual direct labor cost of $450,000 for the year ended December 31. How much was Watson’s overapplied factory overhead? Answel Predetermined Factory O.H. Rate . budgeted. factory.0.H ~» $756,000 budgeted activity of cost.driver > 60,000DLH x $7.2 Predetermined Factory O.H. Rate = $1.75 factory of O.H to be applied per one dollar of D.L. costs Labor efficiency variance equal zero means actual D.L.H worked for actual output is the same as budgeted = 60,000 D.L.H Thus actual per hour direct labor cost = ae sms SIAO = 60,000 * $7.2 = $432,000 DR Factory O.Hf control Factory O.H applied CR “| I Standard costing system R * SIDAO Actual factory O.H $1.75 * (60,000 DLH * $7.2) = $756,000 $775,000 | Thus undeapplied factory O.H would be $19,000 | Normal costing system R x AI AO. $1.75 x $450,000 = $787,508 ‘Thus overapplied factory O.H would be $12,500 Conclusion The amount and direction of under /over applied factory O.H depend on the costing approach used as well as cost driver chosen to allocate factory O.H. 50 Q: Selected Actual data of North coast’s operations for the year just ended is presented below. Description (actual data) ‘Amount(S) Quantity of Products manufactured 648,000 units "Machine utilization 130,000 hours Direct labor usage 53,000 hours Labor rate $14.5 per hour Total factory overhead $1,389,442 Cost of good sold $1,720,960 1ed goods inventory (at year-end) $430,240 Work-in-process inventory (at year-end) 30 1. Total factory O.H is applied based on direct labor cost using a predetermined plant-wide rate 2. The budgeted activity for the year included 20 employees each working 2,250 productive hours per year to produce 540,000 units of product. The machines are highly automated, and each employee can operate two to four machines simultaneously. Normal activity is for each employee to operate three machines. 3. Machine operators are paid $15 per hour 4. Budgeted factory O.H costs for the past year for various levels of activity are shown in the table below. Norhtcoast’s Manufacturing Company Budgeted Annual Costs For Total Factory Overhead Units of product 360,000 540,000 720,000 Labor hours (12 units per D.L.H) 30,000 45,000 60,000 | Machine hours (5 units per M.H) | 72,000 108,000 144,000 | Total factory overhead costs Plant supervision ———=—=—=—«]==«970,000 | $70,000 | ~—*$70,000 Plant rent 340,000 $40,000 $40,000 (~~ Equipment depreciation($4 per M.H) 288,000 | 432,000 576,000 Maintenance 42,000 56,000 60,000 144,600 356,200 388,600 Indirect material 90,000 156,000 180,000 Other manufacturing costs 11,200 97,300 162,000 [Total estimated factory overhead costs ‘$685,800 | 1,201,500 $1,476,600 Sl Predetermined factory O.H. application rate under single rate method (plant-wide rate) $1,201,500 —» budgeted annual. plant.wide. factory.O.H..costs.(based.on.540,000.units 20.employee x 2,250.D.L.H.per employee.per year x$15.per.D.L.H R= $1.78 of factory O.H to be applied per one $ of D.L cost. (DR. Factory O.H controls (actual) cR. Total actual factory O.H $1,389,442 | ee DR. Factory O.H applied-Normal CR. Factory O.H applied Rx (AIS AQ) $1.78 x 53,000 D.L.Hx $14.5 $1,367,930 i DR. Factory O.H applied-Standard CR. Factory O.fi applied R x (SISAO) _648,000.units (Qunits.per.D.LH x $15 x $1.78 nae Under normal costing system Underapplied factory 0.H costs = Actual factory O.H $1,389,442 ~ Applied factory O.H $1,367,930 = $21,512 Ver s | % | Underapplied 0.8. ANlocation DR. Factory O-H. applied $1,367,930 ‘WIP 0 0% $21,512 x0%=0 | DRE. $4,302 FG. | 8430240 | Bo% | SITSID 20% = 84,502 PR. COGS $17,210 COGS | 1,720,960 | 80% | $21,512 * 80%=S17.210 | CR: Factory O.H. control $1,389,442 Fotals | $2,157,200 | 100% $21,512 Assume machine hours were used as an application denominator $2 $1,201,500 108,000.M.H $11.125 of factory O.H to be appl Under Normal costing system Factory O.H applied = R * AI> AO =$11.125 per M.-H * 130,000M.H = $1,446,250 Under standard costing system 648,000.uni¢s Factory O.H applied = R x (SI9AO) % $11,125 per M.H = $1,441,800 Suunits. per. M.H Pane Company uses a job costing system and applies factory overhead to produets on the basis of direct labor cost, Job No. 75, the only job in process on January 1, had the following costs assigned as of that date: direct materials, $40,000 direct labor, $80,000; and factory overhead, $120,000. The following costs were incurred during the year: ‘aceable to jobs: Direct materials $178,000 Direct labor 345,000 Total $523,000 Not traceable to jobs: Factory indirect materials and supplies, $46,000 Indirect labor 235,000 Plant maintenance 73,000 Depreciation on factory equipment 29,000 Other factory costs 76,000 Total 459,000 Pane's profit plan (budget) for the year included budgeted direct labor of $320,000 and budgeted factory overhead of $448,000, Assuming no work-in-process on December 31, calculate over / under applied factory O.H for the year Predetermined single O.H allocation rate = S118:000 $320,000 7 S140 factory O.H per one dollar of direct labor cost. Factory O.H Actual Applied $ 459,000 $1.4% $345,000 = $483,000 Over applied O.H. $24,000 Q: At the beginning of the year, Smith Inc. budgeted the following: Description —————SS~*~*~«~SC=C:«é‘ mount (Units to be produced and sold ——S—S*S*~«@:SC*«~C«éiC OO i Sales S~*~CS~SS~«SSLOD ["Minus: Total variable expenses —s—=~*~S~S~«S 0000 “Total fixed expenses ——sS=S=~*~«dY:=C*«<“i«i CS, | “Operating income ——S~S~S~«~«S*S*«S 0,000” Factory overhead: Variable $30,000 and Fixed 10,000 ‘There were no beginning inventories. At the end of the year, no work was in process, total factory overhead incurred was $39,500, and underapplied factory overhead was 1,500. Factory overhead was applied on the basis of budgeted unit production, How many units were produced this year? A.10,250 B.10,000 C. 9,875 D. 9,500 Answer (D) is correct. __ $30,000 + $10,000 Predetermined single O.H. rate 10,000.units = $4 per unit of output 54 DR. Factory 0. controls (Actual) cR. Total actual factory O.H $39,500 DR. Factory O.H applied-Normal CR. Factory O.H applied = $39,500 - $1,500= | $38,000 Rx (AO) $4 xActual Q of output = $38,000 Actual Q of output =238:02 - 9,500 units Q: When using normal costing, $1,000,000 of overhead was allocated to the production process but $1,020,000 was actually incurred. Which of the following is true of this situation? A. Ifimmaterial, the overapplied overhead decreases the cost of goods sold. B, If material, the underapplied overhead increases the cost of goods sold. C. If material, the overapplied overhead decreases the cost of goods sold. D. If immaterial, the underapplied overhead increases the cost of goods sold. Answer (D) is correet. Since the amount of overhead incurred was greater than what was, budgeted, the difference is the amount that was underapplied. Only immaterial amounts are allowed to directly adjust the cost of goods sold. Conclusion ‘Single (Plant wide) O.H. rate uses only one factory O.H. control account and only one predetermined factory O.H. application rate, Single rate is less complex and less precise than departmental O.H. rates, ‘Single O.H. rate is only appropriate in the following cases: A) Ifall factory O.H. costs are highly correlated with a single cost driver (application base) (i.e.) cost driver is the same over all processes. B) Manufacturing process is limited to single product passing through identical departments in a fixed sequence (i.e.) the manufacturing processes are homogencous. Thus Single O.H. rate is used with a firm with a relatively simple production and accounting system. firm using a plant-wide overhead rate has four departments with the following overhead amounts: $200,000, $300,000, $100,000, and $200,000. The same departments respectively have the following annual labor-hours totals: 30,000, 40,000, 20,000, and 10,000. Assuming labor- hours are wsed as the allocation base, what overhead rate will the first department use? A. $2/labor-hour —-B. $6.67/labor-hourC. $10/labor-hour _D. $8/labor-hour Answer (D) is correct. Since the plant uses a plant-wide overhead rate, the overhead amounts and the labor-hours amounts are totaled and the total plant overhead is divided by the total units of the cost driver: $800,000/1 00,000 labor-hours = $8/labor-hour. Q: Generally, individual departmental rates rather than a plant wide rate for applying ‘manufacturing overhead are used if A. A company wants to adopt a standard cost system. B. A company’s manufacturing operations are all highly automated C. Manufacturing overhead is the largest cost component of its product cost. D. The manufactured products differ in the resources consumed from the individual departments in the plant. Answer (A) is incorrect because a standard cost system can be based on individual or multiple application rates. Answer (B) is incorrect because whether production is machine intensive affects the nature but not necessarily the number of cost drivers. Answer (C) is incorrect because a single plant-wide application rate is acceptable, even with high overhead, if all overhead is highly correlated with a single application base. Answer (D) is correct. Overhead is usually assigned to products based on a predetermined rate or rates. The activity base for overhead allocation should have a cause effect with the incurrence of overhead. Given only one east driver, one overhead application rate is sufficient. If products differ in the resources consumed in individual departments, multiple rates are preferable. Q: Generally, individual departmental rates rather than a plant-wide rate for applying overhead would be used if A, The manufactured products differ in the resources consumed from the individual departments in the plant. B. A company's manufacturing operations are basically labor based C. A company wants to adopt a standard cost system. D. A company's manufacturing operations are all highly automated Answer (A) is correet. individual departmental rates rather than a plant-wide rate for applying overhead would be used if the departments have different cost drivers and the manufactured products differ in the resources consumed from the individual departments in the plant, For example, one department may be labor-intensive, while another may be machine-intensive, or use highly automated processes, See Gleim MCQ" # 530, 534,535,536,537,538,539,545,553, Dual plant wide Pre-determined O.H. application rate Q: Nona Company manufactures a line of products distributed nationally through wholesalers. Presented below are planned manufacturing data for the year and the month of November also actual data for November of the current year. The company applies overhead based on planned machine hours using a predetermined annual rate Financial and quantitative data Description (Planned) | Annual [November [Fixed manufacturing overhead =——s—=~*~S:=SC«S:*T, 200,000 $100,000 Variable manufacturing overhead 2,400,000 200,000 Direct labor hours(D.L.H) 48,000 4,000 Machine Hours (M.H) 240,00 20,000 Actual Data for November ‘Actual Direct labor hours (D.L.H)(AI>A0)__—|f 4,200 Direct labor hours (plan based on output) (SISA0)__| 4,000 ‘Actual Machine hours (MH) (AISA0)—| 21,600 ‘Machine hours (plan based on output)(SIDA0) «| SS~*~«ST;« CS ‘Actual Fixed manufacturing overhead ———=—S—=«~S*~*~*~*S«SC‘T 200 | ‘Actual Variable manufacturing overhead $214,000 | Determine single and dual O.H rates and O.H applied based on normal and standard costing systems and over or underapplied O.H in each case Ans er Pre-determined O.H. application rate based on M.H (single rate) Re Lotal.budgeted.annual.factory.O.H ~> $1,200,000 + $2,400,001 Annual estimated .activity of cost.driver —> 240,000.MH. = S15 per MH Pre-determined O.H. application rate based on M.H (Dual rate) Preferable | 1) Pre-determined fixed O.H. application rate based on MH Annual estimated. fixed.O.H $1,200,000 . ae eee eee —_= $5 per M.H Annual estimated activity of .cost-driver.240,000.M.H. 2) Pre-determined variable O.H. application rate based on M.H Annual.estimated.var iable.O.H$2,400,000 ——— = $10 per MB stimated activity.of .c03t.driver.240,000.MH. Annual. Note: denominator (Activity driver could and should be different for each rate) 37 DR. Factory O.H controls (actual-November) cR. | Fixed actual 0. | $101,200 "Variable actual OH $214,000 | Total actual factory O.H | $315,200 DR. Factory O.H applied-Standard (November) CR. i Fixed O.H applied (R x SI>AO) $5 * 21,000 M.H T "$105,000 Variable O.H applied (R x SIA) $10 * 21,000 MH $210,000 Total applied factory OH (standard costing approach) $315,000 | DR. Factory O.H applied-normal (November) CR. | Fixed O.H applied (R XAI-SA0) $5 * 21,600 M.H $108,000 Variable 0-H applied (R *AT> AO) $10 * 21,600 MH $216,000 | | ‘Total applied factory O.H (standard costing approach) $324,000 | Over/ underapplied (standard costing) ‘Over/ underapplied (normal costing) ——-— Actui Fixed $101,200 Variable 0. $214,000 Total O.H $315,200 Applied 105,000 Over/under $3,800 (over- Favourable) 210,000 | $4,000 (under- unfavourable) 315,000 $200 (under) ‘Over/under $101,200 |] $108,000 Variable OH |} $216,000 $214,000 $6,800 (over-Favourable) $2,000 (over-Favourable) Total OH |} $324,000 $315,200 $8,800 (over-Favourable) 58 ‘rom previous question assume the company applies factory O.H based on D.L-H (labor intensive manufacturing process) what is the over/under applied factory O.H based on standard and normal costing approaches. Q: Pate Company uses a standard FIFO, process-cost system to account for its only product, Matura. Pate has found that direct machine hours (DMH) provide the best estimate of the application of O/H. four (4) standard direct machine hours are allowed for each unit, Using simple linear regression analysis in the form Y= A +B (DMH) Pate has developed the level of 100,000 direct machine hours following factory O/H budget for a normal aetiv |) A |), 8B Factory O.H ITEM (Y) (fixed) (variable per D.M.H) ‘Manufacturing Supp! > | $0.50 Indirect labor $54,750 6.50 Depreciation - Plant and Equipment 27,000 - Property Taxes and Insurance 32,300 - Repairs and Maintenance 14,550 1.25 Utilities 3,400 475 Total O/H $132,000 313 Actual fixed O/H incurred was $133,250, and actual variable O/H was $1,225,000 Pate produced 23,500 equivalent units during the year using 98,700 direct machine hours. r Plant wide predetermined factory O.H allocation rate $132,000 +$13per D.MH = $14.32 — > ” 100,000.D.M.HT | Plant wide predetermined variable factory OH | Plant wide predetermined fixed factory allocation rate OH allocation rate = =813 per D.M.H $132,000 6 39 per D.VLH 100,000.D.M.H 59 [ Description ‘Actual ‘Applied—Standard | Applied-normal | factory O. R*SIDA0 R XAIDAO i Fixed $133,250 | $1.32 * 23,500 units x 4 [$1.32 »98,700 DMH= D.M.H= $124,080 $130,284 | Variable | $1,225,000 | $13%23,500 units «4 ~ | “$13*98,700 DMN = | |__DMHe $1,222,000 $1,283,100 Total factory OH $1,358,250 | $1,346,080 $1,413,384 Q: Alpha Company paid janitors $5 per hour to clean the production area. It initially set the standard cost of janitorial work at $4.50 per hour. What is the appropriate entry to record the application of the 530 hours worked by the janitors? $2,385 CR Variable O/H applied $2,385 According to normal costing system factory O.H applied to WIP = Predetermined 0.1 rate $4.5 per hour * 530 actual hours worked = $2,385 Q: Alpha Company paid janitors $5 per hour to clean the production area. If 530 hours were worked by the janitors for the month of June .The proper journal entry to account for this expense is as follows: k Account title | DR cR DR T t | Factory 0.H control (variable) $2,650 cR Payroll payable or cash 1 $2,650 Q: When allocating costs from one department to another, a dual-rate cost-allocation method may be used. The dual-rate cost-allocation method is most useful. when A. Two of more cost pools are to be allocated. B. Two of more departments’ costs afe to be allocated. C. Two or more products are produced D. Costs are separated into variable-cost and fixed-cost sub pools. Answer (D) is correct. Q: Using the following budget data for Valley Corporation that produces only one product. Calculate the company's predetermined factory overhead application rate for variable overhead Units to be produced 11,000 Units to be sold 10,000 Indirect materials, varying with production $1,000 Indirect labor, varying with production 10,000 Factory supervisor's salary, incurred regardless of production 20,000 Depreciation on factory building and equipment 30,000 60 Utilities to operate factory machines 12,000 Security lighting for factory 2,000 Selling, general and administrative expenses 5,000 A. $2.09 B. $2.30 C. $4.73 D. $5.20 Answer (A) is correct Variable factory O.H = Indirect materials $1,000 + Indirect labor $10,000 + Utilities to operate factory machines $12,000 = $23,000 $23,000 Variable overhead allocation rate =~ = 52.09 ‘arial rhea cee 61 Allocation of common costs among Joint products Joint Process (Common process) ‘Yloint process is an operation (manufacturing / mining...) that yield two or more identifiable products from the resources employed in the process. Further processed (Then separate costs are ~Join process in which ‘Join products added) Common costs are spent |». Become separable ram panee™ | "o> oR Split — off point Sold immediately After split— off point ‘Joint products (main products) may be accompanied by “by- products” that produced simultaneously from common manufacturing process with produets of greater value (joint products) Characteristics that identify joint products (Main produets) 1. Joint Products become identifiable (separable) as individual products only up-on reaching the split-off point. 2, Joint products-in contrast to by - products, have significant sales value in relation to each other, either before or after additional processing, Notes: A, Physical volume has nothing to do with determining a joint product, some products although have significant physical volume but its sales price is insignificant, B. Joint products do not have to be salable at split-off but it can be further processed after split-off point. C. By-product does not have to be salable at split-off but it can be further processed after split-off point. D. By-product although is identifiable as an individual product only upon reaching split-off point but it has relatively minor sales value when compared with joint products, Q: The distinction between joint products and by-products is largely dependent on A Historical costs B. Prime costs. —C. Market value. Salvage value Answer (C) is correct Q: Ina production process where joint produets are produced, the primary factor that will distinguish a joint product from a by-product is the A. Relative total sales value of the products. B. Relative total volume of the products. C. Relative eases of selling the products, D. Accounting method used to allocate joint costs. Answer (A) is correet. Further processing of joint products ‘Common costs are irrelevant in determining whether to continue producing an item, but the decision is based on the comparison of incremental cost with incremental revenue, Q: Beagle Company produces three products (B-40, J-60, and H-102) from a single process. Beagle uses the physical volume method to allocate joint costs of $22,500 per batch to the products. Based on the following information, which product (s) should Beagle continue to process after the split off point in order to maximize profit? Description B40 ¥-60 H-102 Physical units produced per batch 1,500 | _2,000 3,200 Sales value per unit at split off S10 M4 $7.25 Cost per unit of further processing after split off 3.05 1 25 Sales value per unit after further processing 12.25 5.70 9.75 A. B-40 only B, J-60 only C. H-102 only D. 8-40 and H-102 only Answer (B) is correct. The units should be processed after the split off point only if the additional cost is tess than the additional revenue. Only J-60 units warrant further processing: Description B-40 [5-60 H-102 Incremental revenue per unit $2.25 $1.70 32.50 Cost per unit of further processing after split off $3.05 31 $2.5 Recommendation for further processing NO YE! NO : The Raines Company operates a plant that produces two products from a eommon, yaw material. The following information is known about the joint processing operation: + Raw material A is processed into product X and product Y. + Each 1,000 units of raw material A yield 700 units of product X and 300 units of product Y. + Raw material A costs $40 per unit. + To process 1,000 units of raw material A up to the split-off point costs $40,000; the company allocates these costs to the two products on the basis of their sales value at the split-off point. + Product X can be sold immediately for $50 per unit, or it can be processed further ard sold for $62 per unit; additional processing costs are $7 per unit. ‘What is the net benefit or loss on a per-unit basis to sell product X at the split-off pointor process it further? Correct answer: $5 it is profitable to continue processing a product as long as the incremental revenue received (the revenue attributable to the added processing) exceeds the incremental processing costs incurred. Remember that joint costs are irrelevant to a decision relating to selling at split off point or further processing. They are common costs that must be incurred to get the product or service to the split-off point. Q: Whitehall Corporation produces chemicals used in the cleaning industry. During the previous month Whitehall incurred $300,000 of joint costs in producing 60,000 units of AM-12 and 40,000 units of BM-36. Whitehall uses the units-of-production method to allocate joint costs. Currently, AM-12 is sold at split-off for $3.50 per unit. Flank Corporation has approached Whitehall to purchase all of the production of AM- 12 after further processing. The further processing will cost Whitehall $90,000. Concerning AM-12, which one of the following alternatives is most advantageous? A. Whitehall should process further and sell to Flank if the total selling price per unit after further processing is greater than $3, which covers the joint costs. B, Whitehall should continue to sell at split-off unless Flank offers at least $4.50 per unit after further processing, which covers Whitehall’s total costs. C. Whitehall should process further and sell to Flank if the total selling price per unit after further processing is greater than $5. D. Whitehall should process further and sell to Flank if the total selling price per unit afier further processing is greater than $1.59, which covers the incremental costs. Answer (C) is correet, Whitehall should process the AM-12 after split-off as long as the increase in market value (selling price) -at the split-off point is greater than the additional processing costs of $1.50 per unit, AM-12 can be sold at the split-off point for $3.50 per unit. The increase in selling price after further processing has to be greater than the $1.50 per unit processing costs. Therefore, the final selling price has to exceed $5 ($3.50 + $1.50). The joint costs of $300,000 are irrelevant to the decision, since they are sunk costs. Common costs -Joint costs ‘¥Common costs are incurred in the production of two or more inseparable products up to the point at which the products become separable (split-off point). vin general common costs are incurred to benefit more than one cost obiective (products, divisions, departments) 64 ¥Common costs are direct cost to the common process itself but are indireet costs to the joint products, because they can't be traced to a particular (specific) cost object (product) in an economically feasible manner. Hence, joint costs (common costs) must be allocated. This represents the core problem of this paper. ¥The common costs are allocated to main products and secondary products (if any) but By-product never charged any portion of common cost. Reasons to allocate Common Costs: A. Proper valuation of inventory (for unsold units) B. Proper matching of revenue and expenses (for sold units) C. asa secondary purpose for common cost allocation is the desire to full cost products to aid in pricing decisions if prices are based on cost plus basis, But generally selling price is subject to many other variables that must be considered in setting selling prices such as additional processing costs, competitive conditions in sales markets etc... Q: The primary purpose for allocating common costs to joint produets is to determine A. The selling price of a by-product B. Whether or not one of the joint products should be discontinued C. The variance between budgeted and actual common costs. D. The inventory cost of joint products for financial reporting. Answer (A) is incorrect because by-product never charged any portion of common cost. Answer (B) is incorrect because joint products can’t be discontinued individually. Answer (C) is incorrect because to determine the variance between budgeted and actual common ¢ests no need for common cost allocation. Answer (D) is correct. Q: Allocation of Joint costs is useful for A. Setting the selling price of a product B, Determining whether to continue producing an item. C. Evaluating management by means of a responsibility reporting system D, Determining inventory cost for accounting purposes Answer (A) is incorrect because items such as additional processing costs, competitive conditions in sales markets, and the relative contribution margins of 65 all products derived from the common process must be considered in setting selling prices. Answer (B) common costs are irrelevant in determining whether to continue producing an item, but the decision s based on the comparison of incremental cost with incremental revenue. Answer (C) is incorrect because management of one department may have no control over joint costs Answer (D) is correct. Joint costs are useful for inventory costing when two or more identifiable products emerge from a common production process. The joint costs of production must be allocated on some fair basis. Rule (1): by- products never allocated any portion of common costs, Rule (2): If production quantities differ from sales quantities of the joint products the joint costs are allocated always based on quantities produced rather than quantities ld ‘Methods of allocating common-costs among joint (main) products 1, Physical Quantity method, _2, Relative sales value at split -off point. 3. Net-realizable value (NRV) 4, Gross profit method 5. Gross market value method _6. Other- non financial bas Q: All of the following are methods of allocating joint costs to joint products exee A. Physical quantities method. B, Net xealizable value method. C. Separable production cost method. D. Gross market value method. Answer (C) is correct. (1) Physical Quantity Method Q: Sawmill manufactures two lumber products from a joint milling process. The ‘two products developed are mine support braces (MSB) and commercial building lumber (CBL) A standard production run incurs joint costs of $300,000 and results in 60,000 units of MSB and 90,000 units of CBL. Assuming no further processing is done after the split-off point, the amount of joint cost allocated to cach product on a physical quantity allocation basis would be? 66 i T ] Soint products | Quantity-units % Joint cost allocation i MSB 60.900 40% | $300,000 «40% =$120,000 1 lca 90,000 60% | $300,000 * 60% = $180,000 Ta 150,000 100% $300,000 Q: The principal disadvantage of using the physical quantity method of allocating joint costs is that A. Costs assigned to joint produets may have no relationship to value. B. Physical quantities may te difficult to measure C. Additional processing costs affect the allocetion base, D. Joint costs, by definition, should not be separated on a unit basis. Answer (A) is correet. Joint costs are often allocated on the basis of relative sales values at split off or net realizable values. Basing allocations on physical quantities, such as pounds, gallons, ete, is usually not desirable because the costs assigned may have no relationship to value. When large quantities items have relatively low selling prices and small quantities items have relatively high selling prices, the large quantities items might always sell at a loss when physical quantities are used to allocate joint costs. Answer (B) is incorrect because physical quantities are usually easy to measure. Answer (C) is incorrect because additional processing costs will have no effect on the allocation of joint costs based on physical quantities. Answer (D) is incorrect because the purpose of allocating joint costs, under any method, is te separate such costs on a unit basis. 67 Accounting treatment of the yroduet Q: Alas Foods produces the following three supplemental food products simultaneously through refining process costing $93,000. Morefeed is a by-produet, is sold atthe split-off point for $3 per pound Product (Quantity -pounds Assuming Atlas Foods inve ‘Alpha Beta 10,000 5,000 1,000 Morefeed entories Morefeed, the by-product, what is the joint cost to be allocated to alpha and beta, using the physical quantity method? ‘The by-product (Morefeed) net realized value (NRV)= Selling price -any additional cost of by-product =1,000 pound * $3 - Zero = $3,000 By - Product NRV either Inventoried Does not inventoried Used to reduce joint costs (common cost) by its estimated NRV and allocating the remainder of common cost to joint products [Preferable and consistent with GAAP] DR By-Productinventory 3,000 CR WIP (of joint process) 3,000 The effect of previous entry is reducing inventoriable costs of the main products and establishing an inventory account for the by- product thus inventoriable costs are the same amount but taken two forms (main product inventory and by-product inventory). Also income statement is not affected ‘When the by-product is sold DRCASH 3,000 CR By-Product inventory 3,000 Once more income statement is not affected, But Ending inventory is reduced. ‘Means allocable joint costs are not reduced by the NRV of by: Product Thus the total joint costs are allocated to joint products. Accounting treatment 1. If the by-product has a high salability and stable market value then revenue recognition could be occurred hefore the point of sale (i) the by-Product inventory is valued at its estimated selling price. DR By- Product inventory CR Other revenues 3,000 3000 ‘The effect of previous entry is an increase in both ending Inventories and net income, ‘When by-product is sold DR Cash (or A/R) > Actual selling price 3,000 CR By-Product inventory 3,000 The effect of previous entry is reducing ending inventory but no effect on net income, 2. If the value of by-product, can’t reasonably estimated or its selling price is fluctuating greatly then revenue recognition would be deferred until the event of sale. And only memorandum entry is used to record By-Product inventory 68 joint | Quantity, % Allocation of joint costs n of joint costs Products (IENRY of by-product is (IFNRV of by-product doesn’t | inventoried) inventoried Aipha VODA pounds | I 90,000 * 2/5 = $60,000 93,000 * 2/3 = $62,000 Beta | 5,000 pounds | 1/3 1 90,000 * 1/3 = $30,000 93,000 * 1/3 = $31,000 Total 15,000 pounds 100% $90,000 93,000 Q: A company produces three main joint products and one by-product. The by-product's relative sales value is quite low compared with that of the main products. The preferable accounting for the by-product's net realizable value is A. An addition to the revenues of the other products allocated on the basis of their respective net realizable value. B. Revenue in the period it is sold. C.A reduction in the common cost to be allocated to the three main products. D. A separate net realizable value upon which to allocate some of the common costs. Answer (A) is incorrect because treating the net realizable value of a by-product as an addition to the revenues of the other main products is not relevant. Answer (B) is incorrect because the NRV is ordinarily recognized as a contra cast (reduces the allocable common costs to joint products) in the period the by-produet is produced. Answer (C) is correct. Because of the relatively small sales value of by-products, a cost-effective allocation method is used for by-products. The net realizable value of by-product is usually deducted from the common costs that are allocable to the main products (The preferable accounting for the by-products net realizable value is to be inventoried) Answer (D) is incorrect because recognition ofa separate net realizable value upon Which to allocate some of the common costs means allocating some of common costs to by-products, this procedure is not relevant at all Q: A lumber company produces two-by-fours and four-by-eight as joint products and sawdust as a by-product. The packaged sawdust can be sold for $2 per pound. Packaging costs for the sawdust are $ 0.10 per pound and sales commissions ate 10% of the sale price. The by-product net revenue serves to reduce joint processing costs for joint products (inventoried) Joint products are assigned joint costs based on board feet. Data follows: Joint processing costs $50,000 ‘Two-by-fours procluced (board feet) 200,000 Four-by-cights produced (board feet) 100,000 69 ‘Sawdust produced (pounds) 1,000 What is the cost allocated to both main products? er NRV of by-product = 1,000 [($2) ~ ($0.10 + $0.20)] = $1,700 Allocable joint processing costs = $50,000 - $1,700 = $48,300 Joint-products Quantity of board feet | % Allocation of net joint costs 2x4 200,000 2B $48,300 x 2/3 = $32,200 4x8 100,000 1B $48,300 * 1/3 = $16,100 Total 300,000 100% $48,300 70 (2) Relative sales value at split-off point This method of joint cost allocation can be used if 1) Joint products can be sold at the split-off point and as a result 2) Selling price for each joint product is available at split-off point ¥ Note: Allocation of C.C is based on relative sales value at split- off rather than relative sales price per unit at split- off point because quantities of each main product could differ Q: Sawmill manufactures two lumber products from a joint milling process. The two building lumber (CBL) A standard production run incurs joint costs of $300,000 and results in 60,000 units of MSB and 90,000 units of CBL. Each MSB sells for $2 per unit, and each CBL sells for $4 per unit, calculate products developed are mine support braces (MSB) and comme! ‘The amount of joint cost allocated to both products based on a relative sales value at split off. int Product Quantity * Selling Price | % Joint cost allocation MSB 60,000 * $2= $120,000 | 25% $300,000 * 25% = $75,000 CBL 90,000 x $4=$360,000 | 75% | $300,000 *75% =$225,000 Total $480,000 100% $300,000 Q: A company manufactures products (X) and (Y) using a joint process. The joint processing costs are $10,000, Products (X) and (Y) ean be sold at split-off for $12,000 and $8,000, respectively. After split-off, product (X) is processed further at a cost of $5,000 and sold for $21,000, whereas product (¥) is sold without further processing. Ifthe company uses a method based on sales value for allocating joint costs, what is the joint cost allocated to both products? Answer: The appropriate method to use is relative sales value at split-off point because 1) Both joint products can be sold at the split-off point and 2) Selling price for each joint product is available at slt-off point. Joint Product Selling price at split-off | % Toint cost allocation a $12,000 “| 60% $10,000 » 60% = $6,000 w 38,000 40% | $10,000 *75% = $4,000 Total $20,000 100% $10,000 1 Note: NRY of product (X) ($21,000 - $5,000) is ignored because sales prices at split-off are known and both products can be sold at split-off point. Q: Tucariz. Company processes Duo into two joint products, Big and Mini, Duo is purchased in 1,000 gallon drums for $2,000, Processing costs are $3,000 to process the 1,000 gallons of Duo into 800 gallons of Big and 200 gallons of Mini. The selling price is $9 per galton fot Big and $4 per gallon for Mini. If the sales value at split off method is used to allocate joint costs to the final products, the per gallon cost (rounded to the nearest cent) of producing Big is A. $5.63 per gation B.$5 per gallon, C. $4.50 Per gallon __D. $3.38 per gallon. Answer (A) is correet [Description | Qatsplit [Pper | Total sates value | Allocation | Common cost | Common eost | of | Ga) at split off % allocation Per gallon Big B0G 1 | 720 «| 80% 34500 HMI i Mini 1 200 *| 4 $800 | 10% | $500 $2.50 Tow! | 1,000 | mal $8,000 ( 100% $5,000 = Q: From previous question If the sales value at split-off point is used to allocate joint costs to the final products, the total cost assigned to produce Big is A. $500 B. $1,000 C. $4,000 D. $4,500 Correet answer is (D) Q: A lumber products company incurs $140,000 in costs to produce 100,000 board-feet of finished lumber that sells for $1/board-foot and 50,000 board-feet of plywood that sells for $0.50/board-foot ‘Gales value at split-off). Using the sales value at split-off method, what is the cost per board-foot for each joint product? A. $.0.92/board-foot finished lumber; $0 .48/board-foot plywood B. $1.12/board-foot finished lumber; $0.56/board-foot plywood C. $0.64/board-foot finished lumber; $0.32/board-foot plywood D. $0.82/board-foot finished lumber; $0 .44/board-foot plywood the correct answer is (B) Joint products | Q | P(1) | Total | Allocation| Common cost | Common cost sales % allocation per unit value Finished lumber | 100.000) $1 | $100,000 | 80% | $140,000 80%= $1.12 | $112,000 Plywood 30,000 | $0.50 | $25,000 20% $140,000 « 20%- 30.56 _| $28,000 Total = : $125,000 100% i $140,000 - 2 Q: A cheese company produces natural cheese from cow's milk. As a result of the process, a secondary product, whey, is produced in the proportion of one pound for each pound of ‘cheese. The data given based on the standards set for 1,000 pounds of cow's milk. Input: 1,000 pounds of cow's milk at $0.20 per pound 40 hours of labor at $10 per hour Manufacturing overhead is applied on a basis of 100% of direct labor cost Output: 450 pounds of cheese 450 pounds of whey ‘The following prices and demand are expected: Product Price per Pound Demand in Pounds Ce 450 - Whey 0.80 375 Given that the cheese company allocates common costs on the basis of net realizable values (toz), what are the allocated common costs per 1,000 pounds of cow's milk for cheese and whey would be? Answer: ‘Common Costs per 1,000 pounds of cow's milk = Direct materials DM. $200 Direct labor DL. $400 Overhead O.H. $400 Total $1,000 Joint product Quantity(that can be sold) % Joint cost allocation Cheese 450 Pound $2 = $900 75% | $1,000 75% =S750 Whey 375 Pound x $0.80 = $300 25% $1,000 x 25% =$ 250 Total $1,200 100% | $1,000 Q: Petrochemicals Inc. is a small company that acquires high-grade crude oil from low-volume. production wells owned by individuals and small partnerships. The crude oil is processed in a single refinery manufacturing process into Two Oil, Six Oil, and impure distillates. Petrochemicals does not have the technology or capacity to process these products further and sells most of its output each month to major refineries, there were no beginning inventories of finished goods or work- in-process on November 1,The production costs and output of Petrochemicals for November are as follows: Crude oil acquired and plac in production (D-M) $5,000,000 Direct labor and related costs 2,000,000 Factory overhead 3,000,000 Production and sales | Product Production (barrels) Sales (barrels) Price 1 Two - Oil 300,000 80,000 | 80 [ Six- Oil 240,000 120,000 ‘330 lates 120,000 120,000, $15 ‘What isthe portion of joint production costs assigned to each product based upon the relative sales value of output would be? Answer: Joint Product | Quantity Produced | % | Joint cost “Joint costs allocated to Salling price allocation ‘Unis aventory Units sold Two- oil "300,000 * $20 = 6,000,000 40% | $4,000,000 | 4,000,000 * 80/300 = 1,066,666 | 4,000,000 x 220¥300 = 2,933,334 Six -oil 240,000 * $30 = 7,200,000 | 48% | $4,800,000 | 4,800,000 « 120/240 = 2,400,000 | 4,800,000 « 120/240 = 2,400,000 Distillates 120,000 * $15 = 1,800,000 12% [$1,200,000 $1,200,000 Zero Total $15,000,000 100% | 10,000,000 4,666,606 $,333,334 Q: The assignment of raw material costs to the major end products resulting from refining a barrel of crude oil is best described as A. Indirect costing. B. Joint costing. C. Differential costing. _D. Incremental costing. Answer (A) is incorrect because indirect costing is a nonsense term. Direct costing charges products only with variable costs. Answer (B) is correet, Joint products are common products created from processing a single input (e.g., gasoline, diesel fuel, and kerosene), Joint produets have common costs until they ‘nthe split-off point. Joint costing assigns common costs to joint products. ‘Answer (C) is incorrect because differential costing is not a commonly used term, but it could mean costing common products at a fixed differential. Answer (D) is incorrect because Incremental costing is not a common term cither, but it could ‘mean costing in increments. 74 1una Mining Company Inc. (LMC) develops mineral deposits in Canada’s north, During 2012, the company removed 1,200,000 tons of ore from the mine; this level of production is expected to continue for the next twenty-five years, The process of extracting valuable minerals begins with the removal of ore from the ground and crushing it. The crushed ore is then ready to enter the refining process. The total cost of mining and crushing the ore amounts to $30.65 per ton. In order to extract the usable minerals, the crushed ore is passed through three distinet processes: amalgamation, electrolysis and purification (see Exhibit 1). In the first stage, called amalgamation, mercury is added to the unrefined ore. The mercury combined with the valuable minerals forms a compound which is drawn off. This compound is then heated causing the mercury to vaporize leaving a residue which is 30% gold, 60% silver, and 10% platinum. One ounce of residue is obtained for every 3 tons of ore placed in the process. The vaporized mercury is condensed and reused the next day. The total cost of the amalgamation process is $13.51 per ounce of residue recovered. Each year the company must invest an additional $280,000 to replace mercury lost during the recovery process. ‘The second process is called electrolysis. In this process, the residue from the amalgamation process is cast into thick slabs which are hung in large vats of acid. When an electric current is passed through the vat, pure gold is transferred to separate plates. When the gold plates and acid are removed from the vat, silver is recovered from the bottom of the vat. When the acid is filtered for reuse the next day, platinum is zemoved. The total cost of the electrolysis process is $9.60 per ounce of residue placed in the vat. At this point, the gold plates are 99.9% pure and can be sold on world markets. The current and anticipated market price for gold is $400 per ounce. During the past three months, the price has ranged from $350 to $420 per ounce. The silver can be sold after electrolysis for $4.60 per ounce or it can be further purified. Purification costs are $1.30 per ounce of pure silver, which can be sold for $6 per ounce (both the $6 and $4.60 prices have remained relatively stable during the past year and management expects no significant changes during the next year). This purification process reduces the volume of the silver product by 20%. The platinum can be sold after electrolysis for $430 per ounce or further purified and sold for $550 per ounce. Purification results in a 10% reduction in the volume of platinum. The total cost of purification is $48.60 per ounce of pure platinum Luna Mining Company Ine. Summary of Mineral Processing PROG INPUT OUTPUT PROCESS | MARKET PRICE OF cost OUTPUT 1. Amalgamation | 3 tons ore Toz. residue $13.5 1/02. oF NA + + residue + Mercury Mereury $280,000/year for Mereury N/A 2. Electrolysis Toz. | 0.30 0z. pure gold $9,60/0z. of | $400/c2. pure gold residue | 0.60 oz. raw silver input | $4.60/0z. raw silver 0.10 02. raw platinum $430/oz. raw platinum 3. Purification Toz.raw | 0.80 oz. pure silver 5130/07. oF $6/0z. of pure silver silver output 4. Purification Toz. raw | 0.90 02. pure platinum | $48.60/0z. of {$550 Joz. of pure platinum | output platinum Required: 1. Determine the optimal yearly production plan for the ore processed by LMC Ine. 2. Allocate joint cost among joint products 15 Answer: 1. All costs incurred up to the end of the electrolysis process are joint costs. Only after the electrolysis process is completed separate and identified products recovered. Therefore, the cost of mining and crushing the raw ore, the cost of amalgamation including the yearly expenditure on mercury, and the cost of the electrolysis process itself are all joint costs. Joint costs = Mining and crushing ($30.65%1,200,000 Tons) $36,780,000 + amalgamation ( 1,200,000.Tons 3.Ton Electrolysis ($9.60 400,000 Ounce) $3,840,000 $13.51 per ounce) $5,404,000 + yearly expenditure on mercury $280,000 + Joint costs = $46,304,000 The primary concer is whether silver and platinum be sold “as is” or processed further. ‘YAIl costs incurred up to and including the electrolysis stage are sunk cost and therefore irrelevant. ‘vQuantity of gold/silver/platinum recovered in 2012 = he ‘OnS — 400,000 ounce ‘ons Description Silver Platinum Ounces recovered after electrolysis (60% and 10%) | 240,000 40,000 Selling price as “impure” (per ounce) $4.60 $430 Revenue at Spite off point (selling impure) $1,105,000 | $17,200,000 Selling price as “pure” (per ounce) $6 $350 Cost of purifying (per ounce of output) $1.30 $48.60 Net revenue of selling pure 34.70 $501.40 Ounices recovered of “pure” product (80% and 90%) 192,000 36,000 ‘Revenue from selling pure $902,400 _| $18,050,400 _ ‘Recommendation [Sell impure |__ Purify 2. Joint cost allocation Description | _ Revenve at spite off % C.C allocation | Separable Final cost Revenue Gold 120,000 Ounce = $400 72.40% $33,524,096 None, $48,000,000 = $48,000,000 Silver 240,000 Ounce = $4.60 1.66% $768,646 ‘None $1,104,000 |__“=$1,104,000 | Platinum | 40,000 Ounce x $430 25.94% $12,011,258 | $1,749,600 | $19,800,000 = $17,200,000 Total 366,304,000 100% ~ $46,304,000 | $1,749,600 | $68,904,000 See Gleim MCQ # 349, 354, 379, 401, 511, 513, 514, 515, 516,517, 518, 522, 526, 527, 528, @) Net realizable value (NRV) ‘This method is applied only if one or more joint products ean‘t be sold immediately after split- off. ‘This method is used to allocate joint costs if no selling price is given at split-off point for any of the joint products or if one or more of joint products must technically further processed to be sold. Q: Sawmill manufactures two lumber products from a joint milling process. The two products developed are ‘mine support braces (MSB) and unseasoned commercial building lumber (CBL). A standard produetion run incurs joint costs of $300,000 and results in 60,000 units of MSB and 90,000 units of CBL. Each MSB sells for $2 per unit, Assume the commercial building lumber is not marketable at split-off but must be further planed and sized at a cost of $200,000 per production run. During this process, 10,000 units are unavoidably lost; these spoiled units have no discernible value. The remaining units of commercial building lumber are salable at $10 per unit. The mine support braces, although salable immediately at the split-off point, are coated with a tar-like preservative that costs $100,000 per production run. The braces are then sold for $5 each, Using the net realizable value (NRV) basis, what is the completed cost assigned to each unit of both products? Answer: Joint | NRV=(final selling | Joint cost ~~ Total Per unit average product price final Q)- % allocation production total costs (separable costs) costs MSB (60 x $5) - $100= 300 x 25%= 75+ 100= 175 $200 25% 5 175 oO _ {| _4 =$2.917 CBL (80 = $10) - $200 = 75% 300 = 75% = 225 + 200= 425 $600 225 425 30 =$5.3125 ‘otal $800 100% 300 600 Q: Assume the company chooses not to process MSB beyond split-off point what is opportunity loss? ‘¥The opportunity loss on the company's C.M = Incremental revenue - Incremental costs Opportunity loss = 60,000 units x ($5- $2) ~ [$100,000] = $80,000 Q: Assume the final selling price of MSB $3.5 what is the optimal decision and what is the amount of allocated common cost to each product? Answer: then the company would not further process MSB because Incremental costs would exceed Incremental revenue by $10,000 ‘yThe allocation of common cost based on net realizable value (NRV) basis would be as follows 1 Joint ] % T Joint cost Total | Per unit average product allocation production total costs costs | MsB | (60x 82)=si20 [16.67% 300 x 16.67% 50 50 = 50,000 0 - | = $0.833 | CAL { (80 * $10) - $200= [e5s9% 300% 83.33% | 2504200 450 $600 = 250,000 = 450 0 = $5,625, Total $720,000 100% 300,000 300,000 - uucariz Company processes Duo into two joint products, Big and Mini. Duo is purchased in 1,000 gallon drums for $2,000. Processing costs are $3,000 to provess the 1,000 gallons of Duo into 800 gallons of Big and 200 gallons of Mini. The selling price is $9 per gallon for Big and $4 per gallon for Mini. Big can be processed further into 600 gallons of Giant if $1,000 if additional processing costs incurred. Giant can be Sold for $17 per gallon, [f the net-reatizable-value method were used to allocate costs to the joint products, the total cost of producing Giant would be B. $5,564 A. $5,600 . 35,520 YNet benefit of further processing is calculated as follows: D. $4,600 Incremental revenue = (final revenue 600G $17) (Revenue at split off 800G * $9) = $3,000 Incremental cost = $1,000 Net benefit of further processing = $3,000 - $1,000 = $2,000 Note: The appropriate method to use is relative sales value at split off because both products can be sold at this point at a determinable sales price, but the solution would be based on NRV because the question is based on NRV method. Answer (A) is correct. Common cost is D.M $2,000 + $3,000 C.C = $5,000 Des Qat | Pper [Additional] Qafter | P perG’ NRV | % Cc split | G(I) | processing | further | (2) off cost | processing allocation Big | 800G] $9 | $1,000 | 600Gor | S17 | (00x | 92% | $4,600 Giant | $17)- 1,000 = | $9,200 [Mini ) 2006734 ~ - - 200x$4 | 8% $400 = $800 Total | 1,000 $10,000 | 100% | $5,000 Total cost of producing Giant = Common cost $4,600 + Separable cost $1,000= $5,600 B : Tempo Company produces three products from a joint process. The three products are sold after further ‘processing as there is no market for any of the products at the split-off point. Joint costs per batch are $315,000. Other product information is shown below. Deseription Product A | Product B | Product C Units produced per batch 20,000 30,000 50,000 Further processing costs per unit $070 3B 32 Final sales value per unit 3 % If Tempo uses the net realizable value method of allocating joint costs, how much of the joint costs willl be allocated to each unit of Product C? $2.10 B. $2.65 C.83.15 D.$3.78 Answer (D) is correct. Joint produet Q | Final sales | Total sales | Separable| Total | NRV % value per , value cost per Separable unit unit cost Product A 20,000 35 $100,000 ] $0.70 $14,000 | $86,000 19.55% Product B 30,000 36 $180,000 | $3 $90,000 | $90,000 | 20.45% | Product C 30,000 7 $350,000 | $1.72, $86,000 | $264,000 | 60% Total $630,000 ‘$190,000 | $440,000 | 100% ‘Toint product Total joint cost allocation Per unit common cost Product A $315,000 *19.55%= § 61,582 $61, 582=20,000 = $3.08 Product B $315,000 *20.45%= 865,418 ‘$64,418=30,000= $2.15 Product C ‘$315,000 *60%=$189,000 $189,000-> 50,000 units = $3.78 | Total $315,000 Q: Fitzpatrick Corporation uses a joint manufacturing process in the production of two products, Gummo and Xylo. Each batch in the joint manufacturing process yields 5,000 pounds of an intermediate material, Valdene, at a cost of $20,000. Each batch of Gummo uses 60% of the Valdene an icurs $10,000 of separate costs. The resulting 3,000 pounds of Gummo sells for $10 per pound. The remaining, Valdene is used in the production of Xylo which incurs $12,000 of separable costs per batch. Each batch of Xylo yields 2,000 pounds and sells for $12 per pound. Fitzpatrick uses the net realizable value method to allocate the joint material costs. The company is debating whether or not to process Xylo further into a new product, Zinten, which would incur an additional $4,000 in costs and sell for $15 per pound. If Zinten is produced, income would inerease by A. $2,000 Answer (A) is correet. B. $5,760 C. $14,000 9 D. $26,000 | ‘Common Common | Joint Q | Separable {Po T Further cost PQ | process cost | products cost I I 5,000 $20,000 | Gummo | 3,000 | $10,000 gio - Valdene Kylo | 2,000) $12,000 F312 ‘$4,000 35] Incremental revenue to further process Xylo =2,000 pounds * ($15-$12) = $6,000 Incremental cost to further process Xylo = $4,000 Incremental profit that results from further process of Xylo = $6,000 -$4,000 = $2,000 Joint products Q NRV T"% | Common cost allocation I Gummo 173,000 T (3,000 x $10)- si0,000= | 62.5% ‘$20,000 * 62.5% = _ $20,000 $12,500 { Xylo 1 2,000 (2,000 * $12) - $12,000 37.5% | $20,000 * 37.5% = $12,000 $7,500 | $32,000 | 100% ‘$20,000 Fact pattern: A manufacturing company uses a joint production process. Joint production costs during April were ‘$720,000. The company uses the sales method for allocating joint costs. Product information for April was as, follows: | Description Products | R 3 T Units produced 2500 5,000 7,500 I Units sold 2,000 4,000 7,000 Sales prices: At the split-off $100 [ $80. $20 Sales prices Afier further processing $150 Sus __ $30 Costs to process alter split-off $150,000 $150,000 $100,000 QU: Assume that all three products are main products and that, they can be soldat split-off point or processed further whichever is economically beneficial to the company: What product ac products should be processed further? Answer In order to optionally process the product further the Incremental revenue must exceeds incremental costs 80 Joint product Q PB P2 | Incremental | Incremental | Further produced revenue costs processing R 2,500 | $100 | $150 | $125,000 | $150,000 NO s 5,000 | $80 $115 | $175,000 | $150,000 YES T 7,500 | $20 $30 $75,000 | $100,000 o Joint cost allocations should be based on relative sales value at split-off why? Because all three produets can be sold at split-off point, and their sales prices are available at split-off point. Joint products | Q Produced * Sales price at Joint cost allocation based on relative split off point sales prices at split-off point R 2,500 $100 = $250,000 720,00» 2.578 = $225,000 s 5,000 * $80= $400,000 $720,000 * 4/8 = $360,000 T 7,500 ¥ $20= $150,000 $720,000 * 1.5/8 = $135,000 Total $800,000 5720000 Note: Total costs allocated to product (S) that is further processed ~ $360,000 joint costs + $ 150,000 separable costs = $510,000 Q2: Assume that product (T) is treated as by-product and its NRV used to reduce joint costs (inventoried) assume also that products (S) and (T) must be further processed before they can be sold, Calculate NRV of by product and allocate common cost to joint products Answer YNRV of by-product (T) that should be further processed [(7,500 units x $30) — ($100,000)] = $125,000 ¥Thus allocable joint costs = $720,000 -$125,000 = $595,000 YNRV method of joint cost allocation must be used in this case because product (S) must be further processed. Joint product NRV % Joint costs allocation R 2,500 = $100 =250,000 > Need not to |37.04%| 595,000 = 37.04% = $220, 388 be further processed 81 [ 8 [6,000 «$115) — ($150,000)] = 425,000] 62.96% | 595,000 x 62.96% = $374,612 Total ‘$675,000 100% '3595,000 ‘company processes a raw material into products F} -F, and F Each ton of aw material produces five ‘units of Fi, 1690 units of Fa and three units of F5, Joint-processing costs to the split-off point are $15 per ton assuming further processing results in the following per unit figures what is the portion of joint costs allocated to each product based on NRV? Description | Products Fy Fy Fs Additional per unit processing costs per unit $28 $30 $25 Selling price per unit $30 $35 $35 Answer | oint product | Units produced | Perunit NRV | Total NRV % | atocateacc_| i s $30-828=S2 | Sunits«s2=si0 | 20% | sis*20%=$3 | ry 2 $35-$30=$5 | 2unitsxs5=$10 20% | $15 * 20%= $3 Fy 3 $35 -$25=$10 | 3 units x $10 = $30 60% S15 * 60%=$9 Total S50 100% $15 Q: Copeland Inc. produces LOLO in a joint manufacturing process. The company is studying whether to sell LOLO at the split-off point or upgrade the product to become SOSO. The following information has been gathered. 1. Selling price per pound of LOLO IL. Variable manufacturing costs of upgrade process Jl, Avoidable fixed costs of upgrade process IY. Selling price per pound of SOSO V. Joint manufacturing costs to produce LOLO Which items should be reviewed when making the upgrade decision? A.A Il, and TV B.A, UL and IV C.Allitems Dil, 11, 1V, and V Answer (A), (C) and (D) are incorrect because the joint manufacturing costs are the only irrelevant item. Answer (B) is correct. Common, or joint, costs can't be identified with a particular joint product. By definition, joint products have common costs until the split-off point. Costs incurted after the split-off point are separable costs. The decision to continue processing beyond split-offis made separately for each product. The costs relevant to the decision are the separable casts because they can be avoided by selling at the split-off point. 82 They should be compared with the incremental revenues from processing further. Thus, items I (revenue from selling at splt-off point), II (variable costs of upgrade), III (avoidable fixed costs of upgrade), and IV (revenue fiom selling after further processing) are considered in making the upgrade decision. Q: In joint-product costing and analysis, which one of the following costs is relevant when deciding the point at which a product should be sold to maximize profits? A. Separable costs after the split-off point B. Joint costs to the split-off point G les salaries for the period when the units were produced D. Purchase costs of the materials required for the joint products Answer (A) is correct. Joint products are created from processing a common input. Common costs are incurred prior tothe split-off point and cannot be identified with a particular joint product. As. result, common costs are irrelevant to the timing of sale. However, separable costs incurred after the split-off point are relevant because, if incremental revenues exceed the separable costs, products should be processed further, not, sold at the split-off point. Answer (B) is incorrect because joint costs (common costs) have no effect on the decision as to when to sell a product Answer (C) is incorrect because sales salaries for the production period do not affect the decision. Answer (D) is incorrect because purchase costs of the materials required for the joint products are joint costs. Essay: Princess Corporation grows, processes, packages, and sells three apple products- sliced apples that are used in frozen pies, applesauce, and apple juice, The outside skin of the apple, which is removed in the Cutting Department and processed as animal feed, is treated as a by product. Princess uses the net realizable relative sales value method to assign costs of the joint process to its main products. The by-product is inventoried at its market value; Details of Princess” production process are presented below. + The apples are washed and the outside skin is removed in the Cutting Department. The apples are then cored and trimmed for slicing. The three main products and the by-product are recognizable after processing in the Cutting Department. Each product is then transferred to a separate department for final processing. + The trimmed apples are forwarded to the Slicing Department where they are sliced and frozen. Any juice generated during the slicing operation is frozen with the slices. + The pieces of apple trimmed from the fruit are processed into applesauce in the Crushing Department. Again, the juice generated during this operation is used in the applesauce. «The core and any surplus apple generated from the Cutting Department are pulverized into @ liquid in the Juicing Department. There is a loss equal to eight percent of the weight of the good output produced in this department. + The outside skin is chopped into animal feed and packaged in the Feed Department. ‘A total of 270,000 pounds of apples were entered into the Cutting Department during November. The schedule presented below shows the costs incurred in each department, the proportion by weight transferred to the four final processing departments, and the selling price of each end product. f Processing Data and Costs November 2010 | ‘Department Costs y | Selling Price per Incurred | Weight Transferred to | Pound of Final | Departments Product | Cutting $62,000 | ‘None None | Slicing | 11,280! 33% $0.80 | | ‘Crushing 18,550 30% 0.55 Juicing 3,000 | 27% 0.40 84 Feed 700 10% 0.10 ] Total 83,530 100% Required ‘A. Princess Corporation uses the net realizable value method to determine inventory values for its main products and by-products. For the month of November 2010, calculate the 1) Resulting output for apple slices, applesauce, apple juice, and animal feed, in pounds. 2) Net realizable value for each of the three main products. 3) Amount of the cost of the Cutting Department assigned to each of the three main products in accordance with corporate policy 4) Gross margins in dollars for each of the three main products. B. Comment on the significance to management of the gross margin dollar information by main product for planning and control purposes, as opposed to inventory valuation, Solution: A. 1, For the month of November, 2010, Princess Corporations resulting pounds of apple slices, applesauce, apple juice, and animal feed were as follows: Princess Corporation Product Output in Pounds November 2010 Product Inputs | Proportion | Total Pounds | Net Pounds Pounds Lost Slices 270,000 33% 89,100 | 89,100 Sauce 270,000 30% | 81,000 81,000 Juice 270,000 27% 72,900 3,400 | 67,500 (1) Feed 270,000 10% 27,000 27,000 Total 270,000 100% 770,000 3,400 264,600 (1) 72,900 + 1.08 = 67,500 2, The net realizable value for each of the three main products is calculated below. Product Calculation of net Realizable Value at the Split-off Point Net Pounds | Price Selling | Separable NRV Revenue Costs 85 Slices 89,100) $0.80] $71.280 | 11,280 | $60,000 [ Sauce 81,000 055 44,550 3550 36,000 — Juice 67,500 0.40 27,006, 3,000 24,000 a [ Tos tCi«S 1830 | 22a | 120.000 : 3. The net realizable value of the by-product is deducted from the production costs prior to allocation to the main products, as presented below. Net realizable value (NRV) of by-produet = By-product revenue -Separable costs ($0.10 x 270,000 pound) - $700 = $2,700 - $700 = 2,000 Cutting Department Costs to be allocated To Main products = Joint cost - NRV of by- product = $62,000 - $2,000 = $60,000 4, The gross margin in dollars for each of Princess Corporations three main products is as follows: Princess Corporation Gross Margin in Dollars November 2010 Product Revenue Separable Cost | Joint Gross | Gross Margin Slices ~~ $71,280 11,280 $30,000 $30,000 Sauce 44,550 8,550 18,000 18,000 Juice 27,000 3,000 12,000 12,000 Total 142.830 22,830 60,000 60,000 B. The gross margin dollar information by main product is determined by the arbitrary allocation of joint production costs. As a result, these cost figures and the resulting gross margin information are of little significance for planning and control purposes. The allocation is made only for purposes of inventory valuation and income determination. (4) Gross profit method Q: A petroleum products company produces plastic pellets and nylon thread from the same process. For one batch, the products share $9,000 in joint costs and, after the split-off point, pellets have $2,000 in additional processing costs while nylon thread has $5,000 in additional processing costs. A batch produces 2,000 cans of pellets and 1,000 spools of thread. Plastic pellets sell for $4.50/can, and nylon thread sells for $1 1/spool. Using the gross profit method, allocate common cost to each product. Answer: 86 14.50% 2,000 + $11 1,000 = $20,000 C.C $9,000 + Separable cost $2,000 + $5,000 = $16,000 16,000 3.cocs % =5 = 80% $20,000 4. Based on COGS % calculate separate COGS figure for each product 5. Deduct separable cost from COGS figure to get each product share from common cost Joint product |_Q_ | Price | Total revenue | COGS 80% | Separable costs | Common cost Plastic pellets [2,000 $4.50 $9,000 $7,200 $2,000 $5,200 Nylon thread | 1,000 [$11 $11,000 $8,800 $5,000 $3,800 Total {__ $20,000 $16,000 $7,000 39,000 Disadvantages of Gross profit method Assume common cost is $5,000 and the following data Joint product Q produced Price | Total revenue Separable costs A 800 10 $8,000 5,200 r B 1,500 8 12,000 | 7,000 ic 2,000 3 10,000 9,800 530,000 | 22,000 2, COGS % = 822,000 + 85,000 _ ogo, $30,000 Joint product Total Total COGS 90% Separable costs Joint cost revenue _| A $8,000 $7,200 5,200 2,000 B 12,000 10,800 7,000 3,800 c 10,000 9,000 9,800 (800) | 30,000 27,000 22,000 5,000 ‘vAccordingly a part of the main product C separable cost are allocated as common cost to the other products, hence this method could result in negative common cost allocation to some of the main products (those products that has a very high Separable costs % relative to products revenue, the produet C has Separable costs % relative to its revenue = 98% which is higher than general COGS % that was only 90%) Q: A company makes two joint products from recycled plastic: artificial joists for construction and plastic designer chairs. The total joint costs are $140,000, and the process produces 3,000 joists and 2,000 chairs. Joists sell for $20 and chairs for $50. The average cost per unit is determined to be $28, resulting in a gross margin of -$24,000 for joists and $44,000 for chairs Which of the following joint cost allocation methods was used for these calculations? 87 A. Gross profit method B. Net realizable value method C. Sales value at split-off method D. Physical measure method Answer (D) is correct: Average cost per unit is $28, thus total cost = $28 * 5,000 units = $140,000 Accordingly Physical volume is used to allocate joint costs Joint product [_Q | % CC allocation Price [Total revenue | Gross P/L Artificial joists | 3,000 | 60% | $140,000 x 60% = 84,000 | $20 $60,000 (24,000) plastic chairs | 2,000 | 40% $140,000 « 40% = 56,000 $50 $100,000 44,000 Total 5,000 | 100% $140,000 | s160,000 $160,000 The resulting skewed gross margins that make the joists seem Very unprofitable are the primary problem with using the physical measure method, (5) Gross market value method Q: Atlas Foods produces the following three supplemental food products simultaneously through a refining process costing $93,000. The joint products, Alpha and Beta, have a final selling price of $4 per pound and $10 per pound, respectively; additional processing costs of $2 per pound of each product are incurred after the split-off point. Morefeed, a by-product, i sold at the split-off point for $3 per pound Product ‘Quantity -pounds Notes Alpha 10,000 Caloric value 4,400 calories per pound Beta 5,000 Caloric value 11,200 calories per pound | More-feed 1,000 Used as a cattle feed supplement Caloric value 1,000 | calories per pound Assuming the company inventories the by-product, and it incurs no additional processing costs, what isthe joint costto be allocated to each joint product using the gross market value method? (Products Gri warket value __| Allocation of joint costs __] Alpha 10,000 pounds x $4 = $40,000 90,000 *_4/9 = $ 40,000 [Beta 5,000 ponds * $10 = $50,000 90,000 * 5/9 = § 50,000. Total $90,000 $90,000 (Other Basis From previous example Assume Atlas Foods inventories more- feed, the by-product, what is the joint cost to be allocated to main products using the weighted-quantity method based on calorie value per pound? Joint product Products | Calorie Value % ‘Allocation of joint costs based | on caloric value { 88 ‘Alpha 10* 4.4=44 44% | $90,000 * 44% = $39,600 Beta 5*112=56 56% $90,000 * 56% = $50,400 Total 100 100% $90,000 Q: Grapevine Corporation produces two joint products, JP-1 and JP-2, and a single by- product, BP-1, in Department 2 of its manufacturing plant. JP-1 is subsequently transferred to Department 3 where it is refined into a more expensive, higher-priced product, JP-IR, and a by-product known as BP-2. Recently, Santa Fe Company introduced a product that would compete directly with JP-IR and, as a result, Grapevine must re-evaluate its decision to process JP-1 further. The market for JP-1 will not be affected by Santa Fe’s product, and Grapevine plans to continue production of JP-1, even if further processing is terminated. Should this latter action be necessary, Department 3 will be dismantled. Which of the following items should Grapevine consider in its decision to continue or terminate Department 3 operations? 1. The selling price per pound of JP-1 2. The total hourly direct labor cost in Department 3 3. Unit marketing and packaging costs for BP-2. 4. Supervisory salaries of Department 3 personnel who will be transferred elsewhere in the plant, if processing is terminated. 5. Department 2 joint cost allocated to JP-1 and transferred to Department 3. 6. The cost of existing JP-IR inventory. A.2,3,4 Bi1,2,3 C.2,3,5,6 D.1,2,3,4,5 Answer (B) is correct: Q: Which two criteria differentiate relevant costs and revenues from irrelevant data? A. Streamlined activities and incremental gains B. Qualitative and quantitative dec C. Historical basis and the given decision situation D, Future orientation and different alternative choices Answer (D) is correct: Q: Costs are allocated to cost objects in many ways and for many reasons. Which one of the following is a purpose of cost allocation? A. Evaluating revenue center performance. B. Measuring income and assets for external reporting, C. Budgeting cash and controlling expenditures D. Aiding in variable costing for internal reporting. Answer (A) is incorrect because a revenue center is evaluated on the basis of revenue generated, without regard to allocated costs. Answer (B) is correct. Cost allocation is the process of assigning and reassigning costs to cost objects. It is used for those costs that can't be directly associated with a specific cost object. Cost allocation is often used for purposes of measuring income and assets for external reporting purposes. Cost allocation is less meaningful for internal purposes because 89 responsibility accounting systems emphasize controllability, a process often ignored in cost allocation. Answer (C) is incorrect because cost allocation is not necessary for cash budgeting and controlling expenditures. Answer (D) is incorrect because allocations are not needed for variable costing, which concems direct, not indirect, costs. See Gleim MCQ # 348, 512,519, 520, 521, 523, 524, 525, 90 Allocating Service Department costs to production departments ‘ase Study: A Company uses process costing has two service departments, Power and Maintenance, and two production departments, Machining and Assembly. The following factory O.H costs and other information budgeted for the following year: Service Departments Production Departments Description Power | Maintenance | Machining | Assembly | Total Budgeted factory 0.11 costs (Direct) | $62,500 | $40,000 $25,000 $15,000 | $142,500 Budgeted activity- Kilowatt hrs 10,000 50,000 150,000 50,000 260,000 Budgeted activity -Maintenance hours | 250 100 1125 1,125 2,600 ‘YThe amounts $62,500, $40,000, $25,000 and $15,000 represents: A) Budgeted annual factory O11 at department level, each department considered as a separate cost pool. B) It is considered Direct factory O.H1 costs, when the cost object is each individual department (whether production- service department or production department) thus The $62,500 O.H. isa direet cost of power department and $40,000 O.H. is a direct cost of maintenance department and so on. ©) Tot $142,500 D) These total factory O.H. amounts include part fixed and part variable. E) It is preferable to segregate the Budgeted fixed factory O.H from Budgeted variable factory 0.1 factory O.H costs expected to be incurred in the upcoming year (Budgeted O.H costs) to best allocate each component of factory O.H separately to production departments. Q: When allocating service department costs to production departments, the method that does not consider different cost behavior patterns is the A. Step method. B. Reciprocal method. C. Direct method. D. Single-rate method. Answer (A) is incorrect because the step method can be used on a single- or dual-rate basis. Answer (B) is incorrect because the reciprocal method can be used on a single- or duat-rate basis. Answer (C) is incorrect because the direct method can be used on a single- or dual-rate basis. Answer {D) is correct. The single-rate method combines fixed and variable costs. However, dual rates are preferable because they allow variable costs to be allocated on a different basis from fixed costs. F) Budgeted departmental O.H cost depends on the expected level of activity driver (i.c.) 260,000 Kilowatt hours (cost driver) would result in incurrence of $62,500 factory O.H costs in the power department, and 2,600 maintenance hours (cost driver) would result in ineurrenee of $40,000 factory O.H in the maintenance department. G) Both power department and maintenance department are considered production- service departments because they provide services to production departments (machining and assembly) Hi) The budgeted factory O.H. of service departments are indireet costs, when the cost objectives are production departments, 1) The service departments factory 0.4L is a necessary costs for production activity and accordingly these costs are not period costs but rather it considered an indirect product costs (inventoriable) that must be allocated to production departments as an intermediate cost objectives. Then and only then all manufacturing O.H are pooled (accumulated) in the production departments as an intermediate cost objectives. J) Using an appropriate cost driver for each production department the total budgeted manufacturing O.H. accumulated in each production department and the appropriate cost driver of that production department are combined to determine predetermined departmental single factory O.H. application rate. Predetermined single departmental factory O.H. application rate = Pooled.annual.budgeted.departmental. factory.O.H.cost. Annual budgeted activity.of appropriate.cost.driver. for each productiondepariment K) Allocation of service department costs to production departments is necessary to determine predetermined departmental factory O.H. rates. L) This budgeted factory O.H. rate is applied to WIP using either R * SI > AO (under standard costing system) or R * AI > AO (called normal costing system) M) Thus WIP would be charged by Tactory ©. costs incurred in both service departments and production departments. The cost objective in this case would be the ultimate product (or service) Q: Allocation of service department costs to the production departments is necessary to A. Control costs. B, Coordinate production activity. C. Determine overhead rates. D. Maximize efficiency. Answer (A) is incorrect because costs can be controlled by the service departments without allocation. However, allocation encourages cost control by the production departments. If the costs are allocated, ‘managers have an incentive not to use services indisetiminately. Answer (B) is incorrect because allocation does not affect the coordination of production activity, Answer (C) is correet. Service department costs are indirect costs allocated to production departments to better determine overhead rates when the measurement of full (absorption) costs is desired. Overhead should be charged to production on some equitable basis to provide information useful for such purposes as allocation of resources, pricing, measurement of profits, and cost reimbursement. Answer (D) is incorrect because allocation of costs has no effect on the efficiency of the provision of services when the department that receives the allocation has no control over the costs being controlled ay N) Cost objective may be intermediate (production Department) or final (the units of output) and in all cases cost objective should be logically linked with the cost pool through an appropriate cost driver preferably chosen based on a cause and effect basis, Cost Accumulation I Cost Allocation ‘Through Cost pools (1) Cost driver ‘0 Cost objectives (2) Cost driver is the link that Budgeted O.H is accumulated by transfers an indirect cost from cost | cost objective (target) can Each service and production pool to cost objective be Intermediate or final | departments (each represents a 3999 € EEE separate cost pool) Cost driver is the allocation base called common denominator | Cost driver homogeneous cost Wy 999999999 Chosen based on cause and effect N) When allocating (distributing) service department costs to production departments the appropriate criteria upon which to choose the appropriate cost driver are: 1. Cause and effect criterion, it is considered the most appropriate criterion. 2. Benefits received (formula is used when no appropriate causality is found) Note: Whatever the criterion chosen the allocation should be fair; Fairness is an objective not criterion, Q: Cost allocation is the process of assigning indirect costs to a cost object. The indirect costs are ‘grouped in cost pools and then allocated by a common allocation base to the cost object. The base that is employed to allocate a homogeneous cost pool should ‘A, Have a cause-and-effect relationship with the cost items in the cost pool. B. Assign the costs in the pool uniformly to cost objects even if the cost objects use resources in a non uniform way. C. Be a nonfinancial measure (e.g., number of setups) because a nonfinancial measure is more objective. D. Have a high correlation with the cost items in the cost pool as the sole criterion for selection. Answer (A) is correct. A cost allocation base is the common denominator for systematically correlating indirect costs and a cost object. The cost driver of the indirect costs is ordinarily the allocation base. In a homogeneous cost pool, all costs should have the same or a similar cause-and- effect relationship with the cost allocation base. ‘Answer (B) is incorrect because if an allocation base uniformly assigns costs to cost objects when the cost objects use resources in a non uniform way, the base is smoothing or spreading the costs. ‘Smoothing can result in under costing or over costing of products, with adverse effects on product pricing, cost management and control, and decision making. ar ‘Answer (C) is incorrect because financial measures (e.g., sales dollars and direct labor costs) and nonfinancial measures (¢.g., setups and units shipped) can be used as allocation bases. Answer (D) is incorrect because high correlation between the cost items in a pool and the allocation base does not necessarily mean that a cause-and-effect relationship exists. Two variables may move together without such a relationship. The perceived relationship between the cost driver (allocation base) and the indirect costs should have economic plausibility and high correlation. Q: Boston Furniture Company manufactures several steel products. It has three production departments, Fabricating, Assembly, and Finishing, The service departments include Maintenance, Material Handling, and Designing. Currently, the company does not allovate service department costs to the production departments. John Baker, who has recently joined the company as the new cost accountant, believes that service department rates should be developed and charged to the production departments for services requested. If the company adopts this new policy, the production department managers would be least likely to A. Request an excessive amount of service. B. Replace watdated and inefficient systems. C. Refrain from using unnecessary services. D. Be encouraged to control costs. Answer (A) is correct. Q: The most important criterion to ensure an accurate cost allocation is A. Using a simple allocation method. B. Allocating fixed and variable costs by using the same allocation base. C. Using homogeneous cost pools. D. Using multiple drivers for each cost pool. Answer (C) is correct, Q In allocating factory service department costs to producing departments, which one of the following items would most likely be used as an activity base? A. Units of product sold. B. Salary of service department employees. C. Units of electric power consumed. D. Direct materials usage. Answer (A) is incorrect because making allocations on the basis of units sold may not meet the cause- and-effect criterion. Answer (B) is incorrect because the salary of service department employees is a part of the cost allocated, not a basis of allocation. Answer (C) is correct. Service department costs are considered part of factory overhead and should be allocated to the production departments that use the services. A basis reflecting cause and effect should be used to allocate service department costs. For example, the number of kilowatt hours used by each producing department is probably the best allocation base for electricity costs. Answer (D) is incorrect because making allocations on the basis of materials usage may not meet the cause-and-effect criterion. ‘Methods of allocating service department costs to the production departments (A) Direct Method ‘The direct method does not make allocations to other service departments but rather, it allocates each service department factory O.H costs directly to each production department hence ditect method is, the least aceurate method of indirect factory O.H cost allocation methods. The direct method is the simplest method and does not recognize services provided by one service department to another. aE Service Department Production Department Deseription Power | Maintenance | Machining | Assembly | ‘Total Budgeted Direct factory | $62,500 $40,000 $25,000 $15,000 | $142,500 O.H costs Allocating of power costs | ($62,500) $46,875 $15,625 i) based on Kilowatt hrs (3:1) ‘Allocating of maintenance 7 ($40,000) $20,000 | $20,000 | so | costs based on Maintenance hours (1:1) Total a) 0 $91,875 | $50,625 | $142,500 (2) Step-down method (sequential method) The step down method is a sequential process method that allocates service department factory O.HL costs to service departments as well as production departments. However, once a department's costs have been allocated, no additional allocations are made back to that department hence it is called step-down method, Q: Which service department would we start with, to allocate its accumulated factory O.H costs to other service as well as production departments? Answer: Start with service department that provides: 1) Highest service (%) to other service departments 2) Highest service ($) to other service departments 3) Provides services to the Highest number of other service departments ‘Based on first criterion we ean see that: A) Power department provides 50,000 Kilowatt hrs to Maintenance department out of total 250,000 Kilowatt hrs thus the % of services provided by Power department to Maintenance department —50,000.kilowatt hrs 250,000 .kilowatt hrs = 20% B) Maintenance department provides 250 Maintenance hrs to Power department out of total 2,500 Maintenance hrs thus the % of services provided by Maintenance department to Power department __250.maint enancelhrs = 10% 2,500.maint enance hrs Thus the allocat n should be started with Power department. Service Department | Production Department Description Power | Maintenance | Machining | Assembly | Total Budgeted OH eosts $62,500 $40,000 $25,000 | $15,000 | $142,500 r Allocating of power costs based | ($62,500) | $12,500 337,500 | $12,500 | $0 on Kilowatt iss (1:3:1) ‘Allocating of maintenance costs based on Maintenance hours (1:1) ($52,500) $26,250 $26,250 30 Total 30 30 $88,750 | $53,750 [$142,500 | ‘The step down method recognizes service provided by one service department to another but does not recognize reciprocal interdepartmental service. Q: The step-down method of service department cost allocation often begins with allocation of the costs of the service department that A. Provides the greatest percentage of its services to the production departments, B. Provides the greatest percentage of its services to other service departments. C. Provides the greatest total output of services D. Has the highest total costs among the service departments. Answer (A) is incorrect because the step-down method may start with the service department that renders the highest % of its total services to other service (not production) departments. Answer (B) is correct. The step-down method may start with the department that renders the highest % of its total services to other service departments. It then progresses in descending order to the service department rendering the least % of its services to the other service departments. An alternative is to begin with the department that renders the highest dollar value of services to other service departments. A third possibility is to begin with the department that renders service to the greatest number of other service departments. Answer (C) is incorrect because beginning with the service department with the greatest output is not customary because output for each service department is not the same. Answer (D) is incorrect because beginning with the service department with the highest costs is not customary 3) Reciprocal method (simultaneous equations method) The reciprocal method uses simultaneous equations to recognize mutual services (recognize reciprocal interdepartmental service) this method is the most complex but also the most accurate and sophisticated method. Application of Reciprocal Method 1- Total cost of power Department > P 2- Total cost of maintenance Department >M. P=$62,500+ 10% M (1) M = $40,000 + 20% P Q) P = $62,500 + 10% [840,000 + 20% P] a P= $62,500 + $4,000 + .02 P P=$67,857 _by plugging the amount of P to equation (2) M=$40,000 + 20% * $67,857 M=$53,571 i srvice Department Production Department Description | Power | Maintenance | Machining | Assembly | Total | Budgeted Direct factory [~ $62,500) $40,000 | $25,000 ‘$15,000 | $142,500 OH costs Allocating of power costs (867,857) 13,571 $40,715 313,571 30 based on (le Allocating of maintenance | $5,357 ($53,571) $24,107 ‘$24,107 $0 costs based on Maintenance hours (10% :45%:45%) Total “7 80 ‘30 $89,822 $52,678 | $142,500 ‘YReciprocal method should be used when management is using the results of cost allocations to make decisions on pricing products because it is the most accurate method of cost allocation ‘vVery important note: if the exam gives three or more service departments then you need to resolve three or more equations, in this case chose the answer which give results of O.H costs that exceed the original O.H cost for each service department (original cost for power and maintenance were $62,500 and $40,000 and the cost belong to these departments after equation's solution became $67,857 and $53,571 respectively) Homework (easy level) QU: Which of the following methods is generally considered the most acceptable method of allocating service department costs? ‘A. Step-down method B. Activity-based method C. Reciprocal method —D. Direct method Q2: A firm wants a simple method to allocate costs from service departments to production departments but never needs to allocate costs from one service department to another. Which of the following methods would satisfy these requirements? A. Activity-based method B. Direct method C.Reciprocal method —_D. Step-down method Q3: Render Inc. has four support departments (maintenance, power, human resources, and legal) and three operating departments. The support departments provide serviees to the operating departments as av well as to the other support departments. The method of allocating the costs of the support departments that best recognizes the mutual services rendered by support departments to other support departments is the A.Step-down allocation method _B. Dual-rate allocation method C. Reciprocal allocation method. Direct allocation method Q4: A company's maintenance department painted its tennis ball department, the racquet ball department, and its own department. The maintenance department uses square feet as its cost driver. ‘The departments respectively have 80,000, 20,000, and 1,000 square feet of space. The total maintenance department costs before allocation are $60,000. Using the direct method of allocating service department costs, how Touch cost is allocated to each department? A. Maintenance department: $594; tennis balls: $47,525; racquet balls: $11,881 B. Maintenance department: $0; tennis balls: $36,000; racquet balls: $24,000 C. Maintenance department; $0; tennis balls: $48,000; racquet balls: $12,000 D. Maintenance department: $594; tennis balls: $35,644; raequet balls: $23,762 Fact pattern: Bluebird Enterprises has two production departments (P1 and P2) and two service departments ($1 and $2). A breakdown of current period costs and service usage for each department is as follows: Description "Production departments Service departments PL P2 SI S2 Costs | $30,000 $40,000 24,000 $20,000 SI Services I 50% 30% Q 20% $2 Services 40% 40% 20% 0 Q5: If Bluebird uses the ditect method for allocation of service costs, what are the total service costs allocated to the two production departments (P1 and P2)? A, $25,000 to P1 and $19,000 to P2 B, $24,000 to P and $20,000 to P2 C. $20,000 to PI and $15,200 to P2_D. $19,800 to PI and $15,400 to P2 Q6: If Bluebird uses the step method for allocating service costs, allocating service department $1 first, what are the total service costs allocated to the two production departments (P1 and P2)? A. $19,800 to PI and $24,200 to P2 B. $24,400 to PI and $19,600 to P2 C. $20,800 to PI and $23,200 to P2_D. $25,000 10 Pi and $19,000 to P2 Q7: If Bluebird uses the reciprocal method for allocating service costs, what are the total service costs allocated to the two production departments (P1 and P2) (rounding all numbers to the nearest dollar)? A. $24,917 to PI and $19,083 to P2-_B. $25,000 to PI and $19,000 to P2 C. $24,400 to PI and $20,600 to P2.__D. $24,667 to PI and $19,333 to P2 Q8: A company using the step-down method of allocating service department costs has two service departments and two production departments. The amount of each department's costs is used to determine the relative value of each department to the firm for purposes of allocation. HR costs $100,000, and maintenance costs $150,000. These departments respectively use 8,000 and 10,000 labor-hours, Production departments have original department costs and labor-hours of $400,000 (40,000 labor-hours) for tennis balls and $300,000 (30,000 labor-hours) for racquet balls. What is the total cost for the tennis ball department if labor-hours are used as the sole cost driver? A. $475,065 B. $542,857 C. $514,286 D. $596,343 aA Fact pattern: M&P Tool has three service departments that support the production area. Outlined below is the estimated overhead by department for the upcoming year. [ Description Service departments Production departments Receiving ‘Repair Tool ‘Assembly Bolting | Estimated OF 25,000 35,000 10,000 2 2 # of employees 2 2 1 25 12 The Repair Department supports the greatest number of departments, followed by the Tool Department. Overhead cost is allocated to departments based upon the number of employees. Q9: Using the direct method of allocation, how much of the Repair Department's overhead will be allocated to the Tool Department? A. $875 B. $11,667 C. $7,000 D.$0 Q10: Using the step-down method of allocation, the allocation from the Repair Department to the ‘Tool Department would be A.$0 B. $11,667 C. $7,000 D. $875 QU1: A manufacturing company has the following information for its service departments, SI and $2, and its production departments, P1 and P2. Description SI S2 PI P2 Overhead cost $4,200 $7,200 $8,000 $10,000 | Service providedby SI | E | 30% 30% | 40% Service provided by S2 25% - 30% 5% Required allocate service department's cost to production departments using the three common allocation methods. Q12: Render Inc. has four support departments (maintenance, power, human resources, and legal) and three operating departments. The support departments provide services to the operating departments as well as to the other support departments. The method of allocating the costs of the support departments that best recognizes the mutual services rendered by support departments to other support departments is the A. Direct allocation method. B, Dual-rate allocation method. C. Step-down allocation method. _D. Reciprocal allocation method. Fact pattern: Logo Inc. has two data services departments (the Systems Department and the Facilities Department) that provide support to the company’s three production departments (Machining Department, Assembly Department, and Finishing Department). The overhead costs of the Systems Department are allocated to other departments on the basis of computer usage hours. The overhead costs of the Facilities Department are allocated based on square feet occupied (in thousands). Other information pertaining to Logo is as follows. 98 { Department Overhead ‘Computer Usage ‘Square Feet Hours Occupied I Systems $200,000 300 1,000 | Facilities 100,000 900 600 Machining 400,000 3,600 7,000 Assembh 550,000 1,800 3,000. Finishing 620,000 2,700 5,000 9,300 11,600 Q13: If Logo employs the direct method of allocating service department costs, the overhead of the Systems Department would be allocated by dividing the overhead amount by A. 1,200 hours. B. 8,100 hours. C. 9,000 hours. D. 9,300 hours. Q14: If Logo employs the step-down method of allocating service department costs and begins with the Systems Department. Which one of the following correctly denotes the amount of the Systems Department's overhead that would be allocated to the Facilities Department and the Facilities Department’s overhead charges that would be allocated to the Machining Department? [ Choice Systems to Facilities Facilities to Machining | = A $0 $20,000 B $19,355 $20,578 ie $20,000 $20,000 D $20,000 I $24,000 | Faet pattern: Adam Corporation manufactures computer tables and has the following budgeted indirect manufacturing cost information for next yeat. Description Support Departments Operating Departments | Total Maintenance | Systems| Machining | Fabrication Budgeted overhead | $360,000 _[ $95,000 | $200,000 | $300,000 | $955,000 Support work furnished | [From Maintenance 10% | 50% 40% 100% From Systems| 5% [45% 50% 100% | QIS: If Adam uses the step-down method, beginning with the Maintenance Department, to allocate support department costs to production departments, the total overhead (rounded to the nearest dollar) for the Machining Department to allocate to its products would be A. $415,526 B. $422,750 C. $442,053 D. $445,000 Q16; If Adam uses the step-down method; what is the total amount of service department costs that would be allocated to Fabrication department? A. $212,947 B.$422,750 C. $442,053. D. $445,000 Q17: If Adam uses the direct method to allocate support department costs to production departments, the total overhead for the Machining Department to allocate to its products would be A.$418,000—-B.$422,750 C. $442,053. D. $445,000 ‘Homework solution QI: Answer (C) is correct: The reciprocal method fully recognizes all interdepartmental service costs, allocating all costs to other affected departments using complex algorithms Q2: Answer (B) is correct: The direct method is the most direct and simple method of allocating service department costs. This method can't be used to allocate costs to other service departments. Q3: Answer (C) is correct: The method of allocating the costs of the support departments that best recognizes the mutual services rendered by support department to other support departments is the reciprocal allocation method. Q4: Answer (C) is correct: The direct method does not take into account any amount allocated to another service department, even itself, so nothing is allocated to the maintenance department and the square feet for this department are ignored. $60,000 Proportion of maintenance costs allocated to the tennis ball department = * 80,000 S.F= 100,000.8.F $48,000 ; ; __ $60,000 - Proportion of maintenance costs allocated to the racquet balls department =x. 20,000 S.F= 100,000.S.F $12,000 Q5: Answer (A) is correct Description Production departments | Service departments PI P2 SI S2 Costs $30,000 | $40,000 24,000 $20,000 SI Services 50% 30% 0 20% S2 Services 40% 40% 20% 0 S1 cost allocation 248 53 15,000 9,000 (24,000) $2 cost allocation 208 _ 4:4 10,000 10,000 (20,000) Total cost allocated 25,000 19,000 Q6: Answer (B) is correct: Description Production departments | Service departments PL P2 SI S2 Costs $30,000 | $40,000 24,000 [$20,000 SI Services 50% | 30% 0 20% S2 Services [40% 40% | 20% 0 Si cost allocation 24/10 5: 3:2 12,000 7,200 (24,000) 4,800 $2 cost allocation 24.80/8 > 4: 4 12,400 12,400 (24,800) ‘Total cost allocated 24,400 19,600 QT: Answer (A) is correct: ¥S1= 24,000 + 20% S2 ¥S2= 20,000 + 20% $1 S1= 24,000 + 20% (20,000 + 20% $1) S1=24,000 + 4,000 + 0.04 $1 ‘Thus $1= 29,167 and $2 = 25,833 Deseription Production departments | Service departments PI P2 si | 82 Costs $30,000 | $40,000 24,000 $20,000 SI Services 50% 30% 0 20% 52 Services 40% 40% 20% 0 ST cost allocation 5:3:2 14,584 38.750 (29,167) 5,833 $2 costallocation 4:4:2 10,333 10,333 5,167 (25,833) Total cost allocated 24,917 19,083 _] Q8: Answer (B) is correct: The amount of each department's costs is used to determine the relative value of each department to the firm for purposes of allocation, Thus we start with Maintenance department. Description Service departments | Production departments HR | Maintenance | tennis balls | racquet balls Costs $100,000 | $150,000 | $400,000 | $300,000, Labor hours 8,000 10,000 40,000 30,000 Maintenance cost allocation 8:40:30] 15,385 | (150,000) 76,923 37,692 HR cost allocation 4:3 115, 385 = 65,934 49,451 | Total cost allocated 142,857 107,143 | Total OH 542,857 407,143 Q9: Answer (D) is correct: The direct method of service department cost allocation does not recognize the servicing of service departments. All services are assumed to go to production departments only. Therefore, no Repair Department costs would be allocated to either the Tool Department or the Receiving Department. Q10: Answer (D) is correct: The step-down (sequential) method of service department cost allocation recognizes inter-service department services in one direction only. Reciprocal services are ignoted Description Service departments Production departments Receiving @) | Repair (1) | Tool) | Assembly | Bolting || [Estimated OF 25,000 35,000 10,000 2 ? # of employees 2 2 I 25 12 Repair cost allocation 1,750 (35,000) 875 21,875 10,500 21:25:12 Tool cost allocation 358 (10,875) 6971 3.6 2:25:12 Receiving cost allocation (27,308) 18,451 8.857 25:12 Total allocated costs 47,297 22,703 | QUI: 1. Direct method Service Department Production Department Description [st 82 PL P2 Total Overhead cost $4200 $7,200 $8,000 $10,000 | $29,400 Allocating S1 (3:4) (4,200) 1,800 2,400 0 Allocating 82 (3:4.5) (87,200) 2,880 4,320 0 Total 0 0 12,680 16,720 $29,400 2. Step down method | Service Department Production Department Deseri SI 32 PL P2 Total Overhead cost $4,200 $7,200 $8,000 $10,000 | $29,400 Allocating S1 (3:3:4) (4,200) 1,260 1,260 1,680 0 Allocating S2 (3:4.5) - ($8,460) 3,384 3,076 0 Total 0 0 12,644 16,756 | $29,400 3. Reciprocal method S1= 4,200 + 25% $2 (1) S2= 7,200 + 30% SI (2) By solving the equations SI= 6,486 $2= 9,146 Service Department Production Department Description SI 82 Pl P2. | ‘Total Overhead cost $4,200 37,200 $8,000 $10,000 | $29,400 Allocating SI (6,486) 1,946 1,946 2,594 o | 2,286 (6,146) 2,744 4,116 0 - : 12,690 16,710 | $29,400 Q12: Answer (D) is correct. QS: Answer (C Q13: Answer (B) is correct. QU: correct. Q16: Answer (C) is correct. QU nswer (D) is correct. nswer (D) is correct. Q: A company has two service departments (S; and $2) and two production departments (P1 and P2) expected departmental data for the coming year are as follows: Description Si | Estimated factory O.H $27,000 $18,000 | Services provided ByS; 10% By Sz 20% a Services provided to P, 30% 30% Services provided to P2 40% 50% ‘What are the total allocated service department costs to P2 if the company uses the reciprocal method of allocating its service department costs? Service Department Description Si Sz Production Department PI P2 Estimated factory O.H cost $27,000 $18,000 z 7 Service provided by SI - 10% 50% 40% Service provided by $2 20% i - 30% 50% 27,000 +20% 82 (1) Si $18,000 + 10% S) Q) By solving the equations = $27,000 + 20% ($18,000 + 10% $3) $1 =$27,000 + $3,600 +.02 $31,224 By plugging the amount of Sy to equation (2) Sx= $18,000 + 10% * $31,224 So= $21,122 Description Si Sr Py P Total Factory O.H. Costs $27,000 | $18,000 $45,000 ‘Allocation of 8; (10% -50% - 40%) ($31,224) | $3,122 $15,612 | $12,490 $4,224 ‘Allocation of S2 (20% - 30% - 50%) (821,122) $6,337 | $10,561 Total Factory $21,949 $45,000 $23,051 Q: A public accounting firm has two departments, Management Consulting Services (MCS) and Tax Advisory Services (TAS) these two departments use the services of two service departments, Computer Programming (CP) and Computer Operations (CO) The % of each service expected to be used by each department for a typical period are: Description a cP co MCS TAS | cP = 30% 30% 20% | [ co 25% y 45% 30% nanagement consulting and tax advisory services on the basis of estimated costs Based upon this information, the most appropriate method for allocating service department costs is the A, Physical-units method. B. Step-down method. C. Massachusetts Formula. D. Reciprocal method. Answer (A) is incorrect because the physical unit's method is not a service department cost allocation method. It is a method for allocating joint costs to the main products. Answer (B) is incorrect because the step-down method gives only partial recognition to services rendered by service departments to other service departments. Once a service department's costs have been allocated, the costs of subsequent service departments are not reallocated to it, Answer (C) is incorrect because the Massachusetts Formula is only used when there is no apparent causal relationship. But In this instance, the relationship between the service departments and the operating departments is known. Answer (D) is correct. The reciprocal method uses simultaneous equations to allocate costs by vices rendered among all departments. Because it acknowledges all explicitly recognizing the mutual s sources of cost, it should be used when management is using the results of allocations to make decisions on pricing products. Essay: The Peters Company Ltd. consists of four departments: Personnel and Purchasing, which are service departments; Machining and Painting, which are production departments. Budgeted data for the year consisted of the following: Item Personnel Purchasing |” Machining Painting Overhead Costs $40,000 $35,000 $128,000 $48,000 | ¥of Employees 10 15 5 Be Machine Hours 0 0 35,500 5) Direct Labor Costs | 0 0 $32,000 $147,000 Purchase Requisitions 6 2,650 450 Required: A) Allocate the two service departments’ costs using the step-down allocation method, and develop departmental overhead rates for each of the production departments using the most. Logical base for each department B) Calculate and give the journal entry to record the total amount of under- or over-applied overhead for the company, assuming the following actual results for the year: ©) Briefly discuss reasons for allocating service department costs. Deseription Machining Painting Overhead Costs $173,000 $78,540 Direct Labor Costs 338,300 $139,750 Machine Hours | 24,150 29 | Item Personnel | Purchasing | Macl 2 l Painting Overhead Costs 340,000 | $35,000 | $128,000 $48,000 # of Employees 10 15 23 | 138 Machine Hours 0 0 23,500 | 29 Direct Labor Costs 0 0 $52,000] $147,000 Purchase Requisitions 6 0 2,650 50 Allocation of Personnel based on # of (40,000) | 3,371 3618 | 31,017 Employees | Allocation of Purchasing based on Purchase = 68,371) 32,801 5570 Requisitions Total OH $166,419 | $84,581 | Estimated activity of Cost 4 23,500 | $147,000 | MH | DL. Cost Predetermined O.H rates $7.0817 | $0.57538 Q.H Accounts Description | Machining T Painting ‘Actual(DR) | Applied (CR) ‘Actual (DR) ‘Applied (CR) On $173,000 $7.0817 * 24,150 | $78,540 30.57538 * | | Mua =s171,023 $139,750 = $80,409 { Deseription ] ‘Actual 0. } ‘Applied O.H j Total $173,000 + $78,540 = $171,023 + $80,409 = $251,432 | $251,540 | i | Under applied 0.4 = $108 DR factory O.H Applied $251,432 DR COGS $108 | CR factory 0.4 Control $251,540 ©) Reasons for allocating service department costs include: 1) Economic reasons: to provide information for economic decisions (e.g. how to allocate available capacity among products) 2) Motivation: to motivate managers and employees (e.g. 10 discourage using shared services unwisely) 3) Income and asset measurement for external parties: to cost products for financial reporting, purposes. 4) Cost justification: to establish a “fair” price. Q: Indirect and common costs often make up a significant portion of the cost of a product. All of the following are reasons for indirect cost allocation to cost objects except to A. Provide information for economic decision making. B. Justify costs for reimbu semem purposes. C. Measure income and assets for external reporting purposes D. Reduce total costs identified with products. Answer (D) is correet Indirect cost allocation to cost objects include the measure of income and assets for external reporting purposes, to justify costs for reimbursement purposes, and to provide information for economic decision making, clusion 1. Allocation of indirect costs is a distribution of costs that can't be directly assigned to the cost objects that are assumed to have caused them, thus Cost allocation is the process of assigning and reassigning non traceable costs to cost objects. It is used for those costs that can't be directly associated with a specific cost object the allocation process entails © Deciding how costs are to be aggregated (accumulated in cost pools) prior to allocation, © Choosing the cost objects, determining the direct and indirect costs that should be traced to the cost object, and @ Selecting the allocation base to distribute these costs through cost drivers, preferably on a cause and effect criterion 2. Individual departmental rates rather than a plant-wide rate for applying overhead would be preferable if the manufactured products differ in the resources consumed from the individual departments in the plant. Revision Fact pattern: Wilcox Industrial has three support departments, the Information Systems Department Jegal Department and the Personnel Department, and two manufacturing departments, the Machining Department and the Assembly Department. The support departments service each other as well as the ‘two production departments. Company studies have shown that the Personnel Department provides support t0 a greater number of departments than does the legal and Information Systems Departments. QL: If Wilcox uses the direct method of departmental allocation, which one of the following cost allocations would occur? Some of the costs of the A. Personnel Department would be allocated to the Information Systems Department. B. Machining Department would be allocated to the Information Systems Department, C. Information Systems Department would be allocated to the Assembly Department. D. Assembly Department would be allocated to the Machining Department. Answer (C) is correct. Q2: If Wilcox uses the step-down method of departmental allocation, which one of the following cost allocations would not occur? Some of the costs of the A, Personnel Department would be allocated to the Information Systems Department. B. Information Systems Department would be allocated to the Personnel Department. C. Personnel Department would be allocated to the egal Department. D, Personnel Department would be allocated to the Assembly Department and the Machining Department. Answer (B) is correct. Q3: Which one of the following departmental allocations is present in the reciprocal method of departmental allocation? The costs of the A. Assembly Department are allocated to the Information Systems Department and the Personnel Department. B. Information Systems Department is allocated to the Machining Department and the costs of the Machining Department are allocated to the Assembly Department. C. Personnel Department is allocated solely to the Information Systems Department. D. Information Systems Department is allocated to the Personnel Department, Machining Department, and Assembly Department. Answer (D) is correct. Fact pattern: Robin Manufacturing Company has two produetion departments (metal and chrome) and theee service departments (administrative, product support, and maintenance). The following table shows what each department costs were prior to allocation, plus the number of labor hours and the amount of space used by each department. The administrative and product support department costs have a cause-and-effect relationship to labor hours, while the maintenance department costs have a cause-and-effect relationship to square feet. SS | Production Department Description Administrs Maintenance capac Metal Chrome Department | Department | yovPhO"", | Department | Department Department costs before allocation | $150,000 $397,500 $50,000 600,000 | $300,000 | Labor hours 1,000 6,000 8,000 16,000, 10,000 | ‘Space (sq. f.) 15,000 500 10,000 60,000 30,000 L 1 _! Using the step-down method for allocating service department costs to production departments, calculate the total costs allocated to each production department. Answer: the sequence of service department cost allocation is based on the % of services rendered to the other service departments ‘YAdministrative would provide 35% Of its services to other service departments ¥ Maintenance would provide 21.7 % Of its services to other service departments vProduct Support would provide 21.2 % of its services to other service departments 1 luction Department Service Departments neneanaan Description ‘Administrative | Maintenance | 2Youuet Met | Chrome Department | Department | yeoitment | Department | Department Department costs before allocation ‘$150,000 $97,500, $50,000 "$600,000 | $300,000 Tabor hours 7,000 6,000 8,000 16,000 10,000 Space (oq. RY 15,000) 300 70,000 60,000 30,000 Allocation of Administrative costs | ($150,000) $22,500 $50,000 360,000 $37,500 Allocation of Maintenance costs ($120,000) $12,000 $72,000 | $36,000 Allocation of Product (92,000) oceee eree Support costs Total ‘$788,615 | $408,885 Q: A company with three products classifies its costs as belonging to five functions: design, production, marketing, distribution, and customer services. For pricing purposes, all company costs are assigned to the three products. The direct costs of each of the five functions are traced directly to the three products. The indirect costs of each of the five business functions are collected into five separate cost pools and then assigned to the three products using appropriate allocation bases. The allocation base that will most likely be the best for allocating the indirect costs of the distribution function is ‘A. Number of customer phone calls. B. Number of shipments. C. Number of sales persons. D. Dollar sales volume. Answer (A) is incorrect because the number of customer phone calls has little relation to distribution. It is probably more closely related to customer service. Answer (B) is correct. The number of shipments is an appropriate cost driver, A cause-and-effect relationship may exist between the number of shipments and distribution costs. Answer (C) is incorrect because the number of sales persons is not related to distribution, It is more closely related to marketing. Answer (D) is incorrect because the dollar sales volume is not necessarily related to distribution. It is more likely related to marketing. Q: A corporation allocates indirect corporate overhead costs to its operating divisions. The company uses a cause-and-effect criterion in the selection of appropriate allocation bases. Which of the following would be an appropriate allocation base to assign the costs of the corporate personnel department to the operating divisions using a cause-and-effect criterion? A. Number of employees in each division. B. Square footage of space occupied by each division, C. Total service years of employees in each division. D. Total book value of identifiable division assets. Answer (A) is correct. The cause-and-effect criterion seeks a relationship between cost and the cost objective (for example, an operating division) such that changes in total costs can be predicted based on activities of the cost objective. Thus, the number of employces in an operating division is likely to correlate with incurrence of costs by the personnel department. Answer (B) is incorrect because square footage would be more appropriate for allocating building and maintenance costs than personnel costs. Answer (C) is incorrect because total service years of employees in each division are not a basis for predicting changes in personnel department costs. Answer (D) is incorrect because total book value of identifiable division assets is not a basis for predicting changes in personnel department costs. Departmental factory overhead application rates Q: John Sheng, cost accountant at Starlet Company, is developing departmental factory overhead application rates for the company’s tooling and fabricaling departments. The budgeted overhead for each department and the data for one job are shown below. ye Factory O-H cost item Tooling Department ‘Supplies $850 Supervisors’ salaries 1,500 Indirect labor 1,200 Depreciation 1,000 Repairs 4,075 Total budgeted factory overhead 38,025 $16,120 Using the departmental overhead application rates, total overhead applied to Job #231 in the Tooling and Fabricating Departments will be 620 $225 B, $303 C.$537 Ds671 Factory OH cost item ‘Tooting Department Fabricating Department ‘Total budgeted factory overhead $8,625 346,120 Total budgeted direct labor hours 460 3875 per DEH Direct labor hours on Job # 231 Factory O.H appli 12 DLH 12 * $18.75 per DLI 620 $26 per DLW. "3 * $26 per DLH = $78 Answer (B) is correet. Total overhead applies od to Job #231= $223+ S18 $303 Q:A profitable company with five departments uses plant-wide overhead rates for its highly diversified operation. The firm is studying a change to either allocating overhead by using departmental rates or using activity-based costing (ABC). Which one of these two methods will number of cost allocation bases and more accurate costing results? ikely result in the use of a greater f Choice Greater Number of Allocation Bases More Accurate Costing Results A Departmental Departmental B Departmental ‘ABC € ‘ABC Departmental D ABC ‘ABC Answer (D) is correct Service Departments cost alloc: n to operating Departments under responsibility accounting Fact pattern: Danny Ltd. has three service departments, Corporate Wide Advertising, Legal, and Human Resources that are respectively allocated based on revenue, usage and number of employees formation regarding all of the departments is as follows: | Description | Service Departments Operating Departments Total | Advertising | Human | Legal A | B c Resourees {Revenue - =| $3,766 | $4,635 | $3,970 [$12,371 (thousands) | Ussee of legal 5% | 10% 10% | 30% 45% 100% services Employees 16 2 12 140 175 35 400 Costs, before $1,450 $1,885 | $950 | $2,690 | $3,434 | $2,835 13,244 allocation (thousands) Required: 1. Using the direct method, compute the costs allocated to the operating departments, A, B, And C 2. Using the step down method, compute the costs allocated to each operating department. Assume that Human Resource costs are allocated to the advertising and legal departments, along with departments A, B and C and that subsequently legal c A,B, and C. Advertising is not allocated to any other service department. Answer ct method are allocated to advertising and departments Deseription Service Departments Operating Departments Total Advertising | Human | Legal A B c | Resources Costs, before 31450 31885 g050 | $2,690 $3434 | $2,835 13,244 allocation {Usage of legal | 5% 10% 10% 30% Wy 700% | services Employees 16 22 12 140 175 =| 35 400 Revenue 5 38,768 T4635 wero] saa Advertising, (1,450) | 441,411 543,267 465,322 - | based on revenue TLR based on # (77886,000) | 754,000 | 942,500 | 188,500 = of employees | Legal based on (60,000) | 111,765 | 335,294 502,941 - Usage wy Total cost = : = [ 3997176 | 5,285,061 | 3,991,763 | 13.244,000 | | 2. Step down method Deseription Service Departments ‘Operating Departments Total] ‘Ravertising | Human | Legal aA BT © Resources Costs, before | $1,460,000 | “$1,885,000 | $950,000 | $2,690,000 | $3,434,000 | $2,895.00 | 13,244,000 allocation | Usage oftegar Be 1% 10% 30% 18% 100% services Employees 16 2 12 140 175 35 400 Revenue - > z 33,766,000 | $4,635,000 | $3,970,000 | $12,371,000 | HER based on # 79,788 | (1,885,000) | 59,841 | 698,148 | 872,685 | 174,538 - of employees | LegaTbasedon 756,102 > 112,205 | 336,614 | 504,920 | Usage (1.909.841) Advertising based | (1,585,890) - - 482,779 | 394,180 508,931 on revenue Total cost - - = [3,983,132 | 5,237,479 | 4,023,389 | 13,244,000 1 ‘See Gleim MCQ # 453,434,554,555,558,559,560,561,562,564,565,566,567,568,569,570,571,573,574 Single rate V.S. dual rate EX: The Photocopying department provides photocopy services for both departments (A) and (B) and has prepared its total budget using the following information for next year. Budgeted ‘Actual Description | Department (A) | Department (B) Department (A) Department (B) Fixed costs $100,000 $110,000 Variable cost $0.03 per page copied $0.032 per page copied Usage 1,200,000 page | 2,400,000 page | 1,400,000 page 2,100,000 page : Assume that the single-rate method of cost allocation is used and the allocation base is budgeted usage. How much photocopying cost will be allocated to depattments (A) and (B) in the budget and How mu photocopying cost will be allocated to departments (A) and (B) during the year? Predetermined O.H. Application Rate under single rate method $100,000 R= [~~ + $0.03] = $.0577" © ,600,000page ] per page Department Allocated O.H -Budget Allocated O.1- Actual & 1,200,000 pages * $.0577' = $69,333 1,400,000 pages * $.0577' = $80,888.78 B) 2,400,000 pages * $.0577"= $138,667 | 2,100,000 pages * $.0577= $121,333.17 Total O.H allocated | ‘$208,000 $202,221.95 Q: Assume that the dual-rate cost allocation method is used and the allocation base is budgeted usage for fixed costs and budgeted rate for actual usage for variable costs. How much cost would be allocated to departments (A) and (B) during the year? Predetermined fixed O.H. Application Rate ___ $190,000 . = S00 _- so.o27r 3,600,000 page 77 per page Ry = $0.03 per page Department ‘Allocated fixed O.HL Allocated variable 0.1. Total (A) 1,200,000 pages * $0.0277 '= | 1,400,000 pages = $0.03 = $75,333 $33,333 $42,000 ©) 2,400,000 pages * $0.0277'= | 2,100,000 pages $0.03= $129,667 $366,667 $63,000 Total $100,000 $105,000 $205,000 Appropriate allocation basis of service department costs to operating departments Qh: departments by using rhe variable costs of Photocopying (service) departments should be allocated to production A. Actual short-run output based on predetermined rates .Ex (2,100,000 pages * $0.03) = $63,000 B, Actual short-run output based on actual rates. Ex (2,100,000 pages * $0.032) = $67,200 C. The service departments expected costs of long-run eapacity. D. The service department's actual costs based on actual services. Ex (2,100,000 pages * $0,032) = $67,200 Answer (A) is correet. The most appropriate method of factory overhead allocation of variable service department costs to production departments is to multiply the actual usage" of the user department by the predetermined rate. This basis establishes the user department's responsibility for the actual usage at the predetermined rate. Answer (B) i incorrect because the actual rate may differ substantially from the estimated rate, and. the user department usually has no control over the actual rate. Answer (C) is incorrect because the capacity costs of the service department should be allocated by a fixed overhead rate or lump-sum charge based upon the capacity needs of the user department. Answer (D) is incorrect because the user department does not control service department costs. Q2: The fixed costs of service departments should be allocated to production departments based on $100,000 A. Actual short-run use based on predetermined rates. Ex 3,600,000 2,100,000 pages = $58,333 $110,000 B. Actual short-run units based on actual rates. Ex 3,500,000 2,100,000 pages = $66,000 $100,000 SOOO 52,400,000 3,600,000 400,01 C. The service departments expected costs of long-run capacity. Ex pages = $66,667 D. The service department's actual costs based on actual use of services Ex $110,000 x 2100.000-pases. _ 565,999 3,500,000.pages Answer (A) is incorrect because fixed service department cost allocation based on actual short-run use ignores long run needs of the user department and makes service department accountable for capacity variances. Answer (B) is incorrect because allocating fixed service department costs based on actual short-run units or actual rates transfers any efficiencies (savings) or inefficiencies (overspending) of the service department to the user department. Answer (C) is correct. The fixed costs of service departments should be allocated to user departments in lump-sum amounts on the basis of the service department's budgeted costs of long-term capacity to serve, This basis allows the user department to develop (budget) a certain capacity needed from the service departments and to agree on the assessment of costs. Analysis of actual results permits evaluation of the service departments' ability to provide the estimated volume of service and any savings or overspending would be accountable at the service department's level. We Answer (D) is incorrect because allocating the service department's actual costs based on actual use of services transfers any efficiencies or inefficiencies of the service department to the production department, Q: Vincent Hospital has installed a new computer system. The system was designed and constructed based on the anticipated number of hours of usage required by the various hospital departments according to projections made by the departmental managers. Virtually all of the operating costs of the system are fixed. What would be the most systematic and rational manner in which to allocate the new computer system costs to the various hospital departments? A. To each department equally. B. By the anticipated number of hours of usage. C. By actual usage by each department. D. By the revenue generated in each department. Answer (B) is correct. Q: Which one of the following allocation approaches will ensure that the production (user) departments do not underestimate their planned usage of service at the start of the budget period as well as make the service departments cost efficient? A. The use of actual rates and actual hours for both fixed and variable costs. B. Budgeted rates and standard hours allowed for output attained for variable costs and budgeted rates and capacity available for fixed costs. C. The use of rates and quantities based on long-term historical averages for both variable and fixed costs. D. The use of a budgeted lump-sum amount based on estimates provided by the production departments for both variable and fixed costs, Answer (B) is correct. Q: Albert Hathaway recently joined Brannen University as the chief information officer of the University Computing Services Department. His assigned task is to help reduce the recurrent problem of cost overruns due to uncontrolled computer usage by the user community, while at the same time not curtailing the use of information technology for research and teaching. To ensure goal congruence, which one of the following algorithms should be used to allocate the cost of the University Computing Services Department to other departments within the university? A. Actual rate times actual hours of computer usage. B. Actual rate times budgeted hours of computer usage. C. Budgeted rate times actual hours of computer usage. D, Budgeted rate times budgeted hours of computer usage. Answer (C) is correct. Q: The management of ROX Company wishes to encourage all other departments to use the legal department, as circumstances warrant. To accomplish this, legal department costs should be A. Absorbed as a corporate expense B. Allocated to users on the basis of standard cost for the type of service provided C. Allocated to users on the basis of the actual cost of hours used. vy D, Allocated to users on the basis of the budgeted cost of actual hours used Answer (A) is correct: Since the management of ROX Company wishes to encourage all other departments to use the legal department, as circumstances warrant, and the legal department assists the organization as a whole, these legal department costs should be absorbed as a corporate expense and not be allocated to each department based on use. Q: A computer company charges indirect manufacturing costs to « project at a fixed percentage of a cost pool. This project is covered by a cost-plus government contract. Which of the following is an appropriate guideline for determining how indireet manufacturing costs are pooled to the cost pool? ‘A. Establish separate pools for variable and fixed costs. B. Assign prime costs and variable administrative costs to the same pool. C. Establish a separate pool for each assembly line worker to account for wages. D. Assign all manufacturing costs related to the project to the same pool. Answer (A) is correct. Cost pools are accounts in which a variety of similar costs are accumulated prior to allocation to cost objectives. The manufacturing overhead account is a cost pool into which various types of homogenous manufacturing overhead are accumulated prior to their allocation. Indirect manafacturing costs are elements of overhead that should be allocated to many cost pools. Ordina , different allocation methods are applied to variable and fixed costs, thus requiring them to be separated. Establishing separate pools allows the determination of dual overhead rates, As a result, the assessment of capacity costs (fixed costs), the charging of appropriate rates to user departments, and the isolation of variances are facilitated. Because Single rate method combines fixed and variable costs accordingly it does not consider different cost behavior patterns in allocating O.H. costs to eost objects. (Note: if the computer company has only this governmental contract, no need to establish separate pools for variable and fixed costs because all costs would ultimately allocated to one cost object - the governmental contract and the cost of establishing separate cost pools for fixed and variable ‘manufacturing costs would exceed the benefits under this case answer (D) would be correct) Answer (B) is incorrect because prime costs are direct costs, and variable administrative costs are period, not manufacturing cost. The question inquires about indirect manufacturing costs. ‘Answer (C) is incorrect because establishing a separate pool for each assembly line worker to account for wages is not necessary under most cost allocation schemes. in addition indirect wages represents one cost item of manufacturing O.H Answer (D) is incorrect because different allocation methods are usually applied to variable costs and fixed costs. See Gleim MCQ # 556,557, vA Essay: Franzel Company manufactures several different models of luggage In Its four manufacturing departments. These four departments and one service department (Equipment maintenance) are housed in one facility. Due to the distinctive characteristics of the costs and operations of the different manufacturing departments, departmental manufacturing overhead rates are employed for each manufacturing department. Franzel has been reviewing its operations and plans to implement some changes in the budgeting and reporting for the manufacturing departments during the next fiscal year. The Molding Department is being reviewed first, and its cost and operating data for six months of the current fiscal year are presented in the chart below. = the coming year; all of these costs are traceable (direct) costs of the Melding Department except for a formation has been accumulated to assist in developing the manufacturing overhead budget for portion of the fixed costs and the equipment maintenance costs. The fixed costs include the common building and operating costs which are allocated to each of the manufacturing departments on the basis of square feet. The Equipment Maintenance Department costs are charged to the eperating departments for services rendered. These costs represent the actual cost of parts and supplies ($150,000 current fiscal year) plus a charge of $50 per hour. The manager of the Equipment Maintenance Department determines the preventive maintenance schedule for each of the production departments; all other repairs are made on a “first come, first served” basis. Management is comfortable with using the six-month data as the basis for the preliminary budget estimates because most of the activity measures and costs appear to be representative. However, the following adjustments will have to be made for the coming year. Cost Item Cost Adjustment Indirect labor | Cost Increase of 8 % | Equipment maintenance | Cost increase of 10 % for parts, supplies, and the hourly charge | Fixedcoss | SS*~*~*~*~*~*~S~«S 860,000 per month In the past, the overhead application base in the Molding Department has been machine hours for all costs. However, management has used scatter-graphs to analyze the behavior of some costs and has concluded that there is more than one cost driver in this department. As shown in the scatter--graph on the next page which captures data for the 12-month period through June of this year, the number of setups clearly represents the behavior of the setup costs and is now the application base for these costs. Similarly, the cost driver that is most appropriate as an application base for material handling costs is pounds of material processed. Machine hours will continue to be used as an application base for the remaining costs of the Molding Department. Molding Department Manufacturing Overhead Activity Measures and Actual Costs Description [January | February | March | April May June Activity measures: Pounds of material 25,000 | 27,000 | 30,000 |~24,000 | 20,000 | 22,000 Machine hours 11,500 | 13,500 | 12,500) 8500 | 9,500 | 10,000 Units produced 50,000 | 45,000 | 32,000 | 42,000 | 48,000 | 40,000 Machine setups 5 20 16 18 1m | 2 Material handling costs ($1.40/Ib.) | $35,000 | $37,800 | $42,000 | $33,600 | $28,000 | $30,800 | Setup costs 11,500 | 15,250 | 12,000 [14,000 | 14,500 | 22,750 Machine hour costs | Indirect labor (85/machine br.) | 57,500 | 67,500 | 62,500 | 42,500 | 47,500 | 50,000 Power ($0.20/kwhr.) 21,250 24,250 22,750 16,750 18,250 i 19,000 Equipment maintenance 15,000 | 18,000 | 12,000 | 19,000 |~ 16,000 | 35,000 Fixed costs 135,000 | 335,000 | 335,000 | 335,000 | 335,000 | 335,000 Total overhead 475,250 | 497,800 | 486,250 | 460,850, 459,250 | 492,550 [Setup Costs $24,000 23,000 22,000 23,000 20.000 19,000 18,000 17,900 16,000 415,000 14,000 13,000 12,000 11,000 1 a6 rence, ue., January = 4, ete. 1820 Number of Setups: The numbers next to the points on the graph refer to the month of occur- 2 24 26 Jon Stein, vice president of manufacturing, has indicated that he wants to employ a responsibility accounting system where each manufacturing department will be held accountable for all costs included in its manufacturing overhead budget. Actual monthly costs will be compared to the annual budget, and each department will be expected to come in at or below budgeted monthily costs. Based on the production budgets, the activity measures for the Molding Department will be at the following levels for the coming fiscal year. [ ‘ActivityMeasure——S—S—=*&|SSSs« timated Annual Amount Pounds of material 280,000 pounds Machine hours 135,000 hours | Units produced 520,000 units ‘Machine setups 200 setups Power usage 2.5 kilowatt hours per unit produced Equipment maintenance 1,600 hours Required A. Identify the benefits of using scatter graphs in analyzing cost behavior B. 1) Using the cost data presented for the six-month period of the current year, adjusted for the estimated changes expected to occur and the estimated activity measures for the coming year, develop a manufacturing overhead budget for the Molding Department for the coming year. Use the high-low ‘method to determine the fixed and variable components of the setup costs. 2) Develop the overhead rate that should be employed in the Molding Department for the coming year for the costs to be applied on the basis of machine hours. C. Jon Stein has recommended that a responsibility reporting system be employed in the manufacturing departments and that departmental managers are held responsible for all manufacturing overhead costs included in the budget. Describe the likely behavior of the manufacturing departmental managers with respect to their accountability for equipment maintenance costs. Solution: A. the benefits of using scatter-graphs in analyzing cost behavior include the following. + Scatter-graphs provide a quick way of determining if'a causal (linear) relationship exists between costs and activities. + The use of scatter-graphs facilitates the identification of extreme observations (outliers or anomalies) that may require further analysis. In addition, scatter-graphs do not give much weight to outliers when determining the “line of best fit” while other methods may give too much weight to outliers, + Changes in cost behavior patterns are generally more noticeable when displayed on a scatter-graph. + Scatter-graphs can be easily understood as they rely on visual observation and represent a common sense approach. B. 1) the manufacturing overhead budget for the Molding Department of Franzel Company for the comting year is calculated as follows. ”) r Description ‘Amount ‘Material handling (280,000 Ibs. « $1.40 per Ib.) $392,000 | Setup costs [($750 = 200 setups) + ($250 x 12 months)] "7 153,000 | Indirect labor [735,000 hours * ($5.00 x 1.08)] 729,000 Utilities (520,000 units x 2.5 kwihe) * $20 per kw/hr] 260,000 Equipment maintenance | Parts and supplies ($150,000 * 1.10) 165,000 Labor [1,600 hours * (850 *1.10)] 88,000 Fixed costs ($360,000 per month = 12) 4,320,000 Total manufacturing overhead 6,107,000 | () Usi ng the high-low method to determine the fixed and variable components of the setup costs, the month of June must be treated as an outlier and dropped from the analysis. Therefore, the high month is February with 20 setups at a cost of $15,250, and the low month is January with 15 setups at a cost of $11,500. i Description Calculation Variable cost Per setup ] = Change In costs + Change in number of setups \ = ($15,250 - $11,500) + (20 - 15) = $3,750 + 5 = $750 Fixed cost ‘Per month T = Monthly cost - (Setups x $750) \ High month = $15,250 - (20 x $750) = $250 | Low month = $11,500 - (15 * $750) =8250 L 2. The overhead rate that should be employed in the Molding Department for the coming year for the costs to be applied on the basis of machine hours is $41.20 per machine hour, calculated as follows. Description ‘Amount of machine hours: Costs to be applied on the ba Indirect labor $729,000 Power 260,000 Equipment maintenance = Parts | 165,000 = Labor 88,000 Fixed costs 4,320,000 Total annual estimated O.H costs that related to M.H 3,562,000 Total annual estimated M.H 135,000 ML Predetermined O.H rate $41.20 per MH C. The basic premise underlying responsibility accounting is that the performance of departmental ‘managers will be judged by how well they manage those items directly under their control. The inclusion of equipment maintenance costs (as well as certain portions of fixed costs) violates this premise as the manager of the Equipment Maintenance Department determines the preventive maintenance schedule for each of the production departments. The inclusion of the equipment costs is likely to affect the behavior of the manufacturing departmental managers in the following ways. + The managers are likely to become frustrated and dissatisfied with not being able to control all the factors on which their performance will be evaluated. + Depending on the degree of pressure that is applied by top management to achieve the budgeted overhead expenses, an atmosphere of hostility could develop between the departmental managers and top management + Ifthe departmental managers are having difficulty controlling expenses in other areas, they may ask the manager of the Equipment Maintenance Department to postpone the scheduled maintenance. ‘These delays are likely to be detrimental to overall productivity in the long run. And an adversarial relationship could develop between the manufacturing departmental managers and the manager of the Equipment Maintenance Department. ABC Costing System EX: a factory company makes two products, (X) and (Z) for the period about to begin, 1,000 units of each product are to be manufactured. Assume that; the only relevant O.H item is setup cost, also also (X) assume that (X) and (Z) are expected to require 140 and 10 machine setups, respectively and (Z) are expected to require two and three M.H, respectively; and that the setup costs amounted to $6,000. If Factory applies (allocates) setup cost on the basis of M.H, the eross- subsidy per unit arising from this peanut-butter costing approach is determined as follows: description (x) @ ‘Total of units 1,000 1,000 2,000) # of machine setups 140 10 150 # MH. per unit 2 3 O1T- setup costs 36,000 (2) Traditional allocation (based on M.H).. $2.40 $3.60 Per unit (86,000 = 5,000 M.H) =$ 1.20 per MH. (2) ABC allocation based on # of machine setups 5.60 0.400 Per unit Cross subsidization $3.20 $3.20 Per unit Undereosted Over costed (A) Cost per setup = $6,000 + 150 setup = $40 per setup (B) Setup cost per unit of (X) = (140 setups x $40) + 1,000 units = $5.60 (C) Setup cost per unit of (¥) = (10 setups * $40) * 1,000 units = $0.40 YABC is one means of improving a cost system to avoid what has been called peanut-butter of resources that results in product-cost cross-subsidization (the condition in which the miscosting of one product causes the miscosting of other products) In a traditional system, direct labor and direct materials ate traced to products or service units, and a single pool of O.H costs is accumulated for a given organizational unit (plant-wide or departmental), and these costs are then assigned to cost objects [(X) and (Z)] using an allocative rather than a tracing procedure. The effect is an averaging (smoothing) of costs that may result in significant inaccuracy when products or service units use dissimilar amounts of resources. ‘vThe use of ABC normally results in substantially greater unit O.H costs for low-volume products than is reported by traditional product costing, For example, the cost of machine setup may be the same for production runs of widely varying sizes ($40 per setup). This relationship is reflected in ABC system that allocates setup costs on the basis of the number of setups. However, a traditional sys m1 us ng an allocation base such as M.H may under-allocate setup costs to low- volume products. Many companies adopting ABC have found that they have been losing money on

You might also like